SlideShare uma empresa Scribd logo
1 de 190
Baixar para ler offline
USMLE'"'MEDICAL ETHICS:
THE IOOCASES
YOUAREMOSTLIKELYTO SEEON THE EXAM
OTHERBOOKSBYKAPLANMEDICAL
USMT.ETrStcp1Qbook,Third Edition
USMLETrStcp2 CK Qbook,ThirclEdition
USMI-E'n'Step2 ClinicalSkills,Third Edition
LrSN{l.F.rNrStcp3 Qbook,Third ldition
USMLE"Medical
The100
YouAreMost
on the
ConradFischer,M.D.
AssociateChiefof Medicinefor Education
SUNYDownstateSchoolof Medicine
CaterinaOneto,M.D.
P U B L I S H I N G
New York . Chicogo
Cases
Likelyto See
Exam
Ethics:
I ( A P t A N
USMLE" isa registeredtrademarkof theFederationof StateMedicalBoards(FSMI3)of
the United States,Inc.,andtheNationalBoardof MedicalExaniners@(NBMEo),neither
of whichsponsorsor endorsesthisproduct.
This publicationis designedto provideaccurateand authoritativeinformation in regard
to the sub.iectmattercovered.It is soldrvith the understandingthat the publisheris not
engagedin renderinglegal,accounting,or otherprofessionalservice.Iflegal adviceor other
expertassistanceis required,the serviccsof a competentprofessionalshouldbesought.
EditorialDirector:fenniferFarthing
Editor:CynthiaC.Yazbek
ProductionEditor:LeahStrauss
ProductionArtist: EllenGurak
Coverl)esigner:CarlySchnur
O 2006by ConradFischer
Publishedby KaplanPublishing,a divisionof Kaplan,Inc.
888SeventhAvenue
NewYork,NY 10106
AIl rightsreserved.Thetextofthis publication,or anypart thereol maynot bereproduced
in anymannerwhatsoeverwithout written permissionfrom thepublisher.
Printcdin theUnitedtare'of America
September2006
10 9 8 7 6 5 4 3 2 |
I 3-ISBN:978-I -4195-4209-|
l0-ISBN: l-4195-4209-5
KaplanPublishingbooksareavailableat specialquantitydiscountsto usefor salespro-
motions, employeepremiums, or educationalpurposes.Pleasecall our SpecialSales
Departmentto orderor for moreinformationat 800-6219621,ext.4444,ernailkaplan-
pubsales@kaplan.com,or writeto KaplanPublishing,30SouthWackerDrive,Suite2500,
Chicago,IL 606067481.
) o f
ther
;ard
not
her
:.
To:
Antonio Oneto
Lawyer,Politician, Humanist, Fatheq and Loyal Frienil
A man of honor and integrity
c.o.
To:
Truth
C.F.
:d
:s
),
Contents
INTRODUCTIONANDHOWTOUSETHISBOOK ........xiii
ABOUTTHEAUTHOR. .......xv
CHAPTERI Autonomy
CHAPTER2 CompetenceandtheCapacitytoMakeDecisions.......5
Definitions............ .................5
Minors................... .................5
PsychiatricPatients................. .............................8
Capacityto RefuseProceduresin anC)therwiseMentallyl)isabledPatient..................9
C H A P T E R 5 l n f o r m e d C o n s e n t . . . . . . . . . . . 1 1
A1lOptionsMustBeDescribed........ ................I I
All MajorAdverseEffectsMustBeDescribed.............. ...................l2
ConsentIsRequiredfor EachSpecificProcedure ...........................13
BeneficenceIsNot Sufficientto EliminatetheNeedfor Consent................................13
DecisionsMadeWhenCompetentAreValidWhenCapacityIs Lost..........................14
ConsentIsImpliedin anEmergency.................... ...........................15
ThePersonPerforrningtheProcedureShouldObtainConsent..................................l5
TelephoneConsentlsValid.............. ................16
PregnantWomenCanRefuseTherapy........................... ................17
InformedConsentfor a Never-CompetentPerson..................................................,.....17
CHAPTER4 ConfidentialityandMedicalRecords. ......... 19
Confidentiality..... ................t9
Releaseof lnformation....................... ...............20
GiveMedicallnformation to the Patientliirst,Not the Family....................................21
Releaseof Information to Governnental Organizationsand the Courts........,.,,........2l
BreakingConfidentialityto PreventHann to Others....... ..---.........22
MedicalRecords.. ......---.......22
CorrectinsMedical RecordErrors..... ...............23
CHAPTER6 Reproductivelssues. ........ 41
CHAPTERTOrganandTissueDonation ..........45
CHAPTERSReportablelllnesses .........47
CHAPTERl0 SexuallyTransmittedDiseases(STD$. ....... 5i
CHAPTERll Malpractice ........ 55
Definition ............................55
Deviationsfrom LocalStandardsof Care...................... .................56
Informed ConsentProtectionagainstLiability.... ...........................57
Informed Refusalls asImportant asInformed Consent. ...............58
PatientsMust Fully lnform the Physicianof Their MedicalProblems.........................59
RiskManagement ................59
MedicalErrors..... ................59
. t J
...25
...27
...33
...35
...37
..37
..37
..38
..38
4l
. 4 1
.42
.42
.42
.43
45
40
CHAPTER12Doctor/PatientRelationship ......... 6l
CHAPTERt4Doctor/DoctorRelationship ......... 7t
CHAPTERf5Experimentation.. .........75
PRACTTCEQUESTTONS ..........7s
ANSWERSANDEXPLANATIONS... .....129
47
t9
49
50
52
i2
5
;5
'o
7
8
9
9
9
Patient autonomy is the most fundamental principle underlying all health-care ethics.
Autonomy grants every competent adult patient the absolute right to do what he wishes
rvith his own health care.The concept of autonorny is fundamental to the entirety of the
U.S.legal system and has complete acceptanceas an operating principle of day_to_day
decisionmaking. Justice cardozo set this as a clear legar precedent in 1914, in the case
Schloendorffv. Societyof New york Hospital, 105 N.E. 92 (1914). JusticeCardozo wrote,
"Every
human being of adult yearsand a sound '.rind hasthe right to determine what shall
bedone with his own body and a surgeonwho performs an operation without his patient's
consentcommits an assault,for which he is liable in damages,exceptin casesof emergency
rvherethe patient is unconscious and where it is necessaryto operate before consent can
beobtained."lustice cardozo waswriting concerning the need for informed consentwhen
a person undergoes surgery, ln this caseit was made clear that to perform surgery on a
patientwithout his direct consent was equivalent to assaultand battery.
Patient autonomy is a concept derived fron the property rights issuesthat led to the
Declaration of Independence, the U.S. Constitution, and the Bill of Rights. Autonomy
overone'sown medical careis seenin the samelight asfreedom of religior, freedom fron.r
illegalsearchand seizure,freedom of speech,and freedom of assembly.Theserights are so
inlri,'icto our cullurethlt theyrrr conideredaxiomalic.Vithin the l.rrt30 yearrthey
havebeen legally extended to cover the freedon to chooseone's own form of health care.
Ior example,patients have the right to refuseundesired therapy,and they havethe right to
choosewhether or not they will participate in experimentation. Each patient has the right
to havehis wishescarried out even in the event that he losesconsciousnessor the caoacitv
to make dccisions for himsell
Chapter
'l
. Autonomy
I(APTAN MEDICAL
I
ChapterOne
ln part becausetrcatments such as mechanical ventilation and artificial nutrition didn't
exist in the past,r'e need nervlar^'sto delineatepreciselythe ethicsof health care.our abil
ity to devclop new forns of therapy hasoutpaccd our ability to createethical svstemsthat
detcrmine whethcr or not thesesvstemsr,villbe used in a specific case.
. Doesthe developmelltand existenceof a treatnent_such asplacementon a ventila_
tor-mean we should alwaysuseit?
. Docsthe fact that we canplacea nasogastrictube in a con.tatosepatient and admin_
ister artificial nutrition that cankecpthat patient alivealmost indefinitely rneanwe
shoulddo so?
Each patient should decidc the a'swers to tlresequestions for hinrsell rhis is autonomy:
I havethe right to do what I choosewith my own body aslong asI understand the consc
quencesofmy decisions.There is no form ofproperty more personal than one,sown bodyi
so each patient hasthe right to deternire what is done to his body. Theseprinciples may
seemobvious, but they are fundamental in medical ethics.This work is an examination of
the various cthical situations faced clailyby ph,vsicians.
Autonomy representsa patient's right to deter'.rine his or her own health-care decisions.
No form of a treatment can be pursuedrvithout his or lter agreerrent,even if the pro
posed therapy is
"for
her own good." Although beneficence-or doing rvhat is good for
peoplc-is a high ain and ethical principle, autonomy is considereclmore important and
takesprecedence.Eachpatient hasthe right to refusea treatment even ifthat treatment has
no adverseeffectsa'd will heip her.You cannot trcat him or her aqainsthis or her wil] even
if the treatment is for her benefit.
Think of your body as a piece of property such as your house or your car
.fhe
police do
not have the right to searchyour house at their *'him. They m.st obtain a warrant. They
cannot even do something benign, such as step inside vour honc, without your express
agreenent. Likewise,a physician cannot do a colonoscopv or a CBC without your consent.
Unlessyou agreeto it, a physician cannot evcn do a urinalysis or an EKG, eventhough these
aremild and uncomplicatedprocedures.
Along the same lines, a painter cannot repaint your house without your consent even if
he does it for free, the existing house color is ugly, anclthe servicewill only benefit you. A
physician cannot treat your pneumonia or remove a cancer even though the procedure or
treatment is benign and it will only benefit you.
A doctor can violate this autonomy if you are doing something potentially dangerous,
such asbuilding a bomb, becausethis activity can harm others.A doctor can violate your
MEDICAL
cn didn't
fur abil-
€ms that
/entila-
lmin-
n w e
cnomy:
' conse-
n body;
es may
tion of
:isions.
e Pro-
od for
rt and
nt has
I even
ce do
They
press
lsent.
these
en if
,u.A
'e or
ous,
our
onlyifyouhaveHIV,tuberculosis,or asexuallytransmitteddiseasebecause
conditionscanharm an innocent third party. Your right to autonomy is only limiterl
thereispotentialharm to an innocent third party. Even in thesecases,the violation
of@nfidentialityislinited andtightlycontrolled.
Theconceptofpatientautonomyissimilarto thatofvoting.Ballotcounterscannotmake
presumptionsbasedon rvhomtheythink you rvill vote for. They must definitelyknow
whomyouintendedtovotefor whenyou werecastingyour vote.This must bebasedon
whomyouspecificallyvotedfor Another personcannotbring your proxy ballot to the
electionbasedonwhichcandidatehethinks you shouldvotefor or which candidatewill
belpyouthemost.
Youmustcastyourvotein amannerbasedonyour clearlystatedwishes.Youhavearight to
votefortherveakestcandidate.Youhavethe right to makethe
"wrong"
choice.In thesame
way,adoctorcanonlyrendercarefor you basedon rvhatyou tell heryou want.
Patientautonoml givesyou the right to make the wrong choice about your health care.
Evenif youareunableto make or expressyour health-care choices,the physician cannot
simplymakedecisionsfor,vou that go againstyour wishes.
'l'his
standseven if your choices
wouldbeharmfulto you. This is one of tlre most difficult things for a physician to under-
standandacton. l)octors are trained to act in the Lrestinterest of their patients, but the
patient'sright to act againsthis orvn best interest comes first. Patients havethe autonomy
offreewill.
AutonomyI
!!!!p ueorcer
-il
Chapter2: Competenceandthe
Capacityto MakeDecisions
DEFINITIONS
Competenceis a legal term. Competency decisions transpire within the judicial s1'steni.
Onlya court can determine that a patient is incompetent. All adult patients are considered
conpetent unlessspecifically proven otherwise. Physicianscan determine whether or not
apatient hasthe capacity to understand bis medical condition.
The physician makes a determination of the capacity of a patient to comprehend her
rnedicalproblems basedon whether there is an organic delirium due to a medical condi-
tjon such as a sodium problem, hypoxia, drug intoxication, meningitis, encephalitis, or a
psychiatricdisorder.Thesedetcnninations arebasedin large part on a neurological exami-
nationtesting memory, comprehension, reasoning,and judgment. Any physician can make
this determination. The physician does not have to be a psychiatrist. A psychiatrist may
beusefulin rendering decision making capacity determinations in casesthat are complex
or equivocal.If the patient obviously does or does not have the capacity to understand, a
psychiatristis r.rotneeded.
MINORS
Bv definition, a minor is a person under the ageof 18.With some exceptions,minors are
generallynot considered competent to n.raketheir own decisions.Only a parent or a legal
guardiancan give consent for a minor. Neighbors, aunts, uncles,and grandparents cannot
giveconsentfor treatment of a minor. This rule doesnot cover life-threateningor seri-
ousemergencies.Consent is alwaysimplied for emergencytrcatment. A physician should
not withhold blood or surgery in a life-threatening accident just becausethe parent is not
Present.
I(APLAN MEOICAL
I chapterrwo
'
For example, a 10-year_oldboy accider.rtallyruns through a glasswi'dou, at school
and laceratesthe raclial artery. His teacher brings hirn-to the emcrgcncy dcpart_
mcnt. The boy is bleedingand needsboth a blood transfusionand surgeryto cor_
rect the defect.What should you clo?
Emergency treatment of a minor does not neeclexpressrrritten consent. parental consent
is implied. saying that you had .o ask a'other p".ror, ru.h as the tcacher, thc principal,
the school nurse, the babysitter,or the granclparentsfor consent belore givir.rgcmergency
treatment would be the wrong answer.Seekinga court ordcr i, olrn u tv.,,,.rglhoi."l,., ur.,emergency becauseit delaysthe treatment and becausein an emergencyit is implicd that
the parents rvould consent if they were there,
PartialEmancipation
Although only a parent or guardian can give consent for procccluresand therapies for aminor there are some exception_sto this rulc in thc areasof prenatal care,coutraception,
sexuallytransmitteddiscases(STDs),and substanceabusc.The mature minor is genera'y
one.abovethe ageof I5, although this raries by state.USMLE wilt not make hairsplitting
distinctions like giving you a r4-year-old the day before her r5th birlhday. Thc caseon the
exam will be clear
For example, a l6-year-old girl comes to seeyou in clinic to discusscontracep_
tion. She is generally healthy but is not accompanied bv a parent. What should
,voudo?
h all caseslike this involving prenatal care,STDs,contraccption, Hrv anclsubstanceabuse
the answer should bc to trcat rhe patient. Saying thut 1.ou_urt notily thc parents, get acourt order)seeklegalcounser,refusetherapy,or to go to the ethicscorr[]rttee areaI incor_
rect answers.These interventions
u'wantecrpregnan.y,,.i,i,.,,,ij:::i:: ffiI :;::XlTr:Xil.::,:"T,rj;;::,,,':,fthan it is to take the risk that a teenagcrwill get pregnant and necclan abortion later
AboftioninaMinor
The rules on parental notification for abortion are lesscrearbecarse there is no national
standard.Somestatesrequireparentalnotificationand somedon,t.
I(A'I.AN MEDICAL
rol
rt-
)r-
)nsent
rcipal,
'gency
in an
d that
fora
tion,
rally
rung
rthe
'uSe
3 t a
:or-
ing
IOn
nal
Competenceand the Capacityto MakeDecisions
Forexample,a 16-year-oldgirl comcsto seeyou in her first trimesterof preg-
nancy.Sheisseekinganabortion.Whatshouldyou do?
Inthiscase,thereisno clearansweraboutwhetheror not thephysiciansl.rouldnotif, the
parents.Thereisno clearnationalstandardandit dependsupon the stateyou arein. The
mostlikelyrightansrverwill indicatethenceclto encouragethc child to notify the parcnts
herself,whichwouldbebest.Sothe corrcctanswerchoicewill saysomethir.rglike,,encour-
agediscussion,""counsel
her to tell the parentsherself,"or
"suggest
a fan.rilymeeting.,'On
theethicsquestionsfor USMLE,if thereis a choiccthat savsto discuss,confer,meet,or
havevoluntarynotification,thisrvill generallvbethe right thirg to do first.
EmancipatedMinor
A smallnumber of minors, particularlyat older agessuchas l6 or 17,may bc considered
'emancipated'
or freed of the necd to haveparental consent fbr any medical care.The critc
ria arethat the minor is married, self-st4rporting and living indepenclently,in the military,
or theparcntofa child that they themselvessupport.The criteriafor beingan emancipated
n]inor relateto being no longer dependent on one's parentsfor support, In other words, if
theminor does not livc with his parents,has a.job, ar.rdis self-supporting financially, thelr
theminor no longeris dependentupon parentalconsentfor his actions,
An emancipatedminor is free to make health caredecisionsin all areas,not onh, just STDs.
prenatalcare,contraception,or substanceabuse.Seriousmedicalconditionsor procedures
suchas organ donation, surgery, or abortion may require a specific court order to allotv
the legal standing of enancipation to be fully valid. Only answer
"court
order,',
.,judicial
intervention,""court
trial," or
"seek
legal resolution" if the caserepresentsdisagreementor
alackof consensusin the stem.
LimitationsonParentalRightofRefusalforMinors
Although a competent adult can refuse any medical care she wishes,the same right does
not automatically extend for parents concerning their children. parents cannot refuse life-
saYingtreatment for their child based on religious beliel The state has an intercst in the
n'clfareof the child that exceedsthe parental righr to den1.therapy for the child if the child
misht die.
t ( A p tA N
-
MEDICAL
r I cnupt",r*o
For example,a child is in a motor vehicleacciclentand sustainsheadtrauma
requiringsurgeryto drain a hematomathat,if leftuntreated,will befatal.Aspart
of the surgerythe child will needa blood transfusion.The parentsareJchovah's
Witnessesand refuseto giveconsentfor the transfusion.Ihe parents'statedreli
giousbeliefsarethat acceptingblood for their child would be a fateworsethan
death.4rat shouldyou do?
If the child needsblood to savehis life you must givetheblood evenoverthe objection
of the parents.It may seemcontradictoryto seekparentalconsentfor a procedurethat
you will perform evenif they refuse,but in this case,you shouldattemptto obtain their
permissionnonetheless.
Withholding lifesavingtherapyfor a child is consideredcomparableto child abuse.The
parents'right to practicetheir religionin termsof healthcarewould covertheir abilityto
refusea transfusionfor themselves,but not for their child.
Thisethicalconcepthasonly expanded.Parentscannotrefusetherapyfor childrenevenif
theyareseverelybrain damagedor otherwisedevelopmentallydisabled.Societydoesnot
distinguishbetweenindividualsbasedon theirrelative
'worth.'In
otherwords,parentscan-
not refusetracheo-esophagealfistularepairon apatientwith Down syndromejustbecause
thementalcapacityandfunctionalabilityof thechildwill bemuchlessthanthat of achild
without this disease.From this point of view,treatmentagainstparents'wishesin a life-
threateningsituationisequallyvalidfor both afuturegeniusandachildwith cerebralpalsy
who will not achievea mentalageabove2.
One of the only timesparentsareallowedto refusecarefor their child is when the child
is soill or deformedthat deathis inevitable.This is not a true refusalon the part of the
parent.This is reallyjust sayingthat parentscanrefuseonly the futile carethat thedoctor
shouldn'tbegivingany.way.
PSYCHIATRICPATIENTS
A patient'spsychiatrichistoryisintrinsicto theconceptof competenceandto thepatient's
capacityto understandher medicalproblems.A patientwith the clearcapacityto under-
standor onewho clearlydoesnot havecapacitydoesnot needa psychiatricevaluation.
However,a psychiatricevaluationcanbe usefulto help makea determinationof capacity
in ecuivocalor questionablecases.
l (
n
I
s
c
t
$:."9
MrDicAL
:tion
that
:heir
a
t
The
y t o
:nif
not
?n-
rt's
)n,
ity
ruse
rild
ife-
nl,I
rild
the
tor
Competenceand the Capacityto MakeDecisions
All suicidalpatients are considered to lack capacity to understand becar.rseactive suicidal
ideationis deemedto be a sign of impaircd jr.rdgment.In addition, thc level of compctence
necessaryto make financial decisions is different fron.r that necessaryfor an informed
refusal.In other words, a patient rnay have a history of bipolar disorder making it impos-
siblefor him to manage his financial decisions. Howevel the same person might still be
consideredto have capacity to refuserrearment. There is r'virylimitiil iteiriiril liliced on
p,ricnlto estJblih(dp.tcilylo rcfuetrcdtntet.
CAPACITYTOREFUSEPROCEDURESINANOTHERWISEMENTAIIY
DISABTEDPATIENT
A patientwith mental illness or mental retardation that might be consideredincompetent
for other areasof life may still retain the right to refuse medical procedures. The criteria
to determineconlpetencein areasof finance are at a higher standard than those for refus-
ing nedical procedures. Your patient might have schizophrenia, mental retardation, or
autismto the point of needing to live in a group home, but that does not mean they are
incapableof understanding medical procedures.This means that an adult with a mcntal
ageof 8 or l0 nay still be allowed to refuse medical procedures.Our society is reluctant
to strapa patient to his bed and perform procedures that would be painful or uncomfort
ablefor the patient without his consent. For irlstance,certain court casesin the past have
allorveda patient with mental illness to refuse diagnostic procedures even though two
out of three of the reasonsfor the refusal were delusional. This is an affirn.ration of how
deepthe principle of autonony goesin the managenent of patients. In addition, it shows
that beneficence-trying to do the right thing for patients is considered lessimportant
than autonomy. Autonomy is given more wcight in decision making than beneficience.
Autonomy haspriority.
A personmay meet the legalstandard of competenceto refuseor acceptmedical careeven
if sheis not consideredcompetent in other arcasof life, such asfinancial matters.
MEDICAL
I r r
Att OPTIONSMUSTBEDESCRIBED
bu nrust full,vinforrn the patient of the risksanclbenefitsof eachprocedureprior to
undergoingthc procedure.Thc explanationmustbe in languagethatthe paticntunder-
standsandincludefull infbrnation regardingalternativetreatments.
'I'he
paticntcannot
makean informcd choicefor onetreatmentif shecloesnot latow of the existenccof oth-
For example, you inform a patient about the risks iurd benefits of bone marrow
transplantationfcrrchronic myelogenousleukemia.You fully inform the patient
about the risk of transplantation,including the possibility of deveJopinggraft
versushost disease.Afier the transplantationthe patientdevelopsgraftversushost
disease,which is hard to control. The patient learnsthat there is an alternative
treatmentcalledimitanib (gleevec),rvhichtloesnot includc the risk of graftversus
host disease,but which rvill not cure the lcukemia.The patient filessuit against
you. What rvill be the most likely outcomc of the suit?
In this casethe patient rvill probablywin the suit becausehe wasnot fully intbrned about
the alternativesto the therapics mentioned. The physician has an ethical duty to ir.rfbrm
thc patient about all the treirtmcnt options and then allorvthc patient to decidcamong
thcm. Although the phvsician'spref-erenceof procedure or treatment may differ from what
thc patient chooses,the patient has the option to choosetherapy that may not be ^rltatthe
doctordeemsis bestfor hin.
Chapter3: InformedConsent
l
i
I(APLAN M E D I C A L
ChapterThree
!13!9 MEDTcAL
AtLMAJORADVERSEEFFECTSMUSTBEDESCRIBED
Advcrse effectsanclinjury lrorn rnerlicalcaredo not necessarilyrepresenta mistakc or fail
urc of therapy. In the casedescribcd in the previous exantple,the error was not that graft
versushost cliseasedeveloped.Thc patient wasfully informed that this could occur and he
chose the bone marrow triursplantation anyway.The error rasnot informing thc patient
of an alternatir.eoption in treatmcnt. At the same time, a patient could potentially die as
an adverseeffect of treatment. This is only an ethical and legalproblen if the adverseevent
happcns and the patient was not told that it could have happened.The patient n.ight say,
"Doctor,
I would never havc taken digoxin if ,youhad told me it night causea rhythm dis
turbanceor visualproblenr"or'"I rvoulclneverhavehad surgeryifyou had told mc I might
nccd a bloocl transfusion." Thc m.rin point is tt' rcpcct 3utonolly. The patieitt must be
ir.rforn.redof the therapeutic options, the adverseeff-ectsof the procedure, and the harm of
not undergoingthe procedurc.If thcy havethe capacityto understandand thcy chooseto
do it ar.rvr,vay,they havemade an autonomous tlrerapelrtic choice,and therefore,thc patient
bearsthe burclenof any adversccffect,not the physician.
For example, a man underploescoronary angioplasty. He is inforned that tl.re
arterymay rupturc and that thereis a srnallchancehe couldbleedto dcathduring
the surgery to repair thc danaged vessel.He knorvs he could havebypasssurgery
instead.He understandsand choosesthe angioplasty.He dies from a ruptured
blood vessel.The fanily files suit against you. What will be the n.rostlikcly out
come?
Although it is unfortunatethal the patientdied in this case,thereis no liability with regard
to informed conscnt or ethical error.
'l'he
patielt was intbrmed of his trcatment options
ancithe possiblecomplications,anclhe chosethe treatment.
The patient mlrst undcrstand the risks of a procedure just as a drivcr rrust understand
the risks before getting behind the wheel of a car. Why can't you sLlca car manufacturer
if you die in a car accidcnt?Predominantlybecauseyou are an adult with the capacityto
ur.rderstandthe risks of driving anclyou choseto drive an1.way.Tl.reliccnsing processis an
education processthat both tries to make you a safedriver, while alsoproperly informing
you of the risks of driving. Each time you get in a car,there is i.r.npliedconsent that you are
choosing the risk of driving. Evcn if you €letinto a car accident and arreinjured or killecl,
the manufacturer has no liability, aslong asthe car is well ntade,trecauseasa compctent
adult you choseto put yourselfat risk.
tnformedconsent
I
tr
;raft
l h e
i€[t
e a s
r'ent
say,
A t .
Lght
t b e
r o f
e t o
ient
ard
)ns
.nd
rer
to
an
n8
lre
",]
rnt
In addition to understanding the risks of the procedurc, you must inforrn the patient of
whatcould happen if shedoes not choosethcrapy that you offer.
For example, a patient comcs to the emergencydcpartnent with appendicitis. He
is inforrned of the risks of surgery,and refusesthc procedurc both verbally and in
rvriting.
'l'he
patient dies.Vhat was done rvrong here?
Thepatientsmust be informcd both of the risk of thc treatmentasrvellasu,hatwill happcn
iftheydon'tundcrgothe procedure.In this casethe physicianis liablein court becausehe
neverdocunented that he infornred the patient of the possibility of appendiceal rupturc
anddcathif the patient did NOT havethc procedurc.
CONSENTISREQUIREDFOREACHSPECIFICPROCEDURE
If the patient signs a consent form for an opcration on hcr left knee, you cannot, in the
operatingroom, decide to operate on her right knee and assumethat you have conscnt. If
apatientsignsa consent for an appcnclectomy,but when you open her up you find colon
cancetyou cannotjust do the colectomvwithout first informing the patient of the acldi-
tionalprocedureand obtainingher consent.Thcre can be no presumptionfor conscntfor
anythingbeyond what the patient specifically saiclsheconsenteclto. lither the paticnt has
to signconsentin advancefor the other procedures or shehasto regain consciousnessand
havethe additional proccdure explained to her.
BTNEFICENCEISNOTSUFFICIENTTO
ETIMINATETHENEEDFORCONSENT
Trving to bc sincere anrl to do good is very important and takes primacy; howeter, the
patient'sright to control what happcns r.vithhis own body is more irnportant.
For example, a 40 year-old man is undergoing a nasalpolypectomy. In the opcrat-
ingroom you seea lesionon thc nasalturbinatethat the frozenscctiondetermines
to be a cancer,You have found the cancer early but will need to rcsect the nasal
turbinate to cure it. What should you do?
M E D I C A LI ( A P I A N
t 4
I chapterrhree
You cannot rcmo'e tlre cancerousresior.rrvithout the patient's approval. This is true evcn
if the physicianis sincere,tale'ted, accurate,and helpful.f'his is truc eve' if the procedure
will savethe patient'slife, unlessthe illnessis an emergencyr'nan unconscrotrspatrelt.
Beneficcncedoes not elirninate the need for informed consent. lf you live in a very messy
apartlnentyour neighborcannot breakinto your apartmentto cleanit el,enif he doesn,t
stealanvthil.lg.You must col]sent to the cleaning. His good intcntions are not asimportant
asyour right to do what you wanl n'ith your otvn property.
DECISIONSMADEWHENCOMPETENT
AREVATIDWHENCAPACITYISTOST
We must respectthe lastknown wishesof a patient if shelosesthe capacttyto con]munlcate
and state those r'r'ishes.Although it is preferable to have thc patient's last k'own wishes
docunented in writing, following verbally expressedwishesis perfectly valid. oral co'sent
is valid for arrylevelofproceclure if the orar consert can be pro'en. The basisfor varidit). of
oral or written conscntis not whctrrerthe procedureis largeor small.Ir.rother words,it is
not thc casetltat oral conscnt is valid for a stgmoicloscopybut a brain biopsy requireswrit_
tcn consent.A patient cal.tgive oral consent tbr ir heart transplant if the patient is unable
to *'rite.
'l'he
onlv diffrculty is that if challenged,orally exprcssedwishes for treatnrent are
more difficult to prove than lvritten ones.
For example, a 42-ycar-old nran with leukemia repeatcdly refuseschenrotherapy.
He losesconsciousless and his rnother tclls you to givc the chentotherapy.What
should you tell l.rer?
You must rcspectthe l.st kntxvn rvishcsof thc patient. If the paticr.rtcloesnot wa't a treat-
ment, you cannotiust $'iritfor him to loseconsciousnessand then perfornt the treatmcnt.
If this rverepermissible,ther no one courdhavean estatewiI. The ultimate form of ross
of decisitur naking capacitv is death. ,Veurarkeout a will so that when rve lose the capac_
ity to speukfor ourselr.es,our wishes for rvhat to do with our property are respectedafter
cleath.
For example, a 64-ycar-old roman accontpanied by her husband comes to the
emergency room seeking treatment for chest pain. The patient clearJytells you
that shewants to haveher aorta repaired and shesignsconsent for the procedure.
Shelater becomeshypoter.rsiveand losesconsciousness.Her l.rusbandis now the
decision maker and says,
"Let
her die.,,What do you tell him?
t
a
b
a
T
p
P
n
tl
T
s
T
s
o
Y(
OI
aI
I(A9IAN MEDICAL
Informedconsent
I ts
e eyell
:edure
rtient.
messy
oesn't
)rtant
LlCate
lshes
rsent
tyof
, t t l s
urit-
Lable
t are
Apatientt family member cannot wait for her to lose consciousnessand then go against
thepatient'sprevrous expressedwishes regarding treatments and procedures.In the case
above,becausethe patient exprcssedthat shewoulcr riketo haveher aorta repaired her hus-
bandcannotgo againstthis aftershclosesconsciousness.The samereasoningholds true if
apatientrefusesa procedure or treatnent and then losescolrsciousDess.
CONSENTISIMPTIEDINANEMERGENCY
For example, a sO-yearold construction worker arrives at the emergency room
by ambulance after an accident lacerating his arm. He has lost so much blood he
rsunconscious.There is no family member availableto sign consent.What should
youdo?
Thcmanagementof an emergency is different. Consent is implied rn an emergencyfor a
patientwithout the capacity to speakfor himsell This would not apply to a terminally ill
patientwith a pre existing DNR order Neither a court order, nor a hospital administrator,
nor the ethics committee is re<luired to give permission before the doctor can administer
roerap.m an emergency,
THEPERSONPERFORMINGTHEPROCEDURE
SHOUTDOBTAINCONSENT
Thepersonwho is mostknowleclgeableabouttheprocedurcshouldobtaininformedcon
sent.Becauscwe must inform the patientabout all the optionsof treatment,risksof the
options,and risksof not performingthe procedurein a languagethe patientcanunder-
stand,theconsentmustbeobtainedby a personqualificdto maketheexDlanation.
Forexemple,you arean intern who hasconsultedsurgeryto placea subclavian
centralvenousline,Youonlyknow accessmustbeobtained.you do not knorvwhv
theinternaljugularapproachisr.rotbeingused.On thephonethesurgicalresident
says,"Can
).ougo getthe consentrvhileI am comingup?',What shouldyou do?
Youmustnot bein a position to explainthe risksof proceduresthat you did not decide
on.Ifthe patientdevelopsa pneumothoraxandyou do not knowwhy theinternaljugular
approachisnot beingused,you cannotadequatelyinform thepatient.you arenot certain
eat-
ent.
loss
,ac-
fter
j
I(APIAN M'DICAL
16
|
chapterrhree
!(A9rAN
) MEDTCAL
why central accessis being obtained at all. You must, at the risk of seeming difficult, tell
the surgical residentthat he must obtain the consenthimsell If a complication occurs,you
cannot say,
"l
wasjust gettinga papersigned;I didn't know what it meant."
The sameis true for a patient who signsconsent.If you tell the patient that he could havea
pneumothorax and might need a chesttube and document this, and the patient still signs
consent, then you are not at risk. The patient also cannot say latel
"l
rvasjust signing a
piece of paper.I didn't know rvhat it meant."
TETEPHONECONSENTISVATID
Consent obtained by a family member, health-care proxy, or other valid surrogatedecision
maker is valid even if obtained over the phone. This is a legitimate form of consent by an
authorized surrogate decision maker.
Forexample, a 65 year-old r.nanis admitted to the hospital for a seizure.The head
CT shows a ring or contrast-enhancingi lesion consistent with a brain abscess.
The patient remains persistently confused, but is not deteriorating. You need to
perform a brain biopsy but there is no family member or health-care proxy who
comes to visit hini. His wife is housebound from multiple sclerosisand cannot
get to the hospital. You have hcr on the phone but the nurse is refusing to be the
witness for the consent, saying that telephone consent is not valid. What should
you do?
As with all forms of verbal communication, oral advancedirectives,and telePhoneconsent
are more difficult to prove if contested.However, they are equally valid. If a health-care
worker is uncomfortable taking the telephone consent,useanother member of the health-
care team to act asyour witness for the consent.You can educatethe nurse later.You can
take consent tbr cardiothoracic surgery over the phone if that is the only way to speakto
the surrogate.
The real questions about telephone consent are these:
1. Is the personyou arespeakingto reallythe surrogate?
2. Doesthe personknow the paticnt'swishes?
3. Did you get the oral/telephoneconsentwitnessedby another Personso that the person
giving consentcannot later deny havjnggivenconsent?
P
T]
wl
Sc
fr,
m
ol
H
c?
H
s€
Fr
o'
C
n
c(
ll
T
n
a
c
b
h
1
n
c
h
c
t
T
tnformedconsent
I
tz
It,tell
s,Iou
uYea
signs
ringa
PREGNANTWOMENCANREFUSETHERAPY
Theprevailing consensusholds that a fetus is not a'person'until birth. Hence, no matter
whatyour personal feeling may be, the fetus doesnot haveany intrinsic
'rights'
asa person.
So,even though a 34-week-old fetus would be a viable child if the fetus were removed
from the uterus, all health-care decision making and ethics are basedon the choicesof the
motherand her interests.Ifparents havea child born at 34 weeksof gestationalagein need
of a blood transfusion to saveits life, they cannot refuse lifesaving therapy for the child
evenifthey are Jehovah'sWitnesses.The statewould intervene in the interestsof the child.
However,if the same child at 34 weeks of gestational ageis still in the uterus, the mother
canreluse or accept whatever therapy she wishes without specific regard for the fetus.
Hence,a pregnant woman may refusea lifesaving transfusion. Shemay refusea Caesarian
sectionto remove the child even if this will put the life of the fetus at risk.
Foran1.question concerning reproductive rights, decisionsare basedentirely on maternal
wishes.The father has no legal right to make an informed consent fbr any pregnancy-
relatedissuebecausethe questions concern the body of the mother. A mother's autonomy
overher own body is felt to be more important than the rights of the fetus or of the father.
Onlythe mother can sign informed consent for any procedure or treatment during preg-
nancy.Any answerchoice that has
'Ask
the father. ..
"
in it will alwaysbe wrong in terms of
consentissuesduring pregnancy.
INFORMEDCONSENTFORA NEVER-COMPETENTPERSON
This is one of the most difficult subjects in ethics becausethe standard of this manage-
menthassignificantly evolvedover the last severalyears.Ifthe patient hasDown syndrome
andhasa family member to make decisions for her then the question will be straightfor-
ward-ask for the consent of the parent or guardian. If there is no parent or guardian, the
circumstanceis much more difficult. A third party court designeemust make a decision1,
basedon the bestinterestsof the patient even though the patient may neverhur,. ."p..rr.d p
her feelingsbefore.
The bestway to obtain consent for a person who has lost the capacity to make decisions
for him/herself is a health-care prorT or durable power of attorney. This is an advance
directive(written or formal). An advancedirective cannot even be given by a patient who
hasnever had capacity.The same is true of a living will. The next best method of giving
consentis
"substituted judgment." In this casea person who knows the patient well tries
to determine what decision shewould make for herself if shewere awake.This is also not
possiblefor a person who hasnever been competent. The weakestform of consentis to act
:nl0n
byan
d
s,
0
0
't
e
d
nsent
FCAIE
:alth-
ucan
akto
t(APt-Al{ M E D I C A L
18
|
chapterrhree
in the
"best
interests"of the patient. Tl.risis the weakestmethod of grvrng consent because
it is filled with subjectivity and inprccision. However, it is thc best method of obtainins
consent for doctors treating a person who has never had capacity.A legalguar.lian whici
could be a fan.ri11.member must make the decision on behaif of the patient. In the absence
of a family member the guardian is either appointed by the courts or is the administrator
of the health carefacility, such asthe medical director.
I rg
ause
ning
hich
ence
'ator
Chapter4: ConfidentialityandMedicalRecords
CONFIDENTIATITY
Physicianshave a strong professional mandate to maintain the confidentiality of patients.
Communications betseen paticnt and physician are highly privileged and this confiden
tialit,vcar.ronly be violated when there is potential harm to a third party or if thcre is a
court order demanding the information. Medical information cannot be passedto anyone
l ithout thc direct consent of thc patient. Confidentiality also includes keeping a patient's
medicalinformation priyate even from his friends and family unless the patient expressly
saISit is okay to releasethe information. The fact that a patient may have a good rclation
shipwith his familv and fiiends is absolutely no excuseto assumethat the patient wants his
medicalinformation passedon to them. I have an exccllent relationship with my mother;
hon'ever,even though I am a doctor (or mavbe becauseofit) shedoesnot want me to know
herlist of medications. Shehasno obligation to give me a reasonrvhyshedoesnot want rne
to know which mcdications she is taklng. If I call her doctor and sa1
"l just want to help
mom vvith hcr mecls.What is she on?" Her physician is supposedto respond,
"I'm
sorry,
but IoLrr mother hasn't authorized me to give you that information. I know you mean well,
but I iust can't talk to you about your mother's medical problen.rs."
For example, a 42-year-old man is hospitalized with chest pain. The patient is
arake and alert. His wife comesto you demanding inforn.ration about the patient,
saying that she is his wife. She shorvsher identification card verilying this. What
shouldyou tcll her?
bu cannot releasemedical information to anyone about a patient unlessthe patient gives
you pernission to do so.Although it may seem rude and unreasonable,you must tell the
MEDICAL
20
]
ChapterFour
patient's fan.rilymembers that you must ask your patient for permission befbre you
releasehis medicalinfornation.
For example, the wife becontesinfuriatcd and storms off the floor, threatening to
sueyou. You apologize to the patient for upsetting his wife by not speaking.with
her about his private medical problens. The patient responds
"On
the contrary,
Doctor, you did great.Although she is still my wife, we arc finalizing our divorce
and we do not live together. I expect to be clivorced and rcmarriecl lvithin the
next few months. Sheonly wanted information about me to useagainstme in thc
divorce proceeding. Thanks for protecting my confidentiality."
REI.EASEOFINFORMATION
Information transfer between physicians involved in the care of patietts is a common
occurrence. However, the information can only be transferred if the patient has signed a
consent or releaseform requesting the transfer of information. It is the patient who must
sign the consent to releasethe infornation, not the health,care provider. This is how the
system guarantees that the patient's medically privileged inforrnation only transfers to
those people to whom the patient rvants it to go.
For example, yott receivea phone call from another physician who is rvell known
to you in your local community. The physician saysthat one of your former
patients has transferred his careto him and he is asking for a copy of the patient'$
meclicalrecord. What do you tcll him?
You should tell another physician requesting information to send you the patient's signed
releaseform befbre you send him the infonration.
!!l!!p r'reorcnr
00n
: d a
rust
the
i t o
GIVEMEDICALINFORMATIONTOTHE
PATIENTFIRST,NOTTHEFAMITY
For example, your patient is awaiting the resr.rltsof a biopsy to tell whether or not
shehascancer.Her soncallsyou anclasksyou to givehim the infornation bccausc
the family is concerned that the bad news will depresshis mothcr. He is sincerc
and genuine in his concern. r'{hat do you tell hin?
trledicalinformation such asthe result of a biopsy must go to the patient first. There is
no basisfor informing the family and not the patient.It is exactlythe oppositc:without
directinstruction from the patient, the family shoulcinot receivethe patient's confidential
nedicalinformation. Maybethe patient wantsher family to know and maybeshedoesn't.
It is alwaysthe patient's decision. There is a rare exception in the caseof a patient with a
psychiatricdisturbancein rchom to inform if a meclicalcondition might induce a suicide
attel1lpt.
RETEASEOFINFORMATIONTOGOVERNMENTAL
ORGANIZATIONSANDTHECOURTS
For example, an investigator from a local law enforcement a€lencycomes 1(]:'
your office. He shows )'ou proper identification stating that he is a government
employee.He is looking for your patient's immigration statusand for his medical
condition. What do you tell the investigator?
If a n.remberof a larv enforcel.rent agencycomes to you with a subpoena or a court order
that constitutes a searchwarrant then you must lurnish him with the information that he
requests.Ifthe investigator doesnot havea searchwarrant, then you must refusehim access
to the files.You are not under any obJigation to make immigration statusinvestigations of
vour patients nor to provide this information to third parties unless it is at the request of
thepatient. This right of privacy also coversgeneticinformation. You must keepthe medi
calinformation private from a patient's co-workers aswell.
confidentialityandMedicalRecords
I
21
I(APLA''I MEDICAL
I
ChapterFour
BREAKINGCONFIDENTIAI.ITYTOPREVENTHARMTOOTHERS
The^right of a patient to privacy is not absolute. .fhere
are sonle exceptronsas to whenconfidentialitycanbe brokcn in order to protectothc.s.The Tarasofcasc(1976),in whicha mentally ill paticnt toid thc psychiatristof his intent to harn sonreoDe,is a famousexample of this ln this type of case,thc physician must inform larvenlorcement asrvell asthe potential victim. Confidentialitl. is only broken ir-r,h;, ,;;;; pr.l=nr harrn to orhers;this is rarely donc.
Other casesin which it is lawfrl. to break confidentiality inclucle partner notification forsexuallytransmitteddiseasessuchas syphilisand HtV fne patienfs right to confidential_ity in such cascsis lessirr.rportantthan anorher p.rr.r,, ,ish, ;; r;;.ty. Horvever,all effortsmust first bc made to enlist the patient to inform tr.reintiiriate partrer. No lawsuit agai'sta physicia' for brenkingc.nfidcntiality in order to nolify an ir_o..nt,f,la purty that hishealth may be at risk hasbeen successful.
MEDICATRECORDS
The physician or healtrr-carefacility physicalry owns tr.remedical record, but the informa_holr contained rvithin it is the property of the patient. Although the medical record as ap,hysicaloblect rcnrains al.waysin the hands of ih" h"ultl,_.ar".fir.lfit1,, tt . parient has anabsoluteright to free accessto the information it contar.ns.it "-rrr.o.n,onon containedu'ithin a patientt nedicar rccord is cnveredby a' the same rures of confidentiarity asanyother privilcged meticar information. you cannot relcascthe mecricalrecrrd without theconsent of the patient No one exceptthosc directry involved in th. .ur" ot the patiert hasa right to accessto the record. paticnts cirnuot tJkc sulc *rr""rr.,r, ., ,n" O^ysicalnteclicalrecord but they havea rigltt to acccssor copy thc information.
For example, you have a new patient with a complex history r,ho hasbecn tryingto get a copy of her record from her previous doctor. .fhe
other practice said shemust provide them rvith a valicl reason for u.hy she needs thc chart. lbu call theother doctor's office trying to get the chart. The practice administrator inforns
you that thc paticnt is extremely unpreasantand aim.utt. rr.,uiJrt,un, becausethepatient hasnot paid hcr bill thc prior practicefcelsno obligationto provide youwith the chart. The patient rcturns to seeyou thc tbllowing iav anclasksr.vhathasbecome of her record. What tlo ),ou tell her?
I(AP!AN MFDICAL
Con{identialityandMedicalRecords
I
vhen
hich
lous
:llas
for
tial
brts
inst
his
na-
N A
alt
red
Thepatienthas a right to her medical records.No one has a right to interfere with this for
anyreason.You should tell her that she should be allowed a copy of the chart. A patient
doesnot have to give her doctor a reason for requesting hcr own property, and she is
entitledto this information whether or not she is
"pleasant."
Furthermore, the mcdical
recordshouldnot be
"held
hostage"to compela patientto payher medicalbills.The need
for information to take care of patients outweighs the physician'sright to payment.
CORRECTINGMEDICATRECORDERRORS
hen an error in a chart needs correcting the doctor should draw a line through it and
theninitial the correction. This allows anyone reading the chart to seewhat was originally
thereand it ensuresthat medical errors are not being covereclup. you cannot just remove
pagesfrom the chart or cover them over with correction fluid if there are mistakes.This
makesit look asif you arehiding medical errors. If you forget to put in a note or document
somcthingand want to add it the next day,you cannot put a note in the chart with the old
date.If you forgot to put a note in the chart documenting a patient's condition yesterday,
,youcannot write a note today with yesterday'sdate on it. In other words, you canl]ot,back-
date'notes.Your notes must alwaysbear the current date and time.
rny
the
!as
cal
I(AP:-AN MEDICAL
WITHHOTDINGANDWITHDRAWATOFMEDICATTREATMENT
Everycompetent adult l.ith the capacity to understand his own medical problems hasthe
rightto determine what treatments he does or doesnot wish to receive.There is no ethical
or legaldistinction between withholding and withdrawal of medical treatrnent.
For example, a 60-year-old man with diabetes and hl.pertension develops renal
insufficiency to the point of needing dialysis.He is equivocal about spending the
restof his life on dialysis,but he agreesto start. The patient is not depressedand is
fully alert. Six months after starting dialysis,he comesto realizevery clearly that he
absolutelydoes not wish to continue. You have no doubt that the patient has full
capacityto understand the implications of this decision. What should _voudo?
Although there may be an enotional distinction between withholding dialysis and stop-
pingit after it has started,there is no ethical distinction between the two. lf I don,t like to
playbasketballthcre is no legal distinction between my never starting to play basketball
or playing a few gamesand then not doing it anymore. It is n.ryright to stop. If I hlre you
to repair my house, but after a few days I deciclethat I don't like the work you are doing,
I havethe right to tell you to stop working on my house.You cannot say,
"Sorry,
once we
startajob we finish it whether the owner likes it or not." I havethe right to refise to allorv
youto work on my house and the right to tell you to stop after you started.The patient has
theright to stop treatment.
For example, an elderly man with COPD progressesto the point of needing
mechanical ventilation on a chronic basis.He tells you, alier long consideration,
that he just does not want to live on a ventilator. What should you tell hirn?
Chapter5: End-of-Lifelssues
M F D I C A L
26
Youmusthorrorhiswi.he.I hj r).rti.nri. a- ,,r,.r, .:,L-l
mcdicalproblems,sohehas,n.:,:l:t::t l't
an adultrvith the capacityto understan<lhis
t'ator.r.he*oon*.un,*.,,,n.il:riliil:,,.'i:.iT::::;:"l,",,_TJ
asking the family for consent.
For example, a 42_year_old
reavinshimp-"ry,;;;;;,il1;,i'J.,i,"iTlll[Tl.#il.1],:,i.":*;upbcatand cheerful.He sayshc will gerbetter"r;;;;;; to-tb.n.,orntorrr"dp".manentlyon the ventilator.yo' clearlyinform him .h;; ;; ;r'*r."g and irewillneverinrpror.c.He sayshewantsthe ventirarorrn."*r' n..rr.,,irnerscured.whatdo youtcllhim?
()nceagain,you must honor his
understandhis meclical.on.litio,l
wishes This is an adult patient with the capacityto
therikerhoodorhi;;:;;;;:i":"T-,HX; :.:iil::JillHi,:1:J::*ifjequivalent of incompetencc. Overall, this caservill be ,h" ";;i.:;,.,, agree wirh, becausemost ethicaldilemmas do not inr
rather in'olve patients ,.,ho have ,;l-tt
u t:nttt:t'. patient wishing to continue care,but
drirw treatment.
rst capacrtyand the dilemma of whether or not to with_
For example, a woman with
Arterarbw_"",r,,,r,"r,..",,1:'iil::?"ill :i."?:::;i:,Til:.::ilj:r::sions.Shehasthecapacityto understandthatshewill ai.,.rriifrcr*
,fr" ar"nsfusion,althoughshcisnot suicidal.Vhatdoyou tellher?
Youmusthonorherwishes.She
problcms.rhe consequences.r r]l,lt]-^Yl th,the
capacityto unclcrstandircrmedical
;::.,'i'"':.:;*'j.,]:H.{*iirFFHil"lr';i::il::il;an cthicsconrnrilteeevaluation.;::l::-i:t llt
to ruthorize the proccdurc'or to askfoi
capacirytounderstancr,,";:;,;,:l:i;'l';:';:T]11,'::.J,"il:J:::nT.':ji:.i:Tpsyihi.rtricevaluarioni nccc.rr).
For example, an HlV-positive,lehovah,s Witness who is now pregnant needs atransfusion to live and haye a
deprcssedandisfuuyalert.{hf::t;:1:i, ;|,"..atcsoricauv
rcfuses.sheis 'ot
1
t
c
c
T
tl
r(
I
I
t
tl
P
u
cr
D
h
cl
tl
rl
ft:
A
at
lc
bi
!!!!p Meorcar_
.dhis
:y to
bout
t the
ause
but
dth-
lbu must honor her wishes.Yon cannot transfuse!t competeDtadult againsther tvill. The
situatior.rwould be different if thc patient rvcre a n-rinor.in u,hich cascthe doctor rvould
becompclledto transfuse.The fact that the patient is pregnantdoesno1alter the ansrver.
Theprevailingconsensusis that personhoodbeginsaftcr birth. Until delivered,the fetus
isconsidcredanother part of thc mother's boclv.
'I'he
rvrong ansrverswill includc getting a
courtordcr or askingthe father of the child for consent.Another wrong ansrverwould be
uaitinguntil the paticnt is no longer consciousand then transfusingher.
Theanslversto all of the examplesdescribedin this scctionare clearbecausein eachcase
thepatientis an adult rvith the capacitl,to understandhis or her nredicalproblems.If the
casedescritresdeprcssionin the patientthenyou shouldchoosepsychiatricconsultation,or
choosea trial of cither belravjoral therapv or antidepressantntedication asthe ansrvcr.
Patientshavethc right to try therapyfor a while ald thcn stop it if it doesnot suit then].
This is true even if it means they will clie from stopping dialysis, mechanical vcntila
tion, HIV medications,or blood transfusions.The typc of treatnlent does not change
the answer.A CBC or cardiac bypass is ethically antl lcgally indistinguishable. Ticatin€!a
palient wjthout consent is legally ecluivalent to assaultand battery or any other form of
unwantedtouching.Therefore,in a scnsc,treatinga paticr.rtagainsthis wi)l and without his
consentis like mugging the patient or beatinghirn up.
Thereis no distinction betweenwithholding anclwithdrarving care.If you are doing sonc
thing the patient does not ''ant,you cannot sa,v,
"We11,
sorry, but I already started, and I
rcallvhaveto continuc."
ADVANCEDIRECTIVES
Definition
An advancerlircctive is the tncthod by rvhich a Daticnt conmunicates his wishes for his
healthcarein advanceof becoming unable to make dccisionsfor hirrself. The aclvance
directiveis a by procluct of the successof nredical thcrapies such as the mechanical vcn
tilator that can kccp a patient alive when in the past he would have cliecl.Becauseof thcse
therapies,doctorsarenorv in the position of trying 1qictcmine u,hateachpatientwantcd
for himself in terms of his health carc.Thc advancedirectivcis part of the conceptof
autonomy.The advancedirectiyetellsthc physicianrvhatthe paticnl'swishesaresothat the
lcssaccurateforms of dccisionmcLking,suchassubstitutedjudgn.rcntor making a decisiou
basedon irnotherperson'sideaof the bcstinterestsof the paticnt,bccomeavoidable.
End-of-Lifelssues
J
27
ical
l b e
Ling
tor
ntt
no
!!l!!p vrorcnr
F
Health-CareProxv
The ttvo most comnon forms of aclvancedirectives are the living will and the health carc
pror:7 Thc health-careprorrr or
'ir.redical
power of attorney" is thc durabre power of attor
neyfor health-caredecisionsor medicalproxy.The conceptof a,durablc,,powerofattorDey
is critical becausethe word
"durabre"
means it remains in effect evenafter the patient roses
decision-making capacity becauseof medical illness.Other forms of legal proxics, such as
a finar.rcialproxy, bccome ineffective after the patient losesco'rsciousness.The health carc
prorT is a person chosenspecificallyby the patient to 'lake health-caredecisionsfor her in
the evcnt that she cannot make decisions for hcrseH T'hesedecisions are limited to health
care,not finance.Advance directive docunents ntay also have written instructions to give
boundariesto care.For cxample,.rpatient rnay want to receivcantibiotics,but not want
to receivechenotherapy or diall.sis.However, the main focus of the proxy is to designate
a person of
"agent"
who speaksto the physician regarding consent issuesfor aI treatments
and tests,aswell asdiscussesissuesof withdrawing and withholding treatment.The proT
speaksfor the patient. Becausethe patient chooses the prorT as her representative,the
proxy overrules all other decision ntakers. Therc is a strong presumption that the proxy
knows the patient's wishes.The proxy is not there to give his personal opinion as to what
he thinks should be done for tl.repatient. The proxy is there to communicate the patient,s
originalwishcsin order to ensurethat they arecarriedout.
The pror:y is like a messenger.'I'he patient writcs thc message-hcr wisrresfor her own
health care-and the pror:1,delivers the nessage.you woulcl not want your prorT to alter
your wishes any more than you rvould want your mailntan to rcwritc your letters.
The proxy is also like a waiter. The patient tells the waiter lvhat kind of food he wants to
eat (what kind of medicines and testshe rvants).The proxT praccsthc order ir.rthe kitchen.
The prory is not there to alter ).our expressedrvishcs.you would not want to order chicken
and havethe war'tertell the kitchen you want fish. your waiter tries to understand what you
want to eat.
'l'he
waiter doesn,twalk up a'cl tell you,,,you look weak and anemic. you are
having a rare steaktoniglrt, which is what is bcst for you.,'Now the main diffbrence is that
this is a
"restaurant"
in which thc custon.rcris unconscious and can,t tell you exactly what
he wants.
The proxy makcs dccisionsbasedon rwo paranteters:
l. 1he patient'.sdirectly expressedhealth carewishes
2. 4rat thc patient woulti havewanteclif he/shchad capacitv
I crapterrive
M F D I C A LI(A PI"AN
h-care
'attor-
torney
t loses
uch as
h-care
her in
health
o give
- want
ignate
ments
proxy
e, the
ProxF
what
:ient's
lIs to
;hen.
cken
.you
I are
that
ryhat
Thesearethetrvo overriding principles:
t. Thehealthcarepror.y must carry out the written and verbal wishesexpressedto him
regardingthepatient'shealth care;the basisforwhich is sornetimesjusthis understanding
0fwhatthepatientwould rvantif shewerea'vaketo makethe decision.
2. Theprox,voutweighsall othcr potcntjal dccisionmakcrs,including thc family.
For example, a 75-year-old nar arrives at the emergency department febrile,
shortof breath, and confused. Many family men.rbersaccornpany the patient,
includinghis wife, his siblings, his children, and his grandchildren. The physician
'lvantsto perform an emergency lumbar puncture, which the patient's wife and
siblingsare refusing. His 25-year-old granddaughter r,valksup r'vith a health-care
prorylbrm signedby the patient designating her asthe proq'. Sheinsiststhat you
dotheiumtrar puncture stating that washer understanding ofthe patient's wishes.
Therestof the family, including the wife, refusesthe lumbar puncture stating that
theyknorv the patient's wishesbetter.What do 1'oudo?
[egai',youshould honor the health-care prory aboveall other decision makers,regardless
ofthelevelof closenessin biological relationship or frequency of contact. You cannot tell
ivhoin a family knows the paticnts wishesbest unlessthe patient is awaketo tell 1'ou.The
prox,vdesignationis thc patient's wa). of telling ).ou who he feelswill representhis rvishes.
In the absenceof an advancc directive there is a list of relative importance in terms of
surrogatedecisionnakcrs. You should start first with the spouse,then parents,then adult
children,then siblings,tl.renfriends. This is an approximation only. lf the farnily is split in
itsrvishesthcreis no easysolution. When the family is split and there is no proxy, you nrust
reterto lhcclhic.eommittecor thccorrrt for a iudsmerrl.
LivingWill
A living will is a written form of advance directive that outlines the care that a patient
l,ould want for herself if she were to lose the ability to communicate or the capacity to
undcrstandher medicalproblen.rs.The etiologyof the lossof decision-makingcapacityis
irrelevant.
A living will can range from being an extremely precise document outlining the exact
typesof care that a patient wants or does not want all the way to being a vague, useless
documentthat makesnonspecificstatementssuchas
"no
heroic care."The nain problem
End-of-Lifelssues
|
2s
MEDICAL
with the living will is trratmost of the tinrc it lacksprecisionbccausethe patientdoesnot
explicitly statewhich testsaud treatmcntsshewants for herself,A clocumentsaying..no
extraortlinary cilre" is virtua[y rvortrrless.wrr.t croes
"extraordinary
carc" nrean?I)oes that
mean a yentilatoror chemothcrapy,or dialysis,or bloocitcsts,or all of thcnr, or none of
thenr?If the living '' l is cxpricitin listing thc prccisenanes of the tcstsand lreatments
that thc patient *'ould rike to receive(or not to receivc),then it is uselul.For instance,a
living will that says"No
intubatior, no cardiopulmonaryrcsuscitatron,no dialysis,and no
blood transfusions" is vcry Lrsefuland allotvs fbr easyfollowinq.
For example, a 78-year-old woman ts admitted with metastatic cancer Jeaclingto
a changein mcntal status secondaryto hypercalcenria.Shehas a lir.ing will in her
recordthat states,
"In
the cventthat I becomeunableto speakfor mvselffor any
reasonI rvish to expressmy r,r,ishthat I not be intubated or placed on a ventilator
under any circumst.ruces.I alsodo not wish to reccivedialysis.Blood tcstingand
antibioticsareacceptable.,,What shouldyou do?
Thc Iiving will is nost valid antr usabrc*.hcn specific tcstsand treatmerts are outrined. In
the cascabove' follon' the directio' of the living will ancrcarry out the patient's wishes.A
livi'g rvill woulcl overrurethe wishesof the fanrily beca,sctrrc riving will conmunicates
the patieDt'so.wn wishes.As a nlatter of autonon),, the patient! clearly expressedrvishes
alrvavstakeprecedenceoverthc n ishesof other decisionmakers,suchastamily mentbers.
Thc rnajorissue'ith the useof a living will is that it is very difficult, ur advanceof trre'r-
ness,to be celtain which medical treatmer.ttsand tcstswill be necessary.It is very difficuit
for a layperson to sa1',
"I
do not tvant an albumin infusion with ny large volume paracen_
tesis,"or
'A
biopsytbr diagnosticpLrrposesby interventior.ralradi.logy is acceptablc,but I
do not want an opcn biopsvil the operatir.rgroorn,,,or.,l agreeto antibiotics,but not to
amphotcricin." A hcalth-care proxy allows for far greatcr tt.*it itity. Hor.vevcl if a patient
really does vvritcout the specific nantes of the nlost colftmon treatments and the param_
etersfor their use,ther.rthe living rvill can bc a vcry usefui documelr.
NoCapacityandNoAdvanceDirectives
Here is u'hat is vcry ciear atrout withholding and withclrarval of caredecisions:
' An adult with capacitycandecideto acceptor refuseanv therapyoflered.
. An adult I'ithout capacitycanbe managedr,vitha health_careproxy or a living will if
the Iiving l'ill is sufficientlyclearand specificenough.
t
t
p
ft
p
n
b
cl
(ll!!p v:orcer
End-of-LifeIssues
I
5 l
ot
-lo
at
of
tS
a
IO
Llnfortunately,the 'ast majority of patients, e'en at oldcr age a'd with life-threatening
illnesses,do not havea formal advancedirective. Decision making ca' be rmLchmo.. diffil
cultin this circumstance.Ifthe family is united and in agreement,then there is 'o difficulty
llith making decisions for the patient. The main issueagain comes to clemonstratjng tlte
bestevidenccof lctowing the patier.rt'swishes.
Forexample, a 64 year-old man suff-ersa severeiltracrallial bleed leavir]s him
conatoseand paralyzed.His wife, sister,and four chjldren arc in the horpital.
Theycome to seeyou bccausethey are unanimously asking that you remove the
endotrachealtube and lcavethe patient to die. The patient repcatedlymadc this
rvishknown to his family. What shoulclyou do?
Forapatient without the capacitr to understand his 'redical problens and no hcarth care
pror-vor livirg will, the path of rnanagement is clear asrong asa[ of the familv menrbers
ar€in agreenlent.You can rcmovc the cnclotrachealtubc and let thc patrcnt dic if everv_
.ne savsthat is what the patient has saitrhe wanted.'rhe endotrach.artube is r medi.al
therapylikc any other. The patient has the right to refuseit. The patientt fanrily or otrrers
whoknow the patient well can provicle the patient's previously expressedwishes_an oral
advanceddirective.
For example, a 78-year old wouran has been aclmitted to a nursing homc with
adyanceddementia. Shehasdifficulty maintaining oral intake sufficient to survive.
Thenursinghome wantsto placea nasogastrictutrefor-fecding.The husbandand
the son expresslystatethat the patient slid she.,never*lnted to be ntaintained
like a vegetable"ar.rd
"I
don't want to be put on a ventilator or have a feedins tube
dorvn my throat." What should you do?
Arethe family members passinplon information about what the patient said, or are thev
tellingyou to do what the1,think is bcst fbr the patient? you can do virtually anything in
thecareof a patient, even without an aclvancedirecti'e, aslong as the family is united in
passingon what the patient said she wanted for her own care. problems arise when the
familyis in disagreementabout what the patient said or rvhcn.rnsteadof expressingthe
patient'swishes, they are representing what they u.ant for the patient. Having a fan]ily
memberexpressa patient's wishes is equivalent to having him cast a vote on the patient,s
behall If a person cannot speak,another person can mail in her vote for her if she has
clearlytold this person which candidate shervants to vote for. If she told her brother that
lvantedto vote republican and he sendsin her ballot stating this, then he is acting on her
r wishesasheragent.on the otherhand,if heberievesshewould preferto votedemocratifshewereawakeor hebelicvesonecandidateis betterfor her thentl.risrepresentsa muchlowerlevelof evidencefor hervote.Likewise,if hek'ows what shewantsbecauseshetorrlhim, then he canstopthe ventilator,the blood tcsting,the .h.tno,t "rupy,the dialysis,oranyothertreatment.
EthicsCommitteeandReferraltotheCourts
ihen there is no clear adyancedirective and the family does not agreeon what the patient
wanted for hin.rserfor herself,then the right answer is to refer to the etnrcscommrttee andfinally the courts.
Hcre is a list of thc various kinds of consent,with the most desirablelisted first:
. Direct patient rvishverbalizedby the patient
. Forrnaladvancedircctivesuchasa proxy or a living will
. Oral advancedirective
. United group of fimily and friends
. Croup of family membersdisagreeingon what thc patient would havewanted
In caseswhere there is no living will or proxy and the family members are not in agreement,
:h. :*:.:,:p
is to
"encourage
consensus,",,request
discussion,,,or
..talk
with the parties
iltvolved" Ifconsensus is not possible,you should refer the caseto the hospital ethicscom_mittee. The last step to pursue in the absenceof a clear consensusis relerral to the courts
or
"seekjudicial intervention.,,
For example,the Terry Schiavocaseended up in the courts becauseher husband and sev_eral of her friends said the patient told them that shc ncver wantecrto be maintained in apersrstentvegetativestate.The patientt parents stated that she never saiclthat. The ethics
::-.TiU::
does not have direct legalpower and their decisions are not alwaFsuniversally
binding' Ifthe ethicscommittee is unabrcto bu'cr consensus,or the involved parties ignorethe ethics committee, thcn judicial intervention i, n.."rrn.y.

r
a
t
rl
d
li
I(APLAN MEDICAL
End-of-Lifetssues
I
ocrat if
r much
he told
ysis,or
atient
'eand
"00
NoTRESUSCTTATE-(DNR)ORDERS
A
"Do
Not Resuscitate,,(DNR) or<
;::t*:,f*.:f..H.il?#J+,':i;:*,,jffii,H;[i.*::i::::*::J:;thepatienrttr il;;;;;r/Lxrnrc
medications'That is all' nothingmore.Deathof
breath.
ruddenlossof pulse,bloodpressure,and the ability to
Forex&mple,a 42_year_oldHlV_positivernan is admittedfor hematuriathat is
ffii:i"ilfiT':11]:t'J;;i
HeisDNR'u"r"gvi''..""*r,edandthevthink
notwanttodoeith".on.b..or:^"11"1:'-1ry"lllt-Tfhtectomv'however'thevdo
whatshouJdyou tell them?
sethe"patient
isDNR a'd thereforepreterminal."
Forexample,a 42_year_oldman with AII)S is bcingtakento the operatingroom
j:1:::lfl*.-, r]resurgicarresidenrshrussh;,";;i;;;;.,ays,.oh wel,
:i:::::'ff::iffifttli l*l)- tls itcr"oan'trea'vmitie.irth.s,',.g.,vi,of a relicfl"Whatshouldyou tell hrm?
DNRdoesnot mean patientssh(
resuscitation.DNRt,.;;;";;;;';'#,;',,.._";::f
ruli?fJ:J::illJ#:$T:ffiIdorngor servesasan excusefor making" _irtut". oNnlo;;;;.", it is okayjust toletthepatientdie,or thar thc doctor doesnot have,. b. ;;;;;;i,^ patienrcanstilt be
'ties
rm-
lrts
Themostcommonmisunderstandingabout DNR ordersis that beingDNR must mean
jT.ffH'"t$lfr",tiJ,:,;:i*",, l",ti..A"'rh.;.;;;;;Jisuncrerstandinsisthat
bropsies,or dialysis.o*o * ,r, "'..9i1
other aspectsof routine good caresuch"r r;;;:
nomaandneedsab,.0,,"J;;;il:IiJ:;'"Tl]:lh,TiillT',il:[]::::i::fl.;
$:lJ:;l;::lT*ffffi:i::::
*o;-p,t.;;,;;d;',ilo**o**;u.,-.u.,,
whenapatientdiestt ".. i, o,uaa].ilp"1l^Tj:.1?,iT':t
": th''nd-pointoftherapv.usually
ingintravenous;.;t;;;;;r,;;tn'
raptoupgradein theamountof thetnpv,,u.h u, giu'-
maystiluncrergo,",r;,;;;;;"::;1,;r,T:.,":il.1L..:i"j:T:fi:1,Ti,i:n.Apatint
trhenpatientshavecardiopuln.ronaryarrest,thereis presumedconna'resuscitationunlessthe patient specificalryand expressivety;::i.J,X.*:tJi:;
1 a
lcs
Ilv
IE
I intubated and maintained on a ventilator if sheis DNR. This is a confusing point because
endotrachealintubation is often a part of the normal resuscitativeprocess.If the pl rientir
DNR and he loseshis pulse then the doctor would not intubate becausethe intubation here
would be consicleredpart of the'Code' or resuscitative/CPRrlanagement. Hon'ever,ifthe
patient remains alive and has advancing lung disease,the patient can still be intubated ln
. this case,the doctor rvould only defer CPR if the patient were DNR. Doctors Jo not have
'
to rernovethe endotrachealtube of all patients who chooseto be DNR. Many patients will
not allow themselvesto be made DNR becausethey believethe medical staff will not beas
aggressivein their other treatments.
For example,a 68 year-oldman with basalcellcanceris aclmittedfor evaluation
andtreatmentof a fever.He hasbecnDNR for thelastsixmonths.After the chest
X ray,urinal)'sis,andblood culturearedonehestill hasafeverof unknownetiol
ogy.When you askthe residentwhy he hasn'tdone more tests,the responseis,
"Well,
wervill getto it, but thereisn'tmuchof arush,alierall,thcpatientisDNR."
What do you tell hin.r?
DNR doesnot meanarrythingmorethan deferringCPR,suchaschestconpressionsand
electricalcardioversion.In thiscase,it wouldalsoeliminateendotrachealintubationif this
treatmentwerepart of the resuscitativceffort.That is all.Thedoctoris expectedto man-
agepain andthe diaplnosisof othermedicalproblemsjust asaggressivelyasshewouldin
a non-DNR patientunlessthepatienthasspecificallychosento deferthoseotherformsof
therapy.Doctorsdo not automaticallyhaveto havethc DNR orderrcversedjust to bein
the intensivecareunit or to goto surgery.
A
T]
th
o1
rt
gi
n
h
I ( A ? L A N M E D I C A L
-
End-of-Liferssues
I
ie
is
'e
€
n
e
lt
s
FTUIDSANDNUTRITIONISSUES
AdultswiththeCapacitytoUnderstand
Theartificial administration of fluids a'<1nutrition is a medical procedure and treatmentthatcan be acccpted or refused t
othertreatn.rent.,Artificial
odr',ir"
n competent adult in exactrythe same manner as any
rhaneatins..Artinciar,,,o;;*,:::'l::r.ffii:xffi:;T,f,::.Tiil :i::il:l:Sastric'or jejunostomy turre pracement. 'Artificial
nutrition" would also refer to intrave-']onslyadministered nutrition such astotar parentcrar nutrition (TpN), also referred to ashyperalin.rentation.
For exqmple, a 47-year_oldtheoretical physicistwith amyotrophic lateral sclerosis
nasDecomcprogressivelynore clisabledto thc point of beinj virtually immobiie
in a rvhcelchairHe is unablc
gastricreeding,ut.u,,a,.ru,'.o,lo:illT'::,:',':?tijJil,,";#;'":JJ:l;:j,*:
nurse,isinsistingthat you reinsertit. What shouldyou tell her?
Forcibleinscrtionby anyoneof an artificial feedingdeviceinto an adurtpatientwith thecapacityto understandthe meaningof its rcmovalis not allowed.If the patient,swishesareclearlyexprcsscd,thisisthesamcasrefusingaventilator,bloodtesting,or dialysis.Thisrefusalis subjectto the samecriteriaasthe refusaiof oth", tl.,.rupiar.you must be surethereisn'ta severedepressi.nunderryingtherefur", "ra ,n"t,i. riiuent undcrstandsthathema,vdie without the tube.If thereis no depression,tn"n tn. fnt,.r.,t hasthc right torefttseatherapye'en ifhe will diewithout it. putting thc tubetru.t in,o apersonwhohasrcfusedit isconsideredthesan.reasassaultandbattery,.
AdultsWhoHaveLosttheCapacityto Understand
Decidingwhat to do aboutartificialnutrition 'vherthe pati.rt cannotspeakfor herserfisamuchhardercircurnstance.Nutrition is the singlemostdifficult issuern termsof witb_holdingandwithdrawingtreatment.We canneverwithhold ordinarynutrition like foodtoeatandwaterto drink. The stanclardof certaintyregardinga patientt wishesin termsof.artificialnutrition is much higherthan wirh other ,i.ropri."- il"'rnort havevery clearefidenceof whatthepatient,swisheswerein regardto artificialnutrition.
I
I(AP'AN
-
M E D I C A L
If there is a health-care proxy, and thc healthcare agent says,
"The
patient clearly told me
that they didn't want tube feeds,"you nay withhold the therapy. If there is a living will
rvhere the patient themselveswrote,
"I
do not tvish to have a nasogastric tube or other
forms of artificial nutrition," you may withhold the therapy.lf there is no advancedirective,
but the family is in uniforrn agreement that the patient had expressedthe wish never to
have tube fecds,you may withhold the therapy.If clear wishesregarding fluids and nutri-
tion were ncver clearly expressed,thcn there is inplied consent for the feeding basedon
the presumption that it would be the patient's wish to be fed and on the f-actthat feeding
and hydration are in thc patient's best interest.
Emotionally, the standard is different from the standard used to dcternrine whether or not
ordinary testing such as blood tests or CT scansshould be donc. The proxy and fanily
can say,
"The
patient never specifically told rne that he would not want a liver biopsy or
MRI of the brain, but mv understanding of his wishcs in generalis that he doesn't want to
undergo theseprocedurcs."This would not be the same for nutrition. Decisions on with-
holding and withdrawing of artificial fluids and nutrition should bc treated the sameway
asany other medical treatnent. There is a higher standard of evidencefor decision making
in some states.That is why this issueis so complex. Fcw people have left a specific,written
document concerningtheir desirefor tube feecling.
The evidence has to be clear that a paticnt does not want artificial nutritjon. Thc routine
assumption is that rnost people wish to be fed asa part of ordinary care.If the evidenceis
not clear,then a referral to an ethics comnittee, or possibly to the courts, is necessary.The
level of evidence regarding the patient's wishes necessaryfor withholding and withdraw-
ing artificial nutrition hasbeen treated by some courts asthe sameasthe level of evidence
required in a criminal case.The system errs on the sidc of caution-perhaps letting a few
guilty people go free rather than sending a single innocent person to jail or execution. If
you lost consciousnesssuddenly and left no specific instructions, wouldn't you want there
to be a need fbr colrvincing evidence that you didn't want to be fed before your relatives
rvere able to withhold nutrition and possibly let you die? Ethical consensusholds that
decisiolrs to provide or forgo artificial nutrition and hydration (ANH) for patients who
lack capacity should be made according to thc same standardsasthose uscd for any other
medical treatment. Despite this consensus,ccrtain statesimpose more stringent standards
for withholding or withdrawal of ANH compared to other medical trearmenrs.
P
In
hir
lif,
clr
TT
nc
E
A S
ac
ni
th
et
Pa
se
th
pr
to
as
m
m
th
AI
l s
$ l
BI
T
TI
(1:.^9 MEDTcAL
End-of-Lifelssues
I
me
vill
hcr
ive,
to
.ri-
on
rot
ilv
O I
to
L
PHYSICIAN-ASSISTEDSUICIDE
hrphysicianassistedsuicide, the physician provides the patient rvith the means of ending
hisownlife.The doctor doesnot actuallyaclministerthe substancethat endsthe patient's
life.Although there is national controversy on the issue,the answer on the USMLE is ver1.
clear;physician assistedsuicide is alwaysconsidered incorrect and ethically unacceptablc.
Thisis true even if there is a local state law permitting the procedure. W1rat is legal does
n0tautomaticallyequal rvhat is ethical. Generally,physician assistedsuicide is requestedby
patientswho havea terminal diseaseand a limited life expectancyanyway.Neverthelcss,the
severityof the diseaseand eventhe discomfortand sufferingof the patient do not change
theansu,er.Physician assistedsuicide is alwaysconsideredto bc wrong on the USMLE. The
primaryissueis one of intent. Physician assistedsuicide is inimical, or absolutelycontrary,
to the role of the physician to savelife. This is true even if thc patient is requesting the
assistance.A physician cannot ethically honor a patient's wishcs to bc provided with the
meansto end his life.
EUTHANASIA
fith euthanasiathe physician goesevenL.rrtherin ending a patient'slife than in physician-
assistcdsuicide. Euthanasia actually means that the health-care worker is prescribing and
administeringthe method of death. Thele is no place in the United Stateswhere eutha-
nasiais legal.A physician cannot legally administer a lethal injection or any other form of
therapythat will help end life. This is true even if the patient is preterminal. Fluthanasiais
cthicallyunacceptable.
TERMINATSEDATIONOFTHE"tAWOFDOUBTEEFFECT"
Thereis an enormous difference between administering a lethal injection and givir.rgpain
medicationsthat might inadvertentlyshortena patient'slife. The issueis one of intent. If
theintent is to end life, it is wrong. If thc intent is to relievesuffering and accidentally-as
anunintended effect-the patient's life is shortened,then the treatment is acceptable.This
is comparable to the differencc bctwcen a charitable donation and being robbed. If I give
S1,000to charityto help othersit is avirtue. IfI stealevenonedollar from you,it is a crime.
Both eventsresult in thc transfer of monev. however.the ethical distinction is erormous.
For example, a 67 year old man is admitted with metastaticprostate cancerto the
bones.He is in cxcruciating pain despiteyour present treatment. He has a history
ay
..n
1e
is
te
le
If
'e
]S
lt
o
'I
s
!!!!p iiaeorcer
I chapterFive
t!l!!p rueorcar-
of COPDandthehousestaffareconccrnedthatincreasinghisparnmedications
will decreasehis rcspiratorydrive.What shoulcll.ou do?
Aslongasyou arenot purposelygivinghigh_doseopiatesin orderto endthepatient,slife.
it is acceptabieto givethepain medicatior.rsevenif it might decreasehis rcspiratorvdrive.
Theprimary ethicalduty is to rclievesuffering.you cannot.iustleavetl.repatientto suffer.
Givethepatienttheamountof ntedicationtheyneedto rclievetheparncvenif, uninten_
tionally,thereisan adversecffecton the respiratorydrive.
FUTITECARE
Thephysicianis not urder an obrigationto givetreatmentor perform teststhat wi rot
benefitthepatient.Thisistrueevenif thepatientor the familyisdemandingit.
.I.he
major
problemin withholdingor withdrawingtherapyon thebasisof it beingfutileisbeingsure
that therewill beno bencfit.It is hardalwaystr becertainif the treat'rent rvill not herp.If
it isclearthattherewill beno benefitthcnyoushoLrldnot qiveit.
For example,a 57 year-oldwoman with cryptogeniccirrhosisis uncleryour
care.Sheis septicandhassevcrevaricealbleedingaswellasencephalopathvnot
respondingto lactulose.Sheis hypotensiveand on pressorsaslvell asintubated
from respiratoryfailure and you expecther to die from her liver diseasein the
next few days.ShedevelopsHepatorenalsyndromeand hasdevelopeduremia.
The family is requestingplacementof a fistulafor rlialysis.What shouldyou tcll
them?
In this case,thereisa clearurderlying preterminarcondition.you shouldnot startdiarvsis
or placethe fistula.Dialysisin this casewi| not changethe outcome.DirJysisin this casc
would not prolong meaningfullife. Becausein this casetlialysisrvouldonly prolong the
dyingprocess,withholdingit is ethical,evenif thefamily is requestingn.
DETERMINATIONOFDEATHANDBRAINDEATH
'lhc
trvo nethods of def ing death arc ternination of heartbeat and brain death. rf the
heart is still beating,but thc paticnt is brain dead,thcn thc person is clead.Brain_death
criteria have enormous significance for the abilitv to harvest organs for donation as well
asin criminal cases.
Br
Br
an
br
Br
Yc
of
st
br
of
Br
of
p.
u!
tn
End-of-Lifetssues
I
i9
t sLife,
drive.
suffer.
inten
ll not
malor
gsure
elp.If
For example, a man is arrested fcrr arrned robbery in which hc assaultsanothcr
man. The victim has sustained cerebral hcrniation and has lost all spontaneous
respirations,cognitive function, and brainstem reflexes.you are calledasan expert
witnessto advisethe court. The allegedassailant,sclefenselawyer tellsthe.judgethat
thechargeonhisclientshouldor.ybcassaultandbattery,notmurcler,becausethe
patient'sheart is still beating. The defcnselawyer contends that the victim can bc
alivefor many ycars ir.rthis condition.
'l,hc
ma,rimum penaltv in some statesfor
murdcr is life imprisonment or cxecution.The penaltyfor assaultmay be onlv l0
to 20 yearsin prison. What shoulcl vou tell the juclge?
Braindeath is the legal definition of death. An assaultleading to brain death is a murder.
Braindeathis irreversibleand permanent.A bcatingheart that maintainsbrood pressure
andpulse does not equal being alivc. When wc, as physicians, determine the criteria for
braindeath are present,this is the legally acceptedstandard of death.
Braindeathis a lossof brainstemreflexessuchas:
I
,t
c
L.
I
Pupilary light reflex
Cornealreflexes
Oculocephalic(doll's cyes)reflexes
Calotic responsesto icedrvaterstintulation of the tynrpanic membrane
fhe.rb:en(cof .furrt.rneorrsrespir.rtions
vsis
case
gthe
f the
teath
well
bu candeternine lossof respirations by rentoving the ventilator and observing for signs
of respiration.lf the criteria for brain death are rnet, theD an EEG or ccrebral blood-flow
stud)'arenot necessary.In other words, the clinical criteria of the absenceof breathins and
brainstcmreflexesaremore important than an EEG.This is becauseEECactivitywou]d be
of limited mcaning if a patier.rtmects tl.reclinical brain-death crjteria.
Braindcath should only be determined to be present if you have excluded other causes
of markedly decreasedbrainstem and rcspiratory function. you must be certain that the
patientis not suffering fron an overdoseof barbiturates,hypothermia, hypotension, or the
useof neuromuscularblockingagentssuclraspancuroniumrvecurontum,or succinvlcho
line.Thesecanall simulatebrain death.
For example, a 35-year-old yroman is admitted aftcr having a serzurear a parr)i.
HerheadCT scanshowsan intracranialbleed.Sheis intubatedbecauseof the loss
of spoDtaneousrespiration.Therearc no pupilary,corneal,oculocephalic,or cold
caloric reflexeselicited. Which of the follorving shoultl you clonext?
I ( A P L A N
H
MEDICAL
r |
$rapterrive
a,
b.
c.
d.
e.
EEG
EKG
Urine toxicologyscreen
Psychiatriccvaluation
Ethicscommitteeevaluation
This patient hasmet most of the criteria for brain death.An EKG j
nosedeathat anypoint. Auscultationof the heartis all ,h", i, ..;'"irt":",:t::::t#tlf:
stoppingof the hcart'you shouldexcrucreintoxicationwith cNs depressantdrugsandhypothermiaprior to determiningthatthe patientis brain dead.
Brain deathdoesnot specificallyrequiredeterminationby a neurologistif the physician
managingthe patientis comfortablewith the criteriadescribedanclhow to verify them.Thisis similarto not needingapsychiatristin orderto determinccapacity.If thepatientisclearlynot braindeadbecausetheirpupilsarcreactiveor theyhavespontaneousbreathing,
a neurologistis unnecessary.
If thepatientisbrain dead,thentheyaredead.Thephysiciandoesnor needa court orderor a relative'spern.rissionto removelife support.Thedoctordoesnot needto askanyone,spermissionto stop the ventilatoror.other treatment,althoughUSMLEwill alwayswantyou to answer"discuss
with thefamily,,',,explainyour finding.s,,,or ..build
consensus,,firstratherthanjust turning offthe ventirator.Thismaysee- .nritrudi.tory.voushouldalwaysexplainwhat brain deathmeansto the patient,sfanily. you needto answer.,explainthemeaningof brain deathto the family,,asthe first choiceif it appears.
on the otherhand'the farnilyi permissionor conscntis not requiredfor terminatinglifesupport,becausea personwhois brain cleadis considereddead.Insurancecompanreswillnot pay for the hospitalizationor managcmentof thosepatientswl.roaredead.you canharvestorgansfor transplantationfrom abrain-deadpersonif thefamilyconsents.you donot haveto waitfor theperson,sheartto stop.Actually,it ispreferableto removetheorganswhiletheheart isstill beatingbecausetheviabilityof th" t."nrplur.,t.aorganstronglycor_relatesto how longit wasunperfusedafterremovalfrom the donor,sbodv-
A
te
c(
sl
te
at
Pi
P(
AI
n(
T]
th
at
In
of
fe
!(AplAN ) MFDtcAl
l + r
Chapter6: Reproductivelssues
ABORTION
Ar adultwoman hasan unrestricted acccssto abortion through the end ofthe first trimes-
terFirst-trimesterabortions are clearly unrestricted. Women do not need the approval or
consentofanyone elseto obtain a first-trimester abortion. In the secondtrimester the deci_
sionisstill betweena woman and her physician, but the easeof accessto a second trimes_
terabortionis lessclear Statesmay place regulations on free accessto a second_trimester
abortion.However, second-trimester abortions arc preponderantly still performed at the
patient'sdiscretion. Third-trimester abortions are not freely available,becausethe fetus is
p'tentiall)'viable.Third-trimester abortions are clearryrestricted.consent by the father for
anabortionis not required; the fetus is consideredasa part of the woman,sbody antl does
nothavethe individual rights of,personhood' until after birth.
Thereis no compulsion on the part of the physician to perform an abortron if performing
thisprocedureis ethically unacceptableto the physician. The patient hasa right to have an
abortion,but they don't have a right to force you to do it, if it is objectionable to you.
Inaddition,it is considered unethical for a patient to seekan abortion for the purposes
otgendcrselection.It is considered ethically unacceptableto creterminethe gender of the
fetusandthen abort the fetus if the sexin unacceptableto the patient.
Forexample,a 23-year-old woman contesto your office seekingan abortion. you
performgynecologicproceduresand you havebeen trained to d-oabortions in the
past.However,you no longer find it morally acceptableto perform the procedure
althoughyou know how. The paticnt is insisting on having the abortion and is
angrywith you for
"abandoning
your parjcnr...,!'hatshouldyou tell her?
42
|
chaptersix
O:
I q:"*t rule,if therc is a procedurethat the patientwantsbut that you do not feelethicallycomfortableperfornir.rg,you shouldrcferthe patientto onoth.. physician.you
cannotbecompelredto accepta paticntyou d., not wantor to do aprocedurewith rvhich
),oudo not agrce.J'hephysicianmustvoluntarilyagreeto the relationship.
CONTRACEPTION
Thcre is no limitation on the accessto contraception for either a lran or a wontan. lt iscntirelyat the discretion of the patient. This is equally t..r. fo, ,rri,.,o.r. Contraception isone of the issuesfor which a n.rin,
is not necessaryto ot,anir,'.onrro.lJ,ls
considered partially emancipated Parental consent
STERII.IZATION
Both women and men have free accessto sterilization. Consent is or. y necessaryfrom thepatient. Spousalconsentis not I
inslaborti,o',.on.,u."p,io,,,ona",:J.fi1,*'*l,jIl';#J:ri:..Tft:::il:?:f;spouse.Each person has autonomv over hr.sor her own bodv.
MINORS
l"j:11^.:.1:, l,
not necessaryfor cithercontraceptionor prenatatcare.ln casesinvolr-lng reproductiveissuesfor minor-s,the ansrverrvi te eitherto just treatthc minor or to-encourage
discussion,'with the
usMrF;r,t'r;;;;r";;:;::".*:1,':il1"J*l:..."x,:;ff:T::i[?:::,.,,J;issuesinvolving minors. The rules on parental consent in the caseof abortion are lessclear.Some statesrequire parental consent for abortions and others do not. BecauseUSMLE isa national exanrination and the rules on parental _r*n, f'J"rion vary frorn state tostate,there cannot be a singleanswerthat cither says,,noparental consent,,or
,,Ies,
parentalconsent is necessary.,,Therefore, the ansrverwill be,.eniourage the patient to discusstherssuelvith parents.',
D
T
Ir
T]
d(
I ( A P L A N MEDICAL
Reproductivetssues
I
DONATIONOFSPERMANDEGGS
#::il,::,:T:1."1i,:l"i:"j::::::"J^':::10'l',r':donatespernandunfertirizedeggsIn addition, paymeut nlay be recei, , -
--.---"t sl'crrD ano untertrfized eggs.
rvords,there is nn l.-.r
/eo ror spcrm ilncl unti:rtilizecl cgg donations. In ot_h"errvords,thereis no legalor ethicalc,
-' Lcggoonattons.ln
lherr.ic h^-,-.,-_
onrrnndr.rtion t,r sellingsperntand unfertilized
donated,but not sold.
Thercis, however,a prohibition ..
'' Jlrurr. sPcror ano un
.loner.rr.,,+_^. - rr
Lgainstselling tertilizecleggs.Fertiljzcd eggsmayy b e
lo rot feel
lclan.you
'ith rvhich
nan.It is
eptionis
consent
romthe
includ
r of thc
nvolr-
r o r t o
essarl)
uctive
clear.
ILEis
ateto
rental
;sthe
qj:19 MTDICAL
Chapter7. OrganandTissueDonation
l 4 s
AUTONOMYOFTHEDONOR
Organand tissuedonation is a voluntary event entirely at the discretion of the live donor,
Theprinciple of autonomy is fully in play here.
For example, a 35-year-old-man is dying of hepatic failure. His brother is fully
HLA rnatched and a highly compatible donor. There are no other donors at this
time and the patient will likely not survive long enough to find another donor. You
arescreeningthe brother for the donation, but hc is not willing to undergo the
surgeryfor the partial donation.What shouldyou do?
Thereis nothing you or anyone-including a court of law can do to compel a person to
donatean organ or tissueif he clearly choosesnot to do so.The needof the recipient hasno
impacton mandatin€la donor to donate.This is true evenif the donation is uncomplicated
forthe donor and the recipient will die without it.
ORGANDONORNETWORKASKSFORCONSENTFORDONATION
For example, a 30-year-old woman is your patient in the intensive care unit for
respiratory failure. The patient hashad a motor vehicle accident and hassustained
a massiveintracranial hemorrhage. The patient is brain dead and will be removed
from the ventilator. You know that there are numerous patients in your hospital
waiting for organs. The family of the patient is with you. You have an excellent
relationship with the family and they trust you. What should you do about the
donation?
I(APLA'I MFDICAL
46
I chapterseven
Only the organ donor neturcrk or uniform network for organ sharing shoulci obtain con_
sent for an organ donation. The medical tcant taking care of the patient shoukl not ask
for the donation. Even if your relationship witr.r the familv is excelent, the organ donor
network hasan enormouslygreatersuccessrate in obtaining consent.physiciansthat ask
for consent for organ donatiolr are far rnore likely to be refused.Becausea greater number
.f refusalswould leadto a lossof potentiarorgansfor donation when the shortagestatute
of availableorgansis critical,it is againstthe raw f.r doctors to obtain this consent.By
legalstatute,only thosespecificallytrained to obtain consentfor orqan donation should
approachthe family for this consent.
In addition, thereis thc perceptionof an enormous conflict of intereston the part of the
family n'hen a caregiverattempts to obtain consent.{4ren a caregivcr asksfor consent,it
leadssome familiesto believethat the health-careteam is not doing everythingpossible
to preservethe lifc of the paticnt. This makes it scer' that the priority of the health-care
team is to obtain organs. It is essentiarfor the hcarth-care teanl to preserve its reration
ship rvith thc family asthe advocatesfor prcserving the rife of the patient. In addition, the
organdonation networkhasa much greaterchanccof obtainingcolscnt, soif the primarv
health careteamtries to obtain consent,it cor.rldlcad to a lossof neededorsans.
PAYMENTFORDONATIONS
With the exception of renewatrletissuessuch as spermr unfertilizcd eggs,and blood, pay_
n.rcntfor organs is consideredethicarly unacceptable.peoplc must not bc in the busincssof
sellingorgans.The economicaspectsof organ clonationmust be ninimizecl so that pcople
believethat the patients who need organs the most w I get them, not that the rvealthvwill
get preferential treatment. It is,howeveq acceptableto cover the costto the donor of dona-
tior.r.
'l
here is a diff'erencebetwecn reimbursing the donor for the cost of donation and
creatinga finalcial incentivcfor pcopleto,,sell',organs.
ORGANDONORCARDS
Although an organ donor card givesan indication of a patient'swishcs for donation, fam
ily consent is still necessaryfor donation. Family objection can overrule the organ donor
card.
P
II
o
S
rl
r(
Pi
al
ca
SC
ir
P
tl
rl
P
P
lc
tl
Tr
tr
fc
al
bi
!t,qptl N) N4r orc.Ar
47
n con
ot ask
donor
Latask
rnbcr
;latutc
nt. By
hould
of the
ent,tt
rssible
r-care
ttion-
n,the
LlTlAIV
pay-
:ssof
eople
ywill
ona-
L a n d
Physiciansand laboratoriesaremandatedto report a nunber of medicaiillnesses.The
mai'purposein reportinginncssesisbothepicremiologicaswenasto rnterruptthespread
ofcertaincommunicablediseases.The illnessesthat arealrval.sreportableincludeAIDS,
q'philis,tuberculosis,gonorrhea,andallofthe childhooddiseasessuchasmeaslcs,mumos.
rubella'andpertussis.Thc list of other reportablediscasesis extensiye.Thesediseasesare
reportableby sonrebody,not necessarilythephysician.
Phvsiciansarealwayslegallyprotectedfor participatingin partnernotification.In gcneral,
thehealthdepartmentperformsthe majority of contact_tracingeventsaswellasnotilying
thosethathavebeenilr closecontactof thepossibity of infection.
'fhe
nameof theso,.rrce
patientisalwaysprotected.
Partnernotificationexistsfor diseasessuchasHIV/AIDS,syphilis,gonorrhea,andtubercu
losis.In addition,thehealthdepartmentcanincarceratepatientswith tuberculosisto pre
r,entthespreadof diseasc.If a patientwon,ttell his or herpartner,thenyou must ansu/er
thatyou must follow your duty to report.If the sourcepatiert still won,ttell his or her
partner,youarewithin your legarright to tell theinnocentthird party.partnernotification
andreportableillnessesarcanexampleof oncof thefewtimesthatthc patrentsautonomy
canbesupersededbecauseofthe necessityofprotectingothcrs.My rightto autonomyends
rvhereyour safetybegins.I havean absoluteright to privacy,exceptwhen my restaurant
ser'esfoodinfectedwithsarmonera,andthenthepatient'srightstoautonomybecomeress
importantthanprotectingothersfron harm.
Tuberculosishasspecialreportingand public healthissues.In additronto doing contact
tracingofthe contactsin orderto do ppD testing,thereisthespeciarrssueof incarceration
fortuberculosis.Patientswith tubercurosisshouldbeisolatedfor abouttwo weeks,whichis
approximatelytheamountof time thatistakesfor sputumto becomenegativefor acid-fast
bacillilf a patientrefusesto takeantituberculosistherapy,physicianshavethe option of
ChapterB: Reportablelllnesses
[am
lnor
MTDICAL
| .r'uo,"rrirr',
lrcarceratingthepatientto preventthemfrom spreadingthedisease.Thisis onlyfor thosewho stiJlhavepositivestainsof their sputumfor acid_faitbacilli.
Incarcerationfor tuberculosisis not thesamething asbeingarrested.It hasnothingtod0with the criminal-justicesystem.The incarcerationo..ur,-in a hosprtalnot in a prison
or jail. Youcannotforce-feedtuberculosismedications,but you canpreventpeoplefromwalkingat their leisurein the community to spreaddisease.Incarcerattonis a lastresort
andis.onlyusedaftera'other optionshavebeenexhaustedin termsot havingdiscussions
with thepatientandofferingdirectlyobservecltherapyat home.
co
A s 
phv
ity r
ther
hen
wh
add
aut(
test
Pre
POS
Altl
nan
righ
HI
ll{!!P urorcer-
l r s
l d o
ison
rom
SOIt
ions
Chapter9: HIV-Relatedlssues
CONFIDENTIATITY
Aswith all medical information there is a presumption of confidentiality on the part ofthe
phvsician.Becauseof the social stigma of HIV there is an additional layer of confidential-
ity and consent required. When a patient enters the hospital or other health-care facility
thereis general consent given that allows the routine testing of blood for chernistry and
hematologyand so on. There is an additional HIV-related consent required to testfor HIV.
hen a patient signs a releaseto distribute or transmit medical information there is an
additionalconsent required for HIV or AIDS-related information. You cannot mandate
automaticHIV testing of patients without their specificinformed consent that you will be
testingfor HIV
For example, a woman com€s at 10 weeksof pregnancy for prenatal care.Shehas
a historv of sexually transmitted diseasessuch asgonorrhea. You offer HIV test-
ing,which the patient refuses,asa routine part of prenatal care.Shereturns at 14
and l8 weeksof pregnancy but is still refusing becauseof anxiety that shemay be
positive.You inform the patient that there are medications that can reduce trans-
mission from mother to child to lessthan 2 percent. She persistsin her refusal.
4rat should you do now?
Althoughthere are medications to prevent transmission of HIV to the fetus during preg-
nancl,you cannot compel mandatory testing of pregnant women. The woman has the
right to refuse testing as well as to refuse antiretrovirals. Therefore, you should offer
HIV testing universally to all pregnant women-but there is no mandatory testing of the
pregnantwoman without her expressconsent to do so.If the woman is found to be HIV-
positiveyou cannot mandate the use of antiretrovirals even though they are safe and
I ( A P L A N MEDICAL
T]
til
50
|
chapterNine
!l!!!p rureorcar-
effective in preventing transmission of the virus from mother to ch d. Althoueh from
time to time, there is aberrancyin the Jegalsysten.rthat tries to prosecutea drug_u.ingor
alcohol-using pregnant woman, the autonomy of the mother lcgalry outweighs thc safety
of the fetus.
For example, an HlV-positive tvoman comes to labor and clelitery at 40 weeks
of pregnancy.Shehas a very low CD4 coult (lessthan 50) and a high viral load
(more than 500,000).You offer her a Caesariansection and intravenous zidovu_
dine, which can cut the transmission rate in half even on the day of delivery. The
woman is anxious, but clearly has the capacity to understand the implications of
this decisionon both her healthand the healthofher child.Sheis stirrrefusinsthe
C sectionand medications.What should you do next?
Fortunately this circumstanceis rare and the HIV perinatar transmission rate in the united
Statesis well undcr 5 percent.However,a rvoman'sright to chooseher own forms ofhealth
careare consideredsuperior to virtually all other treatment concerns.The wrong answerin
a question like this would be to give the medications an1.way,to gct a court order to compel
the patient to take the zidovudine, to askthe father for consentfor either the zidovudine or
the C section, or to sedatethe patient and pcrforn the C sectior.r.
The autonomy of the mother is regarlvsuperior to beneficencefor the fetus. Althoush a
40- lveekfetus is a viable child, the fetus is still inside the woman's body and doesnt become
a person until it is delivered.A noman has the right to refuse HIV testing in pregnancl,
to refuseantiretroviral medicati.ns in pregnancy,ancito refuse a c-section even if it will
markedly benefit the child.
PARTNERNOTIFICATION
Thc highlevelof confidentialityconcerningHIV canonlybebreachedunderveryspecific
cucumstancessuchaswhenthe healthof a third party is at risk.A crrcumstancesuchas
thiswouldbervhenanHIV-positivepersonhasasexualor needle-sharingpartnerthatisat
risk.Themethodof notificationfollowsthestepsof first counseringapatientto notiryhis
partnersvoluntarily'This wourdbeidealandfollowsthegeneralthemeof uSMLE,rvhich
is to first answer"encourage
discussions,,when listedasoneof the choices.If the patient
is eitheremotionallyunableor unwilling to notj$ their partnersthenext stepis to notiry
the Departmentof Healthto startthe processof contacttracing.Thehealthdepartment
interviewsthepatientand attemptsto constructa list of partnersin orderto notify them.
Pi
ol
T
tt
rf
t(
P
y(
yl
st
s(

t
t
v
s
t
[Conrad fisher] kaplan_medical_usmle_medical_ethic(book_fi.org)
[Conrad fisher] kaplan_medical_usmle_medical_ethic(book_fi.org)
[Conrad fisher] kaplan_medical_usmle_medical_ethic(book_fi.org)
[Conrad fisher] kaplan_medical_usmle_medical_ethic(book_fi.org)
[Conrad fisher] kaplan_medical_usmle_medical_ethic(book_fi.org)
[Conrad fisher] kaplan_medical_usmle_medical_ethic(book_fi.org)
[Conrad fisher] kaplan_medical_usmle_medical_ethic(book_fi.org)
[Conrad fisher] kaplan_medical_usmle_medical_ethic(book_fi.org)
[Conrad fisher] kaplan_medical_usmle_medical_ethic(book_fi.org)
[Conrad fisher] kaplan_medical_usmle_medical_ethic(book_fi.org)
[Conrad fisher] kaplan_medical_usmle_medical_ethic(book_fi.org)
[Conrad fisher] kaplan_medical_usmle_medical_ethic(book_fi.org)
[Conrad fisher] kaplan_medical_usmle_medical_ethic(book_fi.org)
[Conrad fisher] kaplan_medical_usmle_medical_ethic(book_fi.org)
[Conrad fisher] kaplan_medical_usmle_medical_ethic(book_fi.org)
[Conrad fisher] kaplan_medical_usmle_medical_ethic(book_fi.org)
[Conrad fisher] kaplan_medical_usmle_medical_ethic(book_fi.org)
[Conrad fisher] kaplan_medical_usmle_medical_ethic(book_fi.org)
[Conrad fisher] kaplan_medical_usmle_medical_ethic(book_fi.org)
[Conrad fisher] kaplan_medical_usmle_medical_ethic(book_fi.org)
[Conrad fisher] kaplan_medical_usmle_medical_ethic(book_fi.org)
[Conrad fisher] kaplan_medical_usmle_medical_ethic(book_fi.org)
[Conrad fisher] kaplan_medical_usmle_medical_ethic(book_fi.org)
[Conrad fisher] kaplan_medical_usmle_medical_ethic(book_fi.org)
[Conrad fisher] kaplan_medical_usmle_medical_ethic(book_fi.org)
[Conrad fisher] kaplan_medical_usmle_medical_ethic(book_fi.org)
[Conrad fisher] kaplan_medical_usmle_medical_ethic(book_fi.org)
[Conrad fisher] kaplan_medical_usmle_medical_ethic(book_fi.org)
[Conrad fisher] kaplan_medical_usmle_medical_ethic(book_fi.org)
[Conrad fisher] kaplan_medical_usmle_medical_ethic(book_fi.org)
[Conrad fisher] kaplan_medical_usmle_medical_ethic(book_fi.org)
[Conrad fisher] kaplan_medical_usmle_medical_ethic(book_fi.org)
[Conrad fisher] kaplan_medical_usmle_medical_ethic(book_fi.org)
[Conrad fisher] kaplan_medical_usmle_medical_ethic(book_fi.org)
[Conrad fisher] kaplan_medical_usmle_medical_ethic(book_fi.org)
[Conrad fisher] kaplan_medical_usmle_medical_ethic(book_fi.org)
[Conrad fisher] kaplan_medical_usmle_medical_ethic(book_fi.org)
[Conrad fisher] kaplan_medical_usmle_medical_ethic(book_fi.org)
[Conrad fisher] kaplan_medical_usmle_medical_ethic(book_fi.org)
[Conrad fisher] kaplan_medical_usmle_medical_ethic(book_fi.org)
[Conrad fisher] kaplan_medical_usmle_medical_ethic(book_fi.org)
[Conrad fisher] kaplan_medical_usmle_medical_ethic(book_fi.org)
[Conrad fisher] kaplan_medical_usmle_medical_ethic(book_fi.org)
[Conrad fisher] kaplan_medical_usmle_medical_ethic(book_fi.org)
[Conrad fisher] kaplan_medical_usmle_medical_ethic(book_fi.org)
[Conrad fisher] kaplan_medical_usmle_medical_ethic(book_fi.org)
[Conrad fisher] kaplan_medical_usmle_medical_ethic(book_fi.org)
[Conrad fisher] kaplan_medical_usmle_medical_ethic(book_fi.org)
[Conrad fisher] kaplan_medical_usmle_medical_ethic(book_fi.org)
[Conrad fisher] kaplan_medical_usmle_medical_ethic(book_fi.org)
[Conrad fisher] kaplan_medical_usmle_medical_ethic(book_fi.org)
[Conrad fisher] kaplan_medical_usmle_medical_ethic(book_fi.org)
[Conrad fisher] kaplan_medical_usmle_medical_ethic(book_fi.org)
[Conrad fisher] kaplan_medical_usmle_medical_ethic(book_fi.org)
[Conrad fisher] kaplan_medical_usmle_medical_ethic(book_fi.org)
[Conrad fisher] kaplan_medical_usmle_medical_ethic(book_fi.org)
[Conrad fisher] kaplan_medical_usmle_medical_ethic(book_fi.org)
[Conrad fisher] kaplan_medical_usmle_medical_ethic(book_fi.org)
[Conrad fisher] kaplan_medical_usmle_medical_ethic(book_fi.org)
[Conrad fisher] kaplan_medical_usmle_medical_ethic(book_fi.org)
[Conrad fisher] kaplan_medical_usmle_medical_ethic(book_fi.org)
[Conrad fisher] kaplan_medical_usmle_medical_ethic(book_fi.org)
[Conrad fisher] kaplan_medical_usmle_medical_ethic(book_fi.org)
[Conrad fisher] kaplan_medical_usmle_medical_ethic(book_fi.org)
[Conrad fisher] kaplan_medical_usmle_medical_ethic(book_fi.org)
[Conrad fisher] kaplan_medical_usmle_medical_ethic(book_fi.org)
[Conrad fisher] kaplan_medical_usmle_medical_ethic(book_fi.org)
[Conrad fisher] kaplan_medical_usmle_medical_ethic(book_fi.org)
[Conrad fisher] kaplan_medical_usmle_medical_ethic(book_fi.org)
[Conrad fisher] kaplan_medical_usmle_medical_ethic(book_fi.org)
[Conrad fisher] kaplan_medical_usmle_medical_ethic(book_fi.org)
[Conrad fisher] kaplan_medical_usmle_medical_ethic(book_fi.org)
[Conrad fisher] kaplan_medical_usmle_medical_ethic(book_fi.org)
[Conrad fisher] kaplan_medical_usmle_medical_ethic(book_fi.org)
[Conrad fisher] kaplan_medical_usmle_medical_ethic(book_fi.org)
[Conrad fisher] kaplan_medical_usmle_medical_ethic(book_fi.org)
[Conrad fisher] kaplan_medical_usmle_medical_ethic(book_fi.org)
[Conrad fisher] kaplan_medical_usmle_medical_ethic(book_fi.org)
[Conrad fisher] kaplan_medical_usmle_medical_ethic(book_fi.org)
[Conrad fisher] kaplan_medical_usmle_medical_ethic(book_fi.org)
[Conrad fisher] kaplan_medical_usmle_medical_ethic(book_fi.org)
[Conrad fisher] kaplan_medical_usmle_medical_ethic(book_fi.org)
[Conrad fisher] kaplan_medical_usmle_medical_ethic(book_fi.org)
[Conrad fisher] kaplan_medical_usmle_medical_ethic(book_fi.org)
[Conrad fisher] kaplan_medical_usmle_medical_ethic(book_fi.org)
[Conrad fisher] kaplan_medical_usmle_medical_ethic(book_fi.org)
[Conrad fisher] kaplan_medical_usmle_medical_ethic(book_fi.org)
[Conrad fisher] kaplan_medical_usmle_medical_ethic(book_fi.org)
[Conrad fisher] kaplan_medical_usmle_medical_ethic(book_fi.org)
[Conrad fisher] kaplan_medical_usmle_medical_ethic(book_fi.org)
[Conrad fisher] kaplan_medical_usmle_medical_ethic(book_fi.org)
[Conrad fisher] kaplan_medical_usmle_medical_ethic(book_fi.org)
[Conrad fisher] kaplan_medical_usmle_medical_ethic(book_fi.org)
[Conrad fisher] kaplan_medical_usmle_medical_ethic(book_fi.org)
[Conrad fisher] kaplan_medical_usmle_medical_ethic(book_fi.org)
[Conrad fisher] kaplan_medical_usmle_medical_ethic(book_fi.org)
[Conrad fisher] kaplan_medical_usmle_medical_ethic(book_fi.org)
[Conrad fisher] kaplan_medical_usmle_medical_ethic(book_fi.org)
[Conrad fisher] kaplan_medical_usmle_medical_ethic(book_fi.org)
[Conrad fisher] kaplan_medical_usmle_medical_ethic(book_fi.org)
[Conrad fisher] kaplan_medical_usmle_medical_ethic(book_fi.org)
[Conrad fisher] kaplan_medical_usmle_medical_ethic(book_fi.org)
[Conrad fisher] kaplan_medical_usmle_medical_ethic(book_fi.org)
[Conrad fisher] kaplan_medical_usmle_medical_ethic(book_fi.org)
[Conrad fisher] kaplan_medical_usmle_medical_ethic(book_fi.org)
[Conrad fisher] kaplan_medical_usmle_medical_ethic(book_fi.org)
[Conrad fisher] kaplan_medical_usmle_medical_ethic(book_fi.org)
[Conrad fisher] kaplan_medical_usmle_medical_ethic(book_fi.org)
[Conrad fisher] kaplan_medical_usmle_medical_ethic(book_fi.org)
[Conrad fisher] kaplan_medical_usmle_medical_ethic(book_fi.org)
[Conrad fisher] kaplan_medical_usmle_medical_ethic(book_fi.org)
[Conrad fisher] kaplan_medical_usmle_medical_ethic(book_fi.org)
[Conrad fisher] kaplan_medical_usmle_medical_ethic(book_fi.org)
[Conrad fisher] kaplan_medical_usmle_medical_ethic(book_fi.org)
[Conrad fisher] kaplan_medical_usmle_medical_ethic(book_fi.org)
[Conrad fisher] kaplan_medical_usmle_medical_ethic(book_fi.org)
[Conrad fisher] kaplan_medical_usmle_medical_ethic(book_fi.org)
[Conrad fisher] kaplan_medical_usmle_medical_ethic(book_fi.org)
[Conrad fisher] kaplan_medical_usmle_medical_ethic(book_fi.org)
[Conrad fisher] kaplan_medical_usmle_medical_ethic(book_fi.org)
[Conrad fisher] kaplan_medical_usmle_medical_ethic(book_fi.org)
[Conrad fisher] kaplan_medical_usmle_medical_ethic(book_fi.org)
[Conrad fisher] kaplan_medical_usmle_medical_ethic(book_fi.org)
[Conrad fisher] kaplan_medical_usmle_medical_ethic(book_fi.org)
[Conrad fisher] kaplan_medical_usmle_medical_ethic(book_fi.org)
[Conrad fisher] kaplan_medical_usmle_medical_ethic(book_fi.org)
[Conrad fisher] kaplan_medical_usmle_medical_ethic(book_fi.org)
[Conrad fisher] kaplan_medical_usmle_medical_ethic(book_fi.org)
[Conrad fisher] kaplan_medical_usmle_medical_ethic(book_fi.org)
[Conrad fisher] kaplan_medical_usmle_medical_ethic(book_fi.org)
[Conrad fisher] kaplan_medical_usmle_medical_ethic(book_fi.org)
[Conrad fisher] kaplan_medical_usmle_medical_ethic(book_fi.org)
[Conrad fisher] kaplan_medical_usmle_medical_ethic(book_fi.org)
[Conrad fisher] kaplan_medical_usmle_medical_ethic(book_fi.org)
[Conrad fisher] kaplan_medical_usmle_medical_ethic(book_fi.org)

Mais conteúdo relacionado

Mais procurados

4 rs of radiobiology
4 rs of radiobiology4 rs of radiobiology
4 rs of radiobiologyDeepaGautam
 
4 R OF RADIOBIOLOGY.pptx
4 R OF RADIOBIOLOGY.pptx4 R OF RADIOBIOLOGY.pptx
4 R OF RADIOBIOLOGY.pptxSwarnita Sahu
 
Radiotherapy Planning For Esophageal Cancers
Radiotherapy Planning For Esophageal CancersRadiotherapy Planning For Esophageal Cancers
Radiotherapy Planning For Esophageal Cancersfondas vakalis
 
Sample size for phase ii clinical trials-Simons design and mcp mod case studies
Sample size for phase ii clinical trials-Simons design and mcp mod case studies Sample size for phase ii clinical trials-Simons design and mcp mod case studies
Sample size for phase ii clinical trials-Simons design and mcp mod case studies nQuery
 
Dose-Response Relationships for Model Normal Tissues
Dose-Response Relationships for Model Normal TissuesDose-Response Relationships for Model Normal Tissues
Dose-Response Relationships for Model Normal TissuesDr.Rashmi Yadav
 
Radioimmunotherapy
RadioimmunotherapyRadioimmunotherapy
RadioimmunotherapyRavi Krishna
 
Hormone therapy in prostate cancer 1
Hormone therapy in prostate cancer 1Hormone therapy in prostate cancer 1
Hormone therapy in prostate cancer 1Deepika Malik
 
Evolution of Fractionation and Conventional Fractionation in Radiotherapy
Evolution of Fractionation and Conventional Fractionation in RadiotherapyEvolution of Fractionation and Conventional Fractionation in Radiotherapy
Evolution of Fractionation and Conventional Fractionation in RadiotherapyNikhil Sebastian
 
4 R's of radiobiology
 4 R's of radiobiology 4 R's of radiobiology
4 R's of radiobiologySwarnita Sahu
 
Basics of linear quadratic model
Basics of linear quadratic modelBasics of linear quadratic model
Basics of linear quadratic modelAjeet Gandhi
 
Optimizing Radiation Therapy for Paediatric Cancers: A Case Study of Medullob...
Optimizing Radiation Therapy for Paediatric Cancers: A Case Study of Medullob...Optimizing Radiation Therapy for Paediatric Cancers: A Case Study of Medullob...
Optimizing Radiation Therapy for Paediatric Cancers: A Case Study of Medullob...Victor Ekpo
 
Dose rate effect in brachytherapy
Dose rate effect in brachytherapyDose rate effect in brachytherapy
Dose rate effect in brachytherapyradiotherapist90
 
Endovascularbrachytherapy
EndovascularbrachytherapyEndovascularbrachytherapy
EndovascularbrachytherapyParag Roy
 
Pancreatic cancer Management (pancreatic adenocarcinoma)
Pancreatic cancer Management (pancreatic adenocarcinoma)Pancreatic cancer Management (pancreatic adenocarcinoma)
Pancreatic cancer Management (pancreatic adenocarcinoma)Dr mohamed Salat Gonjobe
 
Enfortumab Vedotin Drug Monograph & Patient Case Presentation
Enfortumab Vedotin Drug Monograph & Patient Case PresentationEnfortumab Vedotin Drug Monograph & Patient Case Presentation
Enfortumab Vedotin Drug Monograph & Patient Case PresentationJayButani6
 
Bortezomib as standard of care for multiple myeloma
Bortezomib as standard of care for multiple myelomaBortezomib as standard of care for multiple myeloma
Bortezomib as standard of care for multiple myelomaspa718
 

Mais procurados (20)

4 rs of radiobiology
4 rs of radiobiology4 rs of radiobiology
4 rs of radiobiology
 
4 R OF RADIOBIOLOGY.pptx
4 R OF RADIOBIOLOGY.pptx4 R OF RADIOBIOLOGY.pptx
4 R OF RADIOBIOLOGY.pptx
 
Radiotherapy Planning For Esophageal Cancers
Radiotherapy Planning For Esophageal CancersRadiotherapy Planning For Esophageal Cancers
Radiotherapy Planning For Esophageal Cancers
 
Sample size for phase ii clinical trials-Simons design and mcp mod case studies
Sample size for phase ii clinical trials-Simons design and mcp mod case studies Sample size for phase ii clinical trials-Simons design and mcp mod case studies
Sample size for phase ii clinical trials-Simons design and mcp mod case studies
 
Dose-Response Relationships for Model Normal Tissues
Dose-Response Relationships for Model Normal TissuesDose-Response Relationships for Model Normal Tissues
Dose-Response Relationships for Model Normal Tissues
 
Radioimmunotherapy
RadioimmunotherapyRadioimmunotherapy
Radioimmunotherapy
 
Targeted Therapy in Cancer
Targeted Therapy in Cancer Targeted Therapy in Cancer
Targeted Therapy in Cancer
 
Haemostatic RT
Haemostatic RTHaemostatic RT
Haemostatic RT
 
Esophagus cancer
Esophagus cancerEsophagus cancer
Esophagus cancer
 
Hormone therapy in prostate cancer 1
Hormone therapy in prostate cancer 1Hormone therapy in prostate cancer 1
Hormone therapy in prostate cancer 1
 
Evolution of Fractionation and Conventional Fractionation in Radiotherapy
Evolution of Fractionation and Conventional Fractionation in RadiotherapyEvolution of Fractionation and Conventional Fractionation in Radiotherapy
Evolution of Fractionation and Conventional Fractionation in Radiotherapy
 
4 R's of radiobiology
 4 R's of radiobiology 4 R's of radiobiology
4 R's of radiobiology
 
Basics of linear quadratic model
Basics of linear quadratic modelBasics of linear quadratic model
Basics of linear quadratic model
 
Optimizing Radiation Therapy for Paediatric Cancers: A Case Study of Medullob...
Optimizing Radiation Therapy for Paediatric Cancers: A Case Study of Medullob...Optimizing Radiation Therapy for Paediatric Cancers: A Case Study of Medullob...
Optimizing Radiation Therapy for Paediatric Cancers: A Case Study of Medullob...
 
Dose rate effect in brachytherapy
Dose rate effect in brachytherapyDose rate effect in brachytherapy
Dose rate effect in brachytherapy
 
Endovascularbrachytherapy
EndovascularbrachytherapyEndovascularbrachytherapy
Endovascularbrachytherapy
 
Pancreatic cancer Management (pancreatic adenocarcinoma)
Pancreatic cancer Management (pancreatic adenocarcinoma)Pancreatic cancer Management (pancreatic adenocarcinoma)
Pancreatic cancer Management (pancreatic adenocarcinoma)
 
Enfortumab Vedotin Drug Monograph & Patient Case Presentation
Enfortumab Vedotin Drug Monograph & Patient Case PresentationEnfortumab Vedotin Drug Monograph & Patient Case Presentation
Enfortumab Vedotin Drug Monograph & Patient Case Presentation
 
Beta blockers in STEMI
Beta blockers in STEMIBeta blockers in STEMI
Beta blockers in STEMI
 
Bortezomib as standard of care for multiple myeloma
Bortezomib as standard of care for multiple myelomaBortezomib as standard of care for multiple myeloma
Bortezomib as standard of care for multiple myeloma
 

Semelhante a [Conrad fisher] kaplan_medical_usmle_medical_ethic(book_fi.org)

Clark, hulda the cure for all cancers
Clark, hulda   the cure for all cancersClark, hulda   the cure for all cancers
Clark, hulda the cure for all cancersioannis206
 
From Overtreatment to Patient-Centered Care
From Overtreatment to  Patient-Centered CareFrom Overtreatment to  Patient-Centered Care
From Overtreatment to Patient-Centered CareNASHP HealthPolicy
 
Dysphagia Case Study Essay
Dysphagia Case Study EssayDysphagia Case Study Essay
Dysphagia Case Study EssayMichelle Singh
 
HL MAIRA H.od2kkkkkkkkkkkkkkkkkkkkkkkkkkkkkkkkkk
HL MAIRA H.od2kkkkkkkkkkkkkkkkkkkkkkkkkkkkkkkkkkHL MAIRA H.od2kkkkkkkkkkkkkkkkkkkkkkkkkkkkkkkkkk
HL MAIRA H.od2kkkkkkkkkkkkkkkkkkkkkkkkkkkkkkkkkkMairaHussain6
 
23205042
2320504223205042
23205042radgirl
 
Animal Experimentation Successes And Clinical Research...
Animal Experimentation Successes And Clinical Research...Animal Experimentation Successes And Clinical Research...
Animal Experimentation Successes And Clinical Research...Laura Arrigo
 
20 clinical trials
20 clinical trials20 clinical trials
20 clinical trialsemmavan
 
From surviving to thriving: cancer’s next challenge
From surviving to thriving: cancer’s next challengeFrom surviving to thriving: cancer’s next challenge
From surviving to thriving: cancer’s next challengePwC Russia
 
AETCOM module: Bioethics for Undergraduate Medical Students
AETCOM module: Bioethics for Undergraduate Medical StudentsAETCOM module: Bioethics for Undergraduate Medical Students
AETCOM module: Bioethics for Undergraduate Medical Studentslavanyasumanthraj
 

Semelhante a [Conrad fisher] kaplan_medical_usmle_medical_ethic(book_fi.org) (20)

Common medical diagnosis -an algorithmic approach --3rd ed
Common medical diagnosis -an algorithmic approach --3rd edCommon medical diagnosis -an algorithmic approach --3rd ed
Common medical diagnosis -an algorithmic approach --3rd ed
 
The cure for all cancers
The cure for all cancersThe cure for all cancers
The cure for all cancers
 
Clark, hulda the cure for all cancers
Clark, hulda   the cure for all cancersClark, hulda   the cure for all cancers
Clark, hulda the cure for all cancers
 
Hulda
HuldaHulda
Hulda
 
Ethics
EthicsEthics
Ethics
 
From Overtreatment to Patient-Centered Care
From Overtreatment to  Patient-Centered CareFrom Overtreatment to  Patient-Centered Care
From Overtreatment to Patient-Centered Care
 
The Toughest triage
The Toughest triageThe Toughest triage
The Toughest triage
 
Dysphagia Case Study Essay
Dysphagia Case Study EssayDysphagia Case Study Essay
Dysphagia Case Study Essay
 
HL MAIRA H.od2kkkkkkkkkkkkkkkkkkkkkkkkkkkkkkkkkk
HL MAIRA H.od2kkkkkkkkkkkkkkkkkkkkkkkkkkkkkkkkkkHL MAIRA H.od2kkkkkkkkkkkkkkkkkkkkkkkkkkkkkkkkkk
HL MAIRA H.od2kkkkkkkkkkkkkkkkkkkkkkkkkkkkkkkkkk
 
3 e artikel-eng
3 e artikel-eng3 e artikel-eng
3 e artikel-eng
 
aetcom-bioethics.pptx
aetcom-bioethics.pptxaetcom-bioethics.pptx
aetcom-bioethics.pptx
 
23205042
2320504223205042
23205042
 
IMPLANT crswk
IMPLANT crswkIMPLANT crswk
IMPLANT crswk
 
Animal Experimentation Successes And Clinical Research...
Animal Experimentation Successes And Clinical Research...Animal Experimentation Successes And Clinical Research...
Animal Experimentation Successes And Clinical Research...
 
20 clinical trials
20 clinical trials20 clinical trials
20 clinical trials
 
Medical negligence Law (Problem Solving Answer).
Medical negligence Law (Problem Solving Answer).Medical negligence Law (Problem Solving Answer).
Medical negligence Law (Problem Solving Answer).
 
kedir ppt edited.pptx
kedir ppt edited.pptxkedir ppt edited.pptx
kedir ppt edited.pptx
 
HPI.pptx
HPI.pptxHPI.pptx
HPI.pptx
 
From surviving to thriving: cancer’s next challenge
From surviving to thriving: cancer’s next challengeFrom surviving to thriving: cancer’s next challenge
From surviving to thriving: cancer’s next challenge
 
AETCOM module: Bioethics for Undergraduate Medical Students
AETCOM module: Bioethics for Undergraduate Medical StudentsAETCOM module: Bioethics for Undergraduate Medical Students
AETCOM module: Bioethics for Undergraduate Medical Students
 

Último

Call Girls Darjeeling Just Call 9907093804 Top Class Call Girl Service Available
Call Girls Darjeeling Just Call 9907093804 Top Class Call Girl Service AvailableCall Girls Darjeeling Just Call 9907093804 Top Class Call Girl Service Available
Call Girls Darjeeling Just Call 9907093804 Top Class Call Girl Service AvailableDipal Arora
 
♛VVIP Hyderabad Call Girls Chintalkunta🖕7001035870🖕Riya Kappor Top Call Girl ...
♛VVIP Hyderabad Call Girls Chintalkunta🖕7001035870🖕Riya Kappor Top Call Girl ...♛VVIP Hyderabad Call Girls Chintalkunta🖕7001035870🖕Riya Kappor Top Call Girl ...
♛VVIP Hyderabad Call Girls Chintalkunta🖕7001035870🖕Riya Kappor Top Call Girl ...astropune
 
Bangalore Call Girls Hebbal Kempapura Number 7001035870 Meetin With Bangalor...
Bangalore Call Girls Hebbal Kempapura Number 7001035870  Meetin With Bangalor...Bangalore Call Girls Hebbal Kempapura Number 7001035870  Meetin With Bangalor...
Bangalore Call Girls Hebbal Kempapura Number 7001035870 Meetin With Bangalor...narwatsonia7
 
Kesar Bagh Call Girl Price 9548273370 , Lucknow Call Girls Service
Kesar Bagh Call Girl Price 9548273370 , Lucknow Call Girls ServiceKesar Bagh Call Girl Price 9548273370 , Lucknow Call Girls Service
Kesar Bagh Call Girl Price 9548273370 , Lucknow Call Girls Servicemakika9823
 
Lucknow Call girls - 8800925952 - 24x7 service with hotel room
Lucknow Call girls - 8800925952 - 24x7 service with hotel roomLucknow Call girls - 8800925952 - 24x7 service with hotel room
Lucknow Call girls - 8800925952 - 24x7 service with hotel roomdiscovermytutordmt
 
High Profile Call Girls Coimbatore Saanvi☎️ 8250192130 Independent Escort Se...
High Profile Call Girls Coimbatore Saanvi☎️  8250192130 Independent Escort Se...High Profile Call Girls Coimbatore Saanvi☎️  8250192130 Independent Escort Se...
High Profile Call Girls Coimbatore Saanvi☎️ 8250192130 Independent Escort Se...narwatsonia7
 
Call Girls Horamavu WhatsApp Number 7001035870 Meeting With Bangalore Escorts
Call Girls Horamavu WhatsApp Number 7001035870 Meeting With Bangalore EscortsCall Girls Horamavu WhatsApp Number 7001035870 Meeting With Bangalore Escorts
Call Girls Horamavu WhatsApp Number 7001035870 Meeting With Bangalore Escortsvidya singh
 
Call Girls Coimbatore Just Call 9907093804 Top Class Call Girl Service Available
Call Girls Coimbatore Just Call 9907093804 Top Class Call Girl Service AvailableCall Girls Coimbatore Just Call 9907093804 Top Class Call Girl Service Available
Call Girls Coimbatore Just Call 9907093804 Top Class Call Girl Service AvailableDipal Arora
 
Night 7k to 12k Chennai City Center Call Girls 👉👉 7427069034⭐⭐ 100% Genuine E...
Night 7k to 12k Chennai City Center Call Girls 👉👉 7427069034⭐⭐ 100% Genuine E...Night 7k to 12k Chennai City Center Call Girls 👉👉 7427069034⭐⭐ 100% Genuine E...
Night 7k to 12k Chennai City Center Call Girls 👉👉 7427069034⭐⭐ 100% Genuine E...hotbabesbook
 
Chandrapur Call girls 8617370543 Provides all area service COD available
Chandrapur Call girls 8617370543 Provides all area service COD availableChandrapur Call girls 8617370543 Provides all area service COD available
Chandrapur Call girls 8617370543 Provides all area service COD availableDipal Arora
 
VIP Call Girls Tirunelveli Aaradhya 8250192130 Independent Escort Service Tir...
VIP Call Girls Tirunelveli Aaradhya 8250192130 Independent Escort Service Tir...VIP Call Girls Tirunelveli Aaradhya 8250192130 Independent Escort Service Tir...
VIP Call Girls Tirunelveli Aaradhya 8250192130 Independent Escort Service Tir...narwatsonia7
 
Call Girls Service Surat Samaira ❤️🍑 8250192130 👄 Independent Escort Service ...
Call Girls Service Surat Samaira ❤️🍑 8250192130 👄 Independent Escort Service ...Call Girls Service Surat Samaira ❤️🍑 8250192130 👄 Independent Escort Service ...
Call Girls Service Surat Samaira ❤️🍑 8250192130 👄 Independent Escort Service ...CALL GIRLS
 
Call Girl Number in Panvel Mumbai📲 9833363713 💞 Full Night Enjoy
Call Girl Number in Panvel Mumbai📲 9833363713 💞 Full Night EnjoyCall Girl Number in Panvel Mumbai📲 9833363713 💞 Full Night Enjoy
Call Girl Number in Panvel Mumbai📲 9833363713 💞 Full Night Enjoybabeytanya
 
Call Girls Service Jaipur Grishma WhatsApp ❤8445551418 VIP Call Girls Jaipur
Call Girls Service Jaipur Grishma WhatsApp ❤8445551418 VIP Call Girls JaipurCall Girls Service Jaipur Grishma WhatsApp ❤8445551418 VIP Call Girls Jaipur
Call Girls Service Jaipur Grishma WhatsApp ❤8445551418 VIP Call Girls Jaipurparulsinha
 
Call Girls Siliguri Just Call 9907093804 Top Class Call Girl Service Available
Call Girls Siliguri Just Call 9907093804 Top Class Call Girl Service AvailableCall Girls Siliguri Just Call 9907093804 Top Class Call Girl Service Available
Call Girls Siliguri Just Call 9907093804 Top Class Call Girl Service AvailableDipal Arora
 
Vip Call Girls Anna Salai Chennai 👉 8250192130 ❣️💯 Top Class Girls Available
Vip Call Girls Anna Salai Chennai 👉 8250192130 ❣️💯 Top Class Girls AvailableVip Call Girls Anna Salai Chennai 👉 8250192130 ❣️💯 Top Class Girls Available
Vip Call Girls Anna Salai Chennai 👉 8250192130 ❣️💯 Top Class Girls AvailableNehru place Escorts
 
Top Rated Bangalore Call Girls Richmond Circle ⟟ 8250192130 ⟟ Call Me For Gen...
Top Rated Bangalore Call Girls Richmond Circle ⟟ 8250192130 ⟟ Call Me For Gen...Top Rated Bangalore Call Girls Richmond Circle ⟟ 8250192130 ⟟ Call Me For Gen...
Top Rated Bangalore Call Girls Richmond Circle ⟟ 8250192130 ⟟ Call Me For Gen...narwatsonia7
 
Call Girls Colaba Mumbai ❤️ 9920874524 👈 Cash on Delivery
Call Girls Colaba Mumbai ❤️ 9920874524 👈 Cash on DeliveryCall Girls Colaba Mumbai ❤️ 9920874524 👈 Cash on Delivery
Call Girls Colaba Mumbai ❤️ 9920874524 👈 Cash on Deliverynehamumbai
 
VIP Service Call Girls Sindhi Colony 📳 7877925207 For 18+ VIP Call Girl At Th...
VIP Service Call Girls Sindhi Colony 📳 7877925207 For 18+ VIP Call Girl At Th...VIP Service Call Girls Sindhi Colony 📳 7877925207 For 18+ VIP Call Girl At Th...
VIP Service Call Girls Sindhi Colony 📳 7877925207 For 18+ VIP Call Girl At Th...jageshsingh5554
 
Call Girl Number in Vashi Mumbai📲 9833363713 💞 Full Night Enjoy
Call Girl Number in Vashi Mumbai📲 9833363713 💞 Full Night EnjoyCall Girl Number in Vashi Mumbai📲 9833363713 💞 Full Night Enjoy
Call Girl Number in Vashi Mumbai📲 9833363713 💞 Full Night Enjoybabeytanya
 

Último (20)

Call Girls Darjeeling Just Call 9907093804 Top Class Call Girl Service Available
Call Girls Darjeeling Just Call 9907093804 Top Class Call Girl Service AvailableCall Girls Darjeeling Just Call 9907093804 Top Class Call Girl Service Available
Call Girls Darjeeling Just Call 9907093804 Top Class Call Girl Service Available
 
♛VVIP Hyderabad Call Girls Chintalkunta🖕7001035870🖕Riya Kappor Top Call Girl ...
♛VVIP Hyderabad Call Girls Chintalkunta🖕7001035870🖕Riya Kappor Top Call Girl ...♛VVIP Hyderabad Call Girls Chintalkunta🖕7001035870🖕Riya Kappor Top Call Girl ...
♛VVIP Hyderabad Call Girls Chintalkunta🖕7001035870🖕Riya Kappor Top Call Girl ...
 
Bangalore Call Girls Hebbal Kempapura Number 7001035870 Meetin With Bangalor...
Bangalore Call Girls Hebbal Kempapura Number 7001035870  Meetin With Bangalor...Bangalore Call Girls Hebbal Kempapura Number 7001035870  Meetin With Bangalor...
Bangalore Call Girls Hebbal Kempapura Number 7001035870 Meetin With Bangalor...
 
Kesar Bagh Call Girl Price 9548273370 , Lucknow Call Girls Service
Kesar Bagh Call Girl Price 9548273370 , Lucknow Call Girls ServiceKesar Bagh Call Girl Price 9548273370 , Lucknow Call Girls Service
Kesar Bagh Call Girl Price 9548273370 , Lucknow Call Girls Service
 
Lucknow Call girls - 8800925952 - 24x7 service with hotel room
Lucknow Call girls - 8800925952 - 24x7 service with hotel roomLucknow Call girls - 8800925952 - 24x7 service with hotel room
Lucknow Call girls - 8800925952 - 24x7 service with hotel room
 
High Profile Call Girls Coimbatore Saanvi☎️ 8250192130 Independent Escort Se...
High Profile Call Girls Coimbatore Saanvi☎️  8250192130 Independent Escort Se...High Profile Call Girls Coimbatore Saanvi☎️  8250192130 Independent Escort Se...
High Profile Call Girls Coimbatore Saanvi☎️ 8250192130 Independent Escort Se...
 
Call Girls Horamavu WhatsApp Number 7001035870 Meeting With Bangalore Escorts
Call Girls Horamavu WhatsApp Number 7001035870 Meeting With Bangalore EscortsCall Girls Horamavu WhatsApp Number 7001035870 Meeting With Bangalore Escorts
Call Girls Horamavu WhatsApp Number 7001035870 Meeting With Bangalore Escorts
 
Call Girls Coimbatore Just Call 9907093804 Top Class Call Girl Service Available
Call Girls Coimbatore Just Call 9907093804 Top Class Call Girl Service AvailableCall Girls Coimbatore Just Call 9907093804 Top Class Call Girl Service Available
Call Girls Coimbatore Just Call 9907093804 Top Class Call Girl Service Available
 
Night 7k to 12k Chennai City Center Call Girls 👉👉 7427069034⭐⭐ 100% Genuine E...
Night 7k to 12k Chennai City Center Call Girls 👉👉 7427069034⭐⭐ 100% Genuine E...Night 7k to 12k Chennai City Center Call Girls 👉👉 7427069034⭐⭐ 100% Genuine E...
Night 7k to 12k Chennai City Center Call Girls 👉👉 7427069034⭐⭐ 100% Genuine E...
 
Chandrapur Call girls 8617370543 Provides all area service COD available
Chandrapur Call girls 8617370543 Provides all area service COD availableChandrapur Call girls 8617370543 Provides all area service COD available
Chandrapur Call girls 8617370543 Provides all area service COD available
 
VIP Call Girls Tirunelveli Aaradhya 8250192130 Independent Escort Service Tir...
VIP Call Girls Tirunelveli Aaradhya 8250192130 Independent Escort Service Tir...VIP Call Girls Tirunelveli Aaradhya 8250192130 Independent Escort Service Tir...
VIP Call Girls Tirunelveli Aaradhya 8250192130 Independent Escort Service Tir...
 
Call Girls Service Surat Samaira ❤️🍑 8250192130 👄 Independent Escort Service ...
Call Girls Service Surat Samaira ❤️🍑 8250192130 👄 Independent Escort Service ...Call Girls Service Surat Samaira ❤️🍑 8250192130 👄 Independent Escort Service ...
Call Girls Service Surat Samaira ❤️🍑 8250192130 👄 Independent Escort Service ...
 
Call Girl Number in Panvel Mumbai📲 9833363713 💞 Full Night Enjoy
Call Girl Number in Panvel Mumbai📲 9833363713 💞 Full Night EnjoyCall Girl Number in Panvel Mumbai📲 9833363713 💞 Full Night Enjoy
Call Girl Number in Panvel Mumbai📲 9833363713 💞 Full Night Enjoy
 
Call Girls Service Jaipur Grishma WhatsApp ❤8445551418 VIP Call Girls Jaipur
Call Girls Service Jaipur Grishma WhatsApp ❤8445551418 VIP Call Girls JaipurCall Girls Service Jaipur Grishma WhatsApp ❤8445551418 VIP Call Girls Jaipur
Call Girls Service Jaipur Grishma WhatsApp ❤8445551418 VIP Call Girls Jaipur
 
Call Girls Siliguri Just Call 9907093804 Top Class Call Girl Service Available
Call Girls Siliguri Just Call 9907093804 Top Class Call Girl Service AvailableCall Girls Siliguri Just Call 9907093804 Top Class Call Girl Service Available
Call Girls Siliguri Just Call 9907093804 Top Class Call Girl Service Available
 
Vip Call Girls Anna Salai Chennai 👉 8250192130 ❣️💯 Top Class Girls Available
Vip Call Girls Anna Salai Chennai 👉 8250192130 ❣️💯 Top Class Girls AvailableVip Call Girls Anna Salai Chennai 👉 8250192130 ❣️💯 Top Class Girls Available
Vip Call Girls Anna Salai Chennai 👉 8250192130 ❣️💯 Top Class Girls Available
 
Top Rated Bangalore Call Girls Richmond Circle ⟟ 8250192130 ⟟ Call Me For Gen...
Top Rated Bangalore Call Girls Richmond Circle ⟟ 8250192130 ⟟ Call Me For Gen...Top Rated Bangalore Call Girls Richmond Circle ⟟ 8250192130 ⟟ Call Me For Gen...
Top Rated Bangalore Call Girls Richmond Circle ⟟ 8250192130 ⟟ Call Me For Gen...
 
Call Girls Colaba Mumbai ❤️ 9920874524 👈 Cash on Delivery
Call Girls Colaba Mumbai ❤️ 9920874524 👈 Cash on DeliveryCall Girls Colaba Mumbai ❤️ 9920874524 👈 Cash on Delivery
Call Girls Colaba Mumbai ❤️ 9920874524 👈 Cash on Delivery
 
VIP Service Call Girls Sindhi Colony 📳 7877925207 For 18+ VIP Call Girl At Th...
VIP Service Call Girls Sindhi Colony 📳 7877925207 For 18+ VIP Call Girl At Th...VIP Service Call Girls Sindhi Colony 📳 7877925207 For 18+ VIP Call Girl At Th...
VIP Service Call Girls Sindhi Colony 📳 7877925207 For 18+ VIP Call Girl At Th...
 
Call Girl Number in Vashi Mumbai📲 9833363713 💞 Full Night Enjoy
Call Girl Number in Vashi Mumbai📲 9833363713 💞 Full Night EnjoyCall Girl Number in Vashi Mumbai📲 9833363713 💞 Full Night Enjoy
Call Girl Number in Vashi Mumbai📲 9833363713 💞 Full Night Enjoy
 

[Conrad fisher] kaplan_medical_usmle_medical_ethic(book_fi.org)

  • 1.
  • 3. OTHERBOOKSBYKAPLANMEDICAL USMT.ETrStcp1Qbook,Third Edition USMLETrStcp2 CK Qbook,ThirclEdition USMI-E'n'Step2 ClinicalSkills,Third Edition LrSN{l.F.rNrStcp3 Qbook,Third ldition
  • 4. USMLE"Medical The100 YouAreMost on the ConradFischer,M.D. AssociateChiefof Medicinefor Education SUNYDownstateSchoolof Medicine CaterinaOneto,M.D. P U B L I S H I N G New York . Chicogo Cases Likelyto See Exam Ethics: I ( A P t A N
  • 5. USMLE" isa registeredtrademarkof theFederationof StateMedicalBoards(FSMI3)of the United States,Inc.,andtheNationalBoardof MedicalExaniners@(NBMEo),neither of whichsponsorsor endorsesthisproduct. This publicationis designedto provideaccurateand authoritativeinformation in regard to the sub.iectmattercovered.It is soldrvith the understandingthat the publisheris not engagedin renderinglegal,accounting,or otherprofessionalservice.Iflegal adviceor other expertassistanceis required,the serviccsof a competentprofessionalshouldbesought. EditorialDirector:fenniferFarthing Editor:CynthiaC.Yazbek ProductionEditor:LeahStrauss ProductionArtist: EllenGurak Coverl)esigner:CarlySchnur O 2006by ConradFischer Publishedby KaplanPublishing,a divisionof Kaplan,Inc. 888SeventhAvenue NewYork,NY 10106 AIl rightsreserved.Thetextofthis publication,or anypart thereol maynot bereproduced in anymannerwhatsoeverwithout written permissionfrom thepublisher. Printcdin theUnitedtare'of America September2006 10 9 8 7 6 5 4 3 2 | I 3-ISBN:978-I -4195-4209-| l0-ISBN: l-4195-4209-5 KaplanPublishingbooksareavailableat specialquantitydiscountsto usefor salespro- motions, employeepremiums, or educationalpurposes.Pleasecall our SpecialSales Departmentto orderor for moreinformationat 800-6219621,ext.4444,ernailkaplan- pubsales@kaplan.com,or writeto KaplanPublishing,30SouthWackerDrive,Suite2500, Chicago,IL 606067481.
  • 6. ) o f ther ;ard not her :. To: Antonio Oneto Lawyer,Politician, Humanist, Fatheq and Loyal Frienil A man of honor and integrity c.o. To: Truth C.F. :d :s ),
  • 7. Contents INTRODUCTIONANDHOWTOUSETHISBOOK ........xiii ABOUTTHEAUTHOR. .......xv CHAPTERI Autonomy CHAPTER2 CompetenceandtheCapacitytoMakeDecisions.......5 Definitions............ .................5 Minors................... .................5 PsychiatricPatients................. .............................8 Capacityto RefuseProceduresin anC)therwiseMentallyl)isabledPatient..................9 C H A P T E R 5 l n f o r m e d C o n s e n t . . . . . . . . . . . 1 1 A1lOptionsMustBeDescribed........ ................I I All MajorAdverseEffectsMustBeDescribed.............. ...................l2 ConsentIsRequiredfor EachSpecificProcedure ...........................13 BeneficenceIsNot Sufficientto EliminatetheNeedfor Consent................................13 DecisionsMadeWhenCompetentAreValidWhenCapacityIs Lost..........................14 ConsentIsImpliedin anEmergency.................... ...........................15 ThePersonPerforrningtheProcedureShouldObtainConsent..................................l5 TelephoneConsentlsValid.............. ................16 PregnantWomenCanRefuseTherapy........................... ................17 InformedConsentfor a Never-CompetentPerson..................................................,.....17 CHAPTER4 ConfidentialityandMedicalRecords. ......... 19 Confidentiality..... ................t9 Releaseof lnformation....................... ...............20 GiveMedicallnformation to the Patientliirst,Not the Family....................................21 Releaseof Information to Governnental Organizationsand the Courts........,.,,........2l BreakingConfidentialityto PreventHann to Others....... ..---.........22 MedicalRecords.. ......---.......22 CorrectinsMedical RecordErrors..... ...............23
  • 8. CHAPTER6 Reproductivelssues. ........ 41 CHAPTERTOrganandTissueDonation ..........45 CHAPTERSReportablelllnesses .........47 CHAPTERl0 SexuallyTransmittedDiseases(STD$. ....... 5i CHAPTERll Malpractice ........ 55 Definition ............................55 Deviationsfrom LocalStandardsof Care...................... .................56 Informed ConsentProtectionagainstLiability.... ...........................57 Informed Refusalls asImportant asInformed Consent. ...............58 PatientsMust Fully lnform the Physicianof Their MedicalProblems.........................59 RiskManagement ................59 MedicalErrors..... ................59
  • 9. . t J ...25 ...27 ...33 ...35 ...37 ..37 ..37 ..38 ..38 4l . 4 1 .42 .42 .42 .43 45 40 CHAPTER12Doctor/PatientRelationship ......... 6l CHAPTERt4Doctor/DoctorRelationship ......... 7t CHAPTERf5Experimentation.. .........75 PRACTTCEQUESTTONS ..........7s ANSWERSANDEXPLANATIONS... .....129 47 t9 49 50 52 i2 5 ;5 'o 7 8 9 9 9
  • 10. Patient autonomy is the most fundamental principle underlying all health-care ethics. Autonomy grants every competent adult patient the absolute right to do what he wishes rvith his own health care.The concept of autonorny is fundamental to the entirety of the U.S.legal system and has complete acceptanceas an operating principle of day_to_day decisionmaking. Justice cardozo set this as a clear legar precedent in 1914, in the case Schloendorffv. Societyof New york Hospital, 105 N.E. 92 (1914). JusticeCardozo wrote, "Every human being of adult yearsand a sound '.rind hasthe right to determine what shall bedone with his own body and a surgeonwho performs an operation without his patient's consentcommits an assault,for which he is liable in damages,exceptin casesof emergency rvherethe patient is unconscious and where it is necessaryto operate before consent can beobtained."lustice cardozo waswriting concerning the need for informed consentwhen a person undergoes surgery, ln this caseit was made clear that to perform surgery on a patientwithout his direct consent was equivalent to assaultand battery. Patient autonomy is a concept derived fron the property rights issuesthat led to the Declaration of Independence, the U.S. Constitution, and the Bill of Rights. Autonomy overone'sown medical careis seenin the samelight asfreedom of religior, freedom fron.r illegalsearchand seizure,freedom of speech,and freedom of assembly.Theserights are so inlri,'icto our cullurethlt theyrrr conideredaxiomalic.Vithin the l.rrt30 yearrthey havebeen legally extended to cover the freedon to chooseone's own form of health care. Ior example,patients have the right to refuseundesired therapy,and they havethe right to choosewhether or not they will participate in experimentation. Each patient has the right to havehis wishescarried out even in the event that he losesconsciousnessor the caoacitv to make dccisions for himsell Chapter 'l . Autonomy I(APTAN MEDICAL
  • 11. I ChapterOne ln part becausetrcatments such as mechanical ventilation and artificial nutrition didn't exist in the past,r'e need nervlar^'sto delineatepreciselythe ethicsof health care.our abil ity to devclop new forns of therapy hasoutpaccd our ability to createethical svstemsthat detcrmine whethcr or not thesesvstemsr,villbe used in a specific case. . Doesthe developmelltand existenceof a treatnent_such asplacementon a ventila_ tor-mean we should alwaysuseit? . Docsthe fact that we canplacea nasogastrictube in a con.tatosepatient and admin_ ister artificial nutrition that cankecpthat patient alivealmost indefinitely rneanwe shoulddo so? Each patient should decidc the a'swers to tlresequestions for hinrsell rhis is autonomy: I havethe right to do what I choosewith my own body aslong asI understand the consc quencesofmy decisions.There is no form ofproperty more personal than one,sown bodyi so each patient hasthe right to deternire what is done to his body. Theseprinciples may seemobvious, but they are fundamental in medical ethics.This work is an examination of the various cthical situations faced clailyby ph,vsicians. Autonomy representsa patient's right to deter'.rine his or her own health-care decisions. No form of a treatment can be pursuedrvithout his or lter agreerrent,even if the pro posed therapy is "for her own good." Although beneficence-or doing rvhat is good for peoplc-is a high ain and ethical principle, autonomy is considereclmore important and takesprecedence.Eachpatient hasthe right to refusea treatment even ifthat treatment has no adverseeffectsa'd will heip her.You cannot trcat him or her aqainsthis or her wil] even if the treatment is for her benefit. Think of your body as a piece of property such as your house or your car .fhe police do not have the right to searchyour house at their *'him. They m.st obtain a warrant. They cannot even do something benign, such as step inside vour honc, without your express agreenent. Likewise,a physician cannot do a colonoscopv or a CBC without your consent. Unlessyou agreeto it, a physician cannot evcn do a urinalysis or an EKG, eventhough these aremild and uncomplicatedprocedures. Along the same lines, a painter cannot repaint your house without your consent even if he does it for free, the existing house color is ugly, anclthe servicewill only benefit you. A physician cannot treat your pneumonia or remove a cancer even though the procedure or treatment is benign and it will only benefit you. A doctor can violate this autonomy if you are doing something potentially dangerous, such asbuilding a bomb, becausethis activity can harm others.A doctor can violate your MEDICAL
  • 12. cn didn't fur abil- €ms that /entila- lmin- n w e cnomy: ' conse- n body; es may tion of :isions. e Pro- od for rt and nt has I even ce do They press lsent. these en if ,u.A 'e or ous, our onlyifyouhaveHIV,tuberculosis,or asexuallytransmitteddiseasebecause conditionscanharm an innocent third party. Your right to autonomy is only limiterl thereispotentialharm to an innocent third party. Even in thesecases,the violation of@nfidentialityislinited andtightlycontrolled. Theconceptofpatientautonomyissimilarto thatofvoting.Ballotcounterscannotmake presumptionsbasedon rvhomtheythink you rvill vote for. They must definitelyknow whomyouintendedtovotefor whenyou werecastingyour vote.This must bebasedon whomyouspecificallyvotedfor Another personcannotbring your proxy ballot to the electionbasedonwhichcandidatehethinks you shouldvotefor or which candidatewill belpyouthemost. Youmustcastyourvotein amannerbasedonyour clearlystatedwishes.Youhavearight to votefortherveakestcandidate.Youhavethe right to makethe "wrong" choice.In thesame way,adoctorcanonlyrendercarefor you basedon rvhatyou tell heryou want. Patientautonoml givesyou the right to make the wrong choice about your health care. Evenif youareunableto make or expressyour health-care choices,the physician cannot simplymakedecisionsfor,vou that go againstyour wishes. 'l'his standseven if your choices wouldbeharmfulto you. This is one of tlre most difficult things for a physician to under- standandacton. l)octors are trained to act in the Lrestinterest of their patients, but the patient'sright to act againsthis orvn best interest comes first. Patients havethe autonomy offreewill. AutonomyI !!!!p ueorcer -il
  • 13. Chapter2: Competenceandthe Capacityto MakeDecisions DEFINITIONS Competenceis a legal term. Competency decisions transpire within the judicial s1'steni. Onlya court can determine that a patient is incompetent. All adult patients are considered conpetent unlessspecifically proven otherwise. Physicianscan determine whether or not apatient hasthe capacity to understand bis medical condition. The physician makes a determination of the capacity of a patient to comprehend her rnedicalproblems basedon whether there is an organic delirium due to a medical condi- tjon such as a sodium problem, hypoxia, drug intoxication, meningitis, encephalitis, or a psychiatricdisorder.Thesedetcnninations arebasedin large part on a neurological exami- nationtesting memory, comprehension, reasoning,and judgment. Any physician can make this determination. The physician does not have to be a psychiatrist. A psychiatrist may beusefulin rendering decision making capacity determinations in casesthat are complex or equivocal.If the patient obviously does or does not have the capacity to understand, a psychiatristis r.rotneeded. MINORS Bv definition, a minor is a person under the ageof 18.With some exceptions,minors are generallynot considered competent to n.raketheir own decisions.Only a parent or a legal guardiancan give consent for a minor. Neighbors, aunts, uncles,and grandparents cannot giveconsentfor treatment of a minor. This rule doesnot cover life-threateningor seri- ousemergencies.Consent is alwaysimplied for emergencytrcatment. A physician should not withhold blood or surgery in a life-threatening accident just becausethe parent is not Present. I(APLAN MEOICAL
  • 14. I chapterrwo ' For example, a 10-year_oldboy accider.rtallyruns through a glasswi'dou, at school and laceratesthe raclial artery. His teacher brings hirn-to the emcrgcncy dcpart_ mcnt. The boy is bleedingand needsboth a blood transfusionand surgeryto cor_ rect the defect.What should you clo? Emergency treatment of a minor does not neeclexpressrrritten consent. parental consent is implied. saying that you had .o ask a'other p".ror, ru.h as the tcacher, thc principal, the school nurse, the babysitter,or the granclparentsfor consent belore givir.rgcmergency treatment would be the wrong answer.Seekinga court ordcr i, olrn u tv.,,,.rglhoi."l,., ur.,emergency becauseit delaysthe treatment and becausein an emergencyit is implicd that the parents rvould consent if they were there, PartialEmancipation Although only a parent or guardian can give consent for procccluresand therapies for aminor there are some exception_sto this rulc in thc areasof prenatal care,coutraception, sexuallytransmitteddiscases(STDs),and substanceabusc.The mature minor is genera'y one.abovethe ageof I5, although this raries by state.USMLE wilt not make hairsplitting distinctions like giving you a r4-year-old the day before her r5th birlhday. Thc caseon the exam will be clear For example, a l6-year-old girl comes to seeyou in clinic to discusscontracep_ tion. She is generally healthy but is not accompanied bv a parent. What should ,voudo? h all caseslike this involving prenatal care,STDs,contraccption, Hrv anclsubstanceabuse the answer should bc to trcat rhe patient. Saying thut 1.ou_urt notily thc parents, get acourt order)seeklegalcounser,refusetherapy,or to go to the ethicscorr[]rttee areaI incor_ rect answers.These interventions u'wantecrpregnan.y,,.i,i,.,,,ij:::i:: ffiI :;::XlTr:Xil.::,:"T,rj;;::,,,':,fthan it is to take the risk that a teenagcrwill get pregnant and necclan abortion later AboftioninaMinor The rules on parental notification for abortion are lesscrearbecarse there is no national standard.Somestatesrequireparentalnotificationand somedon,t. I(A'I.AN MEDICAL
  • 15. rol rt- )r- )nsent rcipal, 'gency in an d that fora tion, rally rung rthe 'uSe 3 t a :or- ing IOn nal Competenceand the Capacityto MakeDecisions Forexample,a 16-year-oldgirl comcsto seeyou in her first trimesterof preg- nancy.Sheisseekinganabortion.Whatshouldyou do? Inthiscase,thereisno clearansweraboutwhetheror not thephysiciansl.rouldnotif, the parents.Thereisno clearnationalstandardandit dependsupon the stateyou arein. The mostlikelyrightansrverwill indicatethenceclto encouragethc child to notify the parcnts herself,whichwouldbebest.Sothe corrcctanswerchoicewill saysomethir.rglike,,encour- agediscussion,""counsel her to tell the parentsherself,"or "suggest a fan.rilymeeting.,'On theethicsquestionsfor USMLE,if thereis a choiccthat savsto discuss,confer,meet,or havevoluntarynotification,thisrvill generallvbethe right thirg to do first. EmancipatedMinor A smallnumber of minors, particularlyat older agessuchas l6 or 17,may bc considered 'emancipated' or freed of the necd to haveparental consent fbr any medical care.The critc ria arethat the minor is married, self-st4rporting and living indepenclently,in the military, or theparcntofa child that they themselvessupport.The criteriafor beingan emancipated n]inor relateto being no longer dependent on one's parentsfor support, In other words, if theminor does not livc with his parents,has a.job, ar.rdis self-supporting financially, thelr theminor no longeris dependentupon parentalconsentfor his actions, An emancipatedminor is free to make health caredecisionsin all areas,not onh, just STDs. prenatalcare,contraception,or substanceabuse.Seriousmedicalconditionsor procedures suchas organ donation, surgery, or abortion may require a specific court order to allotv the legal standing of enancipation to be fully valid. Only answer "court order,', .,judicial intervention,""court trial," or "seek legal resolution" if the caserepresentsdisagreementor alackof consensusin the stem. LimitationsonParentalRightofRefusalforMinors Although a competent adult can refuse any medical care she wishes,the same right does not automatically extend for parents concerning their children. parents cannot refuse life- saYingtreatment for their child based on religious beliel The state has an intercst in the n'clfareof the child that exceedsthe parental righr to den1.therapy for the child if the child misht die. t ( A p tA N - MEDICAL
  • 16. r I cnupt",r*o For example,a child is in a motor vehicleacciclentand sustainsheadtrauma requiringsurgeryto drain a hematomathat,if leftuntreated,will befatal.Aspart of the surgerythe child will needa blood transfusion.The parentsareJchovah's Witnessesand refuseto giveconsentfor the transfusion.Ihe parents'statedreli giousbeliefsarethat acceptingblood for their child would be a fateworsethan death.4rat shouldyou do? If the child needsblood to savehis life you must givetheblood evenoverthe objection of the parents.It may seemcontradictoryto seekparentalconsentfor a procedurethat you will perform evenif they refuse,but in this case,you shouldattemptto obtain their permissionnonetheless. Withholding lifesavingtherapyfor a child is consideredcomparableto child abuse.The parents'right to practicetheir religionin termsof healthcarewould covertheir abilityto refusea transfusionfor themselves,but not for their child. Thisethicalconcepthasonly expanded.Parentscannotrefusetherapyfor childrenevenif theyareseverelybrain damagedor otherwisedevelopmentallydisabled.Societydoesnot distinguishbetweenindividualsbasedon theirrelative 'worth.'In otherwords,parentscan- not refusetracheo-esophagealfistularepairon apatientwith Down syndromejustbecause thementalcapacityandfunctionalabilityof thechildwill bemuchlessthanthat of achild without this disease.From this point of view,treatmentagainstparents'wishesin a life- threateningsituationisequallyvalidfor both afuturegeniusandachildwith cerebralpalsy who will not achievea mentalageabove2. One of the only timesparentsareallowedto refusecarefor their child is when the child is soill or deformedthat deathis inevitable.This is not a true refusalon the part of the parent.This is reallyjust sayingthat parentscanrefuseonly the futile carethat thedoctor shouldn'tbegivingany.way. PSYCHIATRICPATIENTS A patient'spsychiatrichistoryisintrinsicto theconceptof competenceandto thepatient's capacityto understandher medicalproblems.A patientwith the clearcapacityto under- standor onewho clearlydoesnot havecapacitydoesnot needa psychiatricevaluation. However,a psychiatricevaluationcanbe usefulto help makea determinationof capacity in ecuivocalor questionablecases. l ( n I s c t $:."9 MrDicAL
  • 17. :tion that :heir a t The y t o :nif not ?n- rt's )n, ity ruse rild ife- nl,I rild the tor Competenceand the Capacityto MakeDecisions All suicidalpatients are considered to lack capacity to understand becar.rseactive suicidal ideationis deemedto be a sign of impaircd jr.rdgment.In addition, thc level of compctence necessaryto make financial decisions is different fron.r that necessaryfor an informed refusal.In other words, a patient rnay have a history of bipolar disorder making it impos- siblefor him to manage his financial decisions. Howevel the same person might still be consideredto have capacity to refuserrearment. There is r'virylimitiil iteiriiril liliced on p,ricnlto estJblih(dp.tcilylo rcfuetrcdtntet. CAPACITYTOREFUSEPROCEDURESINANOTHERWISEMENTAIIY DISABTEDPATIENT A patientwith mental illness or mental retardation that might be consideredincompetent for other areasof life may still retain the right to refuse medical procedures. The criteria to determineconlpetencein areasof finance are at a higher standard than those for refus- ing nedical procedures. Your patient might have schizophrenia, mental retardation, or autismto the point of needing to live in a group home, but that does not mean they are incapableof understanding medical procedures.This means that an adult with a mcntal ageof 8 or l0 nay still be allowed to refuse medical procedures.Our society is reluctant to strapa patient to his bed and perform procedures that would be painful or uncomfort ablefor the patient without his consent. For irlstance,certain court casesin the past have allorveda patient with mental illness to refuse diagnostic procedures even though two out of three of the reasonsfor the refusal were delusional. This is an affirn.ration of how deepthe principle of autonony goesin the managenent of patients. In addition, it shows that beneficence-trying to do the right thing for patients is considered lessimportant than autonomy. Autonomy is given more wcight in decision making than beneficience. Autonomy haspriority. A personmay meet the legalstandard of competenceto refuseor acceptmedical careeven if sheis not consideredcompetent in other arcasof life, such asfinancial matters. MEDICAL
  • 18. I r r Att OPTIONSMUSTBEDESCRIBED bu nrust full,vinforrn the patient of the risksanclbenefitsof eachprocedureprior to undergoingthc procedure.Thc explanationmustbe in languagethatthe paticntunder- standsandincludefull infbrnation regardingalternativetreatments. 'I'he paticntcannot makean informcd choicefor onetreatmentif shecloesnot latow of the existenccof oth- For example, you inform a patient about the risks iurd benefits of bone marrow transplantationfcrrchronic myelogenousleukemia.You fully inform the patient about the risk of transplantation,including the possibility of deveJopinggraft versushost disease.Afier the transplantationthe patientdevelopsgraftversushost disease,which is hard to control. The patient learnsthat there is an alternative treatmentcalledimitanib (gleevec),rvhichtloesnot includc the risk of graftversus host disease,but which rvill not cure the lcukemia.The patient filessuit against you. What rvill be the most likely outcomc of the suit? In this casethe patient rvill probablywin the suit becausehe wasnot fully intbrned about the alternativesto the therapics mentioned. The physician has an ethical duty to ir.rfbrm thc patient about all the treirtmcnt options and then allorvthc patient to decidcamong thcm. Although the phvsician'spref-erenceof procedure or treatment may differ from what thc patient chooses,the patient has the option to choosetherapy that may not be ^rltatthe doctordeemsis bestfor hin. Chapter3: InformedConsent l i I(APLAN M E D I C A L
  • 19. ChapterThree !13!9 MEDTcAL AtLMAJORADVERSEEFFECTSMUSTBEDESCRIBED Advcrse effectsanclinjury lrorn rnerlicalcaredo not necessarilyrepresenta mistakc or fail urc of therapy. In the casedescribcd in the previous exantple,the error was not that graft versushost cliseasedeveloped.Thc patient wasfully informed that this could occur and he chose the bone marrow triursplantation anyway.The error rasnot informing thc patient of an alternatir.eoption in treatmcnt. At the same time, a patient could potentially die as an adverseeffect of treatment. This is only an ethical and legalproblen if the adverseevent happcns and the patient was not told that it could have happened.The patient n.ight say, "Doctor, I would never havc taken digoxin if ,youhad told me it night causea rhythm dis turbanceor visualproblenr"or'"I rvoulclneverhavehad surgeryifyou had told mc I might nccd a bloocl transfusion." Thc m.rin point is tt' rcpcct 3utonolly. The patieitt must be ir.rforn.redof the therapeutic options, the adverseeff-ectsof the procedure, and the harm of not undergoingthe procedurc.If thcy havethe capacityto understandand thcy chooseto do it ar.rvr,vay,they havemade an autonomous tlrerapelrtic choice,and therefore,thc patient bearsthe burclenof any adversccffect,not the physician. For example, a man underploescoronary angioplasty. He is inforned that tl.re arterymay rupturc and that thereis a srnallchancehe couldbleedto dcathduring the surgery to repair thc danaged vessel.He knorvs he could havebypasssurgery instead.He understandsand choosesthe angioplasty.He dies from a ruptured blood vessel.The fanily files suit against you. What will be the n.rostlikcly out come? Although it is unfortunatethal the patientdied in this case,thereis no liability with regard to informed conscnt or ethical error. 'l'he patielt was intbrmed of his trcatment options ancithe possiblecomplications,anclhe chosethe treatment. The patient mlrst undcrstand the risks of a procedure just as a drivcr rrust understand the risks before getting behind the wheel of a car. Why can't you sLlca car manufacturer if you die in a car accidcnt?Predominantlybecauseyou are an adult with the capacityto ur.rderstandthe risks of driving anclyou choseto drive an1.way.Tl.reliccnsing processis an education processthat both tries to make you a safedriver, while alsoproperly informing you of the risks of driving. Each time you get in a car,there is i.r.npliedconsent that you are choosing the risk of driving. Evcn if you €letinto a car accident and arreinjured or killecl, the manufacturer has no liability, aslong asthe car is well ntade,trecauseasa compctent adult you choseto put yourselfat risk.
  • 20. tnformedconsent I tr ;raft l h e i€[t e a s r'ent say, A t . Lght t b e r o f e t o ient ard )ns .nd rer to an n8 lre ",] rnt In addition to understanding the risks of the procedurc, you must inforrn the patient of whatcould happen if shedoes not choosethcrapy that you offer. For example, a patient comcs to the emergencydcpartnent with appendicitis. He is inforrned of the risks of surgery,and refusesthc procedurc both verbally and in rvriting. 'l'he patient dies.Vhat was done rvrong here? Thepatientsmust be informcd both of the risk of thc treatmentasrvellasu,hatwill happcn iftheydon'tundcrgothe procedure.In this casethe physicianis liablein court becausehe neverdocunented that he infornred the patient of the possibility of appendiceal rupturc anddcathif the patient did NOT havethc procedurc. CONSENTISREQUIREDFOREACHSPECIFICPROCEDURE If the patient signs a consent form for an opcration on hcr left knee, you cannot, in the operatingroom, decide to operate on her right knee and assumethat you have conscnt. If apatientsignsa consent for an appcnclectomy,but when you open her up you find colon cancetyou cannotjust do the colectomvwithout first informing the patient of the acldi- tionalprocedureand obtainingher consent.Thcre can be no presumptionfor conscntfor anythingbeyond what the patient specifically saiclsheconsenteclto. lither the paticnt has to signconsentin advancefor the other procedures or shehasto regain consciousnessand havethe additional proccdure explained to her. BTNEFICENCEISNOTSUFFICIENTTO ETIMINATETHENEEDFORCONSENT Trving to bc sincere anrl to do good is very important and takes primacy; howeter, the patient'sright to control what happcns r.vithhis own body is more irnportant. For example, a 40 year-old man is undergoing a nasalpolypectomy. In the opcrat- ingroom you seea lesionon thc nasalturbinatethat the frozenscctiondetermines to be a cancer,You have found the cancer early but will need to rcsect the nasal turbinate to cure it. What should you do? M E D I C A LI ( A P I A N
  • 21. t 4 I chapterrhree You cannot rcmo'e tlre cancerousresior.rrvithout the patient's approval. This is true evcn if the physicianis sincere,tale'ted, accurate,and helpful.f'his is truc eve' if the procedure will savethe patient'slife, unlessthe illnessis an emergencyr'nan unconscrotrspatrelt. Beneficcncedoes not elirninate the need for informed consent. lf you live in a very messy apartlnentyour neighborcannot breakinto your apartmentto cleanit el,enif he doesn,t stealanvthil.lg.You must col]sent to the cleaning. His good intcntions are not asimportant asyour right to do what you wanl n'ith your otvn property. DECISIONSMADEWHENCOMPETENT AREVATIDWHENCAPACITYISTOST We must respectthe lastknown wishesof a patient if shelosesthe capacttyto con]munlcate and state those r'r'ishes.Although it is preferable to have thc patient's last k'own wishes docunented in writing, following verbally expressedwishesis perfectly valid. oral co'sent is valid for arrylevelofproceclure if the orar consert can be pro'en. The basisfor varidit). of oral or written conscntis not whctrrerthe procedureis largeor small.Ir.rother words,it is not thc casetltat oral conscnt is valid for a stgmoicloscopybut a brain biopsy requireswrit_ tcn consent.A patient cal.tgive oral consent tbr ir heart transplant if the patient is unable to *'rite. 'l'he onlv diffrculty is that if challenged,orally exprcssedwishes for treatnrent are more difficult to prove than lvritten ones. For example, a 42-ycar-old nran with leukemia repeatcdly refuseschenrotherapy. He losesconsciousless and his rnother tclls you to givc the chentotherapy.What should you tell l.rer? You must rcspectthe l.st kntxvn rvishcsof thc patient. If the paticr.rtcloesnot wa't a treat- ment, you cannotiust $'iritfor him to loseconsciousnessand then perfornt the treatmcnt. If this rverepermissible,ther no one courdhavean estatewiI. The ultimate form of ross of decisitur naking capacitv is death. ,Veurarkeout a will so that when rve lose the capac_ ity to speukfor ourselr.es,our wishes for rvhat to do with our property are respectedafter cleath. For example, a 64-ycar-old roman accontpanied by her husband comes to the emergency room seeking treatment for chest pain. The patient clearJytells you that shewants to haveher aorta repaired and shesignsconsent for the procedure. Shelater becomeshypoter.rsiveand losesconsciousness.Her l.rusbandis now the decision maker and says, "Let her die.,,What do you tell him? t a b a T p P n tl T s T s o Y( OI aI I(A9IAN MEDICAL
  • 22. Informedconsent I ts e eyell :edure rtient. messy oesn't )rtant LlCate lshes rsent tyof , t t l s urit- Lable t are Apatientt family member cannot wait for her to lose consciousnessand then go against thepatient'sprevrous expressedwishes regarding treatments and procedures.In the case above,becausethe patient exprcssedthat shewoulcr riketo haveher aorta repaired her hus- bandcannotgo againstthis aftershclosesconsciousness.The samereasoningholds true if apatientrefusesa procedure or treatnent and then losescolrsciousDess. CONSENTISIMPTIEDINANEMERGENCY For example, a sO-yearold construction worker arrives at the emergency room by ambulance after an accident lacerating his arm. He has lost so much blood he rsunconscious.There is no family member availableto sign consent.What should youdo? Thcmanagementof an emergency is different. Consent is implied rn an emergencyfor a patientwithout the capacity to speakfor himsell This would not apply to a terminally ill patientwith a pre existing DNR order Neither a court order, nor a hospital administrator, nor the ethics committee is re<luired to give permission before the doctor can administer roerap.m an emergency, THEPERSONPERFORMINGTHEPROCEDURE SHOUTDOBTAINCONSENT Thepersonwho is mostknowleclgeableabouttheprocedurcshouldobtaininformedcon sent.Becauscwe must inform the patientabout all the optionsof treatment,risksof the options,and risksof not performingthe procedurein a languagethe patientcanunder- stand,theconsentmustbeobtainedby a personqualificdto maketheexDlanation. Forexemple,you arean intern who hasconsultedsurgeryto placea subclavian centralvenousline,Youonlyknow accessmustbeobtained.you do not knorvwhv theinternaljugularapproachisr.rotbeingused.On thephonethesurgicalresident says,"Can ).ougo getthe consentrvhileI am comingup?',What shouldyou do? Youmustnot bein a position to explainthe risksof proceduresthat you did not decide on.Ifthe patientdevelopsa pneumothoraxandyou do not knowwhy theinternaljugular approachisnot beingused,you cannotadequatelyinform thepatient.you arenot certain eat- ent. loss ,ac- fter j I(APIAN M'DICAL
  • 23. 16 | chapterrhree !(A9rAN ) MEDTCAL why central accessis being obtained at all. You must, at the risk of seeming difficult, tell the surgical residentthat he must obtain the consenthimsell If a complication occurs,you cannot say, "l wasjust gettinga papersigned;I didn't know what it meant." The sameis true for a patient who signsconsent.If you tell the patient that he could havea pneumothorax and might need a chesttube and document this, and the patient still signs consent, then you are not at risk. The patient also cannot say latel "l rvasjust signing a piece of paper.I didn't know rvhat it meant." TETEPHONECONSENTISVATID Consent obtained by a family member, health-care proxy, or other valid surrogatedecision maker is valid even if obtained over the phone. This is a legitimate form of consent by an authorized surrogate decision maker. Forexample, a 65 year-old r.nanis admitted to the hospital for a seizure.The head CT shows a ring or contrast-enhancingi lesion consistent with a brain abscess. The patient remains persistently confused, but is not deteriorating. You need to perform a brain biopsy but there is no family member or health-care proxy who comes to visit hini. His wife is housebound from multiple sclerosisand cannot get to the hospital. You have hcr on the phone but the nurse is refusing to be the witness for the consent, saying that telephone consent is not valid. What should you do? As with all forms of verbal communication, oral advancedirectives,and telePhoneconsent are more difficult to prove if contested.However, they are equally valid. If a health-care worker is uncomfortable taking the telephone consent,useanother member of the health- care team to act asyour witness for the consent.You can educatethe nurse later.You can take consent tbr cardiothoracic surgery over the phone if that is the only way to speakto the surrogate. The real questions about telephone consent are these: 1. Is the personyou arespeakingto reallythe surrogate? 2. Doesthe personknow the paticnt'swishes? 3. Did you get the oral/telephoneconsentwitnessedby another Personso that the person giving consentcannot later deny havjnggivenconsent? P T] wl Sc fr, m ol H c? H s€ Fr o' C n c( ll T n a c b h 1 n c h c t T
  • 24. tnformedconsent I tz It,tell s,Iou uYea signs ringa PREGNANTWOMENCANREFUSETHERAPY Theprevailing consensusholds that a fetus is not a'person'until birth. Hence, no matter whatyour personal feeling may be, the fetus doesnot haveany intrinsic 'rights' asa person. So,even though a 34-week-old fetus would be a viable child if the fetus were removed from the uterus, all health-care decision making and ethics are basedon the choicesof the motherand her interests.Ifparents havea child born at 34 weeksof gestationalagein need of a blood transfusion to saveits life, they cannot refuse lifesaving therapy for the child evenifthey are Jehovah'sWitnesses.The statewould intervene in the interestsof the child. However,if the same child at 34 weeks of gestational ageis still in the uterus, the mother canreluse or accept whatever therapy she wishes without specific regard for the fetus. Hence,a pregnant woman may refusea lifesaving transfusion. Shemay refusea Caesarian sectionto remove the child even if this will put the life of the fetus at risk. Foran1.question concerning reproductive rights, decisionsare basedentirely on maternal wishes.The father has no legal right to make an informed consent fbr any pregnancy- relatedissuebecausethe questions concern the body of the mother. A mother's autonomy overher own body is felt to be more important than the rights of the fetus or of the father. Onlythe mother can sign informed consent for any procedure or treatment during preg- nancy.Any answerchoice that has 'Ask the father. .. " in it will alwaysbe wrong in terms of consentissuesduring pregnancy. INFORMEDCONSENTFORA NEVER-COMPETENTPERSON This is one of the most difficult subjects in ethics becausethe standard of this manage- menthassignificantly evolvedover the last severalyears.Ifthe patient hasDown syndrome andhasa family member to make decisions for her then the question will be straightfor- ward-ask for the consent of the parent or guardian. If there is no parent or guardian, the circumstanceis much more difficult. A third party court designeemust make a decision1, basedon the bestinterestsof the patient even though the patient may neverhur,. ."p..rr.d p her feelingsbefore. The bestway to obtain consent for a person who has lost the capacity to make decisions for him/herself is a health-care prorT or durable power of attorney. This is an advance directive(written or formal). An advancedirective cannot even be given by a patient who hasnever had capacity.The same is true of a living will. The next best method of giving consentis "substituted judgment." In this casea person who knows the patient well tries to determine what decision shewould make for herself if shewere awake.This is also not possiblefor a person who hasnever been competent. The weakestform of consentis to act :nl0n byan d s, 0 0 't e d nsent FCAIE :alth- ucan akto t(APt-Al{ M E D I C A L
  • 25. 18 | chapterrhree in the "best interests"of the patient. Tl.risis the weakestmethod of grvrng consent because it is filled with subjectivity and inprccision. However, it is thc best method of obtainins consent for doctors treating a person who has never had capacity.A legalguar.lian whici could be a fan.ri11.member must make the decision on behaif of the patient. In the absence of a family member the guardian is either appointed by the courts or is the administrator of the health carefacility, such asthe medical director.
  • 26. I rg ause ning hich ence 'ator Chapter4: ConfidentialityandMedicalRecords CONFIDENTIATITY Physicianshave a strong professional mandate to maintain the confidentiality of patients. Communications betseen paticnt and physician are highly privileged and this confiden tialit,vcar.ronly be violated when there is potential harm to a third party or if thcre is a court order demanding the information. Medical information cannot be passedto anyone l ithout thc direct consent of thc patient. Confidentiality also includes keeping a patient's medicalinformation priyate even from his friends and family unless the patient expressly saISit is okay to releasethe information. The fact that a patient may have a good rclation shipwith his familv and fiiends is absolutely no excuseto assumethat the patient wants his medicalinformation passedon to them. I have an exccllent relationship with my mother; hon'ever,even though I am a doctor (or mavbe becauseofit) shedoesnot want me to know herlist of medications. Shehasno obligation to give me a reasonrvhyshedoesnot want rne to know which mcdications she is taklng. If I call her doctor and sa1 "l just want to help mom vvith hcr mecls.What is she on?" Her physician is supposedto respond, "I'm sorry, but IoLrr mother hasn't authorized me to give you that information. I know you mean well, but I iust can't talk to you about your mother's medical problen.rs." For example, a 42-year-old man is hospitalized with chest pain. The patient is arake and alert. His wife comesto you demanding inforn.ration about the patient, saying that she is his wife. She shorvsher identification card verilying this. What shouldyou tcll her? bu cannot releasemedical information to anyone about a patient unlessthe patient gives you pernission to do so.Although it may seem rude and unreasonable,you must tell the MEDICAL
  • 27. 20 ] ChapterFour patient's fan.rilymembers that you must ask your patient for permission befbre you releasehis medicalinfornation. For example, the wife becontesinfuriatcd and storms off the floor, threatening to sueyou. You apologize to the patient for upsetting his wife by not speaking.with her about his private medical problens. The patient responds "On the contrary, Doctor, you did great.Although she is still my wife, we arc finalizing our divorce and we do not live together. I expect to be clivorced and rcmarriecl lvithin the next few months. Sheonly wanted information about me to useagainstme in thc divorce proceeding. Thanks for protecting my confidentiality." REI.EASEOFINFORMATION Information transfer between physicians involved in the care of patietts is a common occurrence. However, the information can only be transferred if the patient has signed a consent or releaseform requesting the transfer of information. It is the patient who must sign the consent to releasethe infornation, not the health,care provider. This is how the system guarantees that the patient's medically privileged inforrnation only transfers to those people to whom the patient rvants it to go. For example, yott receivea phone call from another physician who is rvell known to you in your local community. The physician saysthat one of your former patients has transferred his careto him and he is asking for a copy of the patient'$ meclicalrecord. What do you tcll him? You should tell another physician requesting information to send you the patient's signed releaseform befbre you send him the infonration. !!l!!p r'reorcnr
  • 28. 00n : d a rust the i t o GIVEMEDICALINFORMATIONTOTHE PATIENTFIRST,NOTTHEFAMITY For example, your patient is awaiting the resr.rltsof a biopsy to tell whether or not shehascancer.Her soncallsyou anclasksyou to givehim the infornation bccausc the family is concerned that the bad news will depresshis mothcr. He is sincerc and genuine in his concern. r'{hat do you tell hin? trledicalinformation such asthe result of a biopsy must go to the patient first. There is no basisfor informing the family and not the patient.It is exactlythe oppositc:without directinstruction from the patient, the family shoulcinot receivethe patient's confidential nedicalinformation. Maybethe patient wantsher family to know and maybeshedoesn't. It is alwaysthe patient's decision. There is a rare exception in the caseof a patient with a psychiatricdisturbancein rchom to inform if a meclicalcondition might induce a suicide attel1lpt. RETEASEOFINFORMATIONTOGOVERNMENTAL ORGANIZATIONSANDTHECOURTS For example, an investigator from a local law enforcement a€lencycomes 1(]:' your office. He shows )'ou proper identification stating that he is a government employee.He is looking for your patient's immigration statusand for his medical condition. What do you tell the investigator? If a n.remberof a larv enforcel.rent agencycomes to you with a subpoena or a court order that constitutes a searchwarrant then you must lurnish him with the information that he requests.Ifthe investigator doesnot havea searchwarrant, then you must refusehim access to the files.You are not under any obJigation to make immigration statusinvestigations of vour patients nor to provide this information to third parties unless it is at the request of thepatient. This right of privacy also coversgeneticinformation. You must keepthe medi calinformation private from a patient's co-workers aswell. confidentialityandMedicalRecords I 21 I(APLA''I MEDICAL
  • 29. I ChapterFour BREAKINGCONFIDENTIAI.ITYTOPREVENTHARMTOOTHERS The^right of a patient to privacy is not absolute. .fhere are sonle exceptronsas to whenconfidentialitycanbe brokcn in order to protectothc.s.The Tarasofcasc(1976),in whicha mentally ill paticnt toid thc psychiatristof his intent to harn sonreoDe,is a famousexample of this ln this type of case,thc physician must inform larvenlorcement asrvell asthe potential victim. Confidentialitl. is only broken ir-r,h;, ,;;;; pr.l=nr harrn to orhers;this is rarely donc. Other casesin which it is lawfrl. to break confidentiality inclucle partner notification forsexuallytransmitteddiseasessuchas syphilisand HtV fne patienfs right to confidential_ity in such cascsis lessirr.rportantthan anorher p.rr.r,, ,ish, ;; r;;.ty. Horvever,all effortsmust first bc made to enlist the patient to inform tr.reintiiriate partrer. No lawsuit agai'sta physicia' for brenkingc.nfidcntiality in order to nolify an ir_o..nt,f,la purty that hishealth may be at risk hasbeen successful. MEDICATRECORDS The physician or healtrr-carefacility physicalry owns tr.remedical record, but the informa_holr contained rvithin it is the property of the patient. Although the medical record as ap,hysicaloblect rcnrains al.waysin the hands of ih" h"ultl,_.ar".fir.lfit1,, tt . parient has anabsoluteright to free accessto the information it contar.ns.it "-rrr.o.n,onon containedu'ithin a patientt nedicar rccord is cnveredby a' the same rures of confidentiarity asanyother privilcged meticar information. you cannot relcascthe mecricalrecrrd without theconsent of the patient No one exceptthosc directry involved in th. .ur" ot the patiert hasa right to accessto the record. paticnts cirnuot tJkc sulc *rr""rr.,r, ., ,n" O^ysicalnteclicalrecord but they havea rigltt to acccssor copy thc information. For example, you have a new patient with a complex history r,ho hasbecn tryingto get a copy of her record from her previous doctor. .fhe other practice said shemust provide them rvith a valicl reason for u.hy she needs thc chart. lbu call theother doctor's office trying to get the chart. The practice administrator inforns you that thc paticnt is extremely unpreasantand aim.utt. rr.,uiJrt,un, becausethepatient hasnot paid hcr bill thc prior practicefcelsno obligationto provide youwith the chart. The patient rcturns to seeyou thc tbllowing iav anclasksr.vhathasbecome of her record. What tlo ),ou tell her? I(AP!AN MFDICAL
  • 30. Con{identialityandMedicalRecords I vhen hich lous :llas for tial brts inst his na- N A alt red Thepatienthas a right to her medical records.No one has a right to interfere with this for anyreason.You should tell her that she should be allowed a copy of the chart. A patient doesnot have to give her doctor a reason for requesting hcr own property, and she is entitledto this information whether or not she is "pleasant." Furthermore, the mcdical recordshouldnot be "held hostage"to compela patientto payher medicalbills.The need for information to take care of patients outweighs the physician'sright to payment. CORRECTINGMEDICATRECORDERRORS hen an error in a chart needs correcting the doctor should draw a line through it and theninitial the correction. This allows anyone reading the chart to seewhat was originally thereand it ensuresthat medical errors are not being covereclup. you cannot just remove pagesfrom the chart or cover them over with correction fluid if there are mistakes.This makesit look asif you arehiding medical errors. If you forget to put in a note or document somcthingand want to add it the next day,you cannot put a note in the chart with the old date.If you forgot to put a note in the chart documenting a patient's condition yesterday, ,youcannot write a note today with yesterday'sdate on it. In other words, you canl]ot,back- date'notes.Your notes must alwaysbear the current date and time. rny the !as cal I(AP:-AN MEDICAL
  • 31. WITHHOTDINGANDWITHDRAWATOFMEDICATTREATMENT Everycompetent adult l.ith the capacity to understand his own medical problems hasthe rightto determine what treatments he does or doesnot wish to receive.There is no ethical or legaldistinction between withholding and withdrawal of medical treatrnent. For example, a 60-year-old man with diabetes and hl.pertension develops renal insufficiency to the point of needing dialysis.He is equivocal about spending the restof his life on dialysis,but he agreesto start. The patient is not depressedand is fully alert. Six months after starting dialysis,he comesto realizevery clearly that he absolutelydoes not wish to continue. You have no doubt that the patient has full capacityto understand the implications of this decision. What should _voudo? Although there may be an enotional distinction between withholding dialysis and stop- pingit after it has started,there is no ethical distinction between the two. lf I don,t like to playbasketballthcre is no legal distinction between my never starting to play basketball or playing a few gamesand then not doing it anymore. It is n.ryright to stop. If I hlre you to repair my house, but after a few days I deciclethat I don't like the work you are doing, I havethe right to tell you to stop working on my house.You cannot say, "Sorry, once we startajob we finish it whether the owner likes it or not." I havethe right to refise to allorv youto work on my house and the right to tell you to stop after you started.The patient has theright to stop treatment. For example, an elderly man with COPD progressesto the point of needing mechanical ventilation on a chronic basis.He tells you, alier long consideration, that he just does not want to live on a ventilator. What should you tell hirn? Chapter5: End-of-Lifelssues M F D I C A L
  • 32. 26 Youmusthorrorhiswi.he.I hj r).rti.nri. a- ,,r,.r, .:,L-l mcdicalproblems,sohehas,n.:,:l:t::t l't an adultrvith the capacityto understan<lhis t'ator.r.he*oon*.un,*.,,,n.il:riliil:,,.'i:.iT::::;:"l,",,_TJ asking the family for consent. For example, a 42_year_old reavinshimp-"ry,;;;;;,il1;,i'J.,i,"iTlll[Tl.#il.1],:,i.":*;upbcatand cheerful.He sayshc will gerbetter"r;;;;;; to-tb.n.,orntorrr"dp".manentlyon the ventilator.yo' clearlyinform him .h;; ;; ;r'*r."g and irewillneverinrpror.c.He sayshewantsthe ventirarorrn."*r' n..rr.,,irnerscured.whatdo youtcllhim? ()nceagain,you must honor his understandhis meclical.on.litio,l wishes This is an adult patient with the capacityto therikerhoodorhi;;:;;;;:i":"T-,HX; :.:iil::JillHi,:1:J::*ifjequivalent of incompetencc. Overall, this caservill be ,h" ";;i.:;,.,, agree wirh, becausemost ethicaldilemmas do not inr rather in'olve patients ,.,ho have ,;l-tt u t:nttt:t'. patient wishing to continue care,but drirw treatment. rst capacrtyand the dilemma of whether or not to with_ For example, a woman with Arterarbw_"",r,,,r,"r,..",,1:'iil::?"ill :i."?:::;i:,Til:.::ilj:r::sions.Shehasthecapacityto understandthatshewill ai.,.rriifrcr* ,fr" ar"nsfusion,althoughshcisnot suicidal.Vhatdoyou tellher? Youmusthonorherwishes.She problcms.rhe consequences.r r]l,lt]-^Yl th,the capacityto unclcrstandircrmedical ;::.,'i'"':.:;*'j.,]:H.{*iirFFHil"lr';i::il::il;an cthicsconrnrilteeevaluation.;::l::-i:t llt to ruthorize the proccdurc'or to askfoi capacirytounderstancr,,";:;,;,:l:i;'l';:';:T]11,'::.J,"il:J:::nT.':ji:.i:Tpsyihi.rtricevaluarioni nccc.rr). For example, an HlV-positive,lehovah,s Witness who is now pregnant needs atransfusion to live and haye a deprcssedandisfuuyalert.{hf::t;:1:i, ;|,"..atcsoricauv rcfuses.sheis 'ot 1 t c c T tl r( I I t tl P u cr D h cl tl rl ft: A at lc bi !!!!p Meorcar_
  • 33. .dhis :y to bout t the ause but dth- lbu must honor her wishes.Yon cannot transfuse!t competeDtadult againsther tvill. The situatior.rwould be different if thc patient rvcre a n-rinor.in u,hich cascthe doctor rvould becompclledto transfuse.The fact that the patient is pregnantdoesno1alter the ansrver. Theprevailingconsensusis that personhoodbeginsaftcr birth. Until delivered,the fetus isconsidcredanother part of thc mother's boclv. 'I'he rvrong ansrverswill includc getting a courtordcr or askingthe father of the child for consent.Another wrong ansrverwould be uaitinguntil the paticnt is no longer consciousand then transfusingher. Theanslversto all of the examplesdescribedin this scctionare clearbecausein eachcase thepatientis an adult rvith the capacitl,to understandhis or her nredicalproblems.If the casedescritresdeprcssionin the patientthenyou shouldchoosepsychiatricconsultation,or choosea trial of cither belravjoral therapv or antidepressantntedication asthe ansrvcr. Patientshavethc right to try therapyfor a while ald thcn stop it if it doesnot suit then]. This is true even if it means they will clie from stopping dialysis, mechanical vcntila tion, HIV medications,or blood transfusions.The typc of treatnlent does not change the answer.A CBC or cardiac bypass is ethically antl lcgally indistinguishable. Ticatin€!a palient wjthout consent is legally ecluivalent to assaultand battery or any other form of unwantedtouching.Therefore,in a scnsc,treatinga paticr.rtagainsthis wi)l and without his consentis like mugging the patient or beatinghirn up. Thereis no distinction betweenwithholding anclwithdrarving care.If you are doing sonc thing the patient does not ''ant,you cannot sa,v, "We11, sorry, but I already started, and I rcallvhaveto continuc." ADVANCEDIRECTIVES Definition An advancerlircctive is the tncthod by rvhich a Daticnt conmunicates his wishes for his healthcarein advanceof becoming unable to make dccisionsfor hirrself. The aclvance directiveis a by procluct of the successof nredical thcrapies such as the mechanical vcn tilator that can kccp a patient alive when in the past he would have cliecl.Becauseof thcse therapies,doctorsarenorv in the position of trying 1qictcmine u,hateachpatientwantcd for himself in terms of his health carc.Thc advancedirectivcis part of the conceptof autonomy.The advancedirectiyetellsthc physicianrvhatthe paticnl'swishesaresothat the lcssaccurateforms of dccisionmcLking,suchassubstitutedjudgn.rcntor making a decisiou basedon irnotherperson'sideaof the bcstinterestsof the paticnt,bccomeavoidable. End-of-Lifelssues J 27 ical l b e Ling tor ntt no !!l!!p vrorcnr
  • 34. F Health-CareProxv The ttvo most comnon forms of aclvancedirectives are the living will and the health carc pror:7 Thc health-careprorrr or 'ir.redical power of attorney" is thc durabre power of attor neyfor health-caredecisionsor medicalproxy.The conceptof a,durablc,,powerofattorDey is critical becausethe word "durabre" means it remains in effect evenafter the patient roses decision-making capacity becauseof medical illness.Other forms of legal proxics, such as a finar.rcialproxy, bccome ineffective after the patient losesco'rsciousness.The health carc prorT is a person chosenspecificallyby the patient to 'lake health-caredecisionsfor her in the evcnt that she cannot make decisions for hcrseH T'hesedecisions are limited to health care,not finance.Advance directive docunents ntay also have written instructions to give boundariesto care.For cxample,.rpatient rnay want to receivcantibiotics,but not want to receivechenotherapy or diall.sis.However, the main focus of the proxy is to designate a person of "agent" who speaksto the physician regarding consent issuesfor aI treatments and tests,aswell asdiscussesissuesof withdrawing and withholding treatment.The proT speaksfor the patient. Becausethe patient chooses the prorT as her representative,the proxy overrules all other decision ntakers. Therc is a strong presumption that the proxy knows the patient's wishes.The proxy is not there to give his personal opinion as to what he thinks should be done for tl.repatient. The proxy is there to communicate the patient,s originalwishcsin order to ensurethat they arecarriedout. The pror:y is like a messenger.'I'he patient writcs thc message-hcr wisrresfor her own health care-and the pror:1,delivers the nessage.you woulcl not want your prorT to alter your wishes any more than you rvould want your mailntan to rcwritc your letters. The proxy is also like a waiter. The patient tells the waiter lvhat kind of food he wants to eat (what kind of medicines and testshe rvants).The proxT praccsthc order ir.rthe kitchen. The prory is not there to alter ).our expressedrvishcs.you would not want to order chicken and havethe war'tertell the kitchen you want fish. your waiter tries to understand what you want to eat. 'l'he waiter doesn,twalk up a'cl tell you,,,you look weak and anemic. you are having a rare steaktoniglrt, which is what is bcst for you.,'Now the main diffbrence is that this is a "restaurant" in which thc custon.rcris unconscious and can,t tell you exactly what he wants. The proxy makcs dccisionsbasedon rwo paranteters: l. 1he patient'.sdirectly expressedhealth carewishes 2. 4rat thc patient woulti havewanteclif he/shchad capacitv I crapterrive M F D I C A LI(A PI"AN
  • 35. h-care 'attor- torney t loses uch as h-care her in health o give - want ignate ments proxy e, the ProxF what :ient's lIs to ;hen. cken .you I are that ryhat Thesearethetrvo overriding principles: t. Thehealthcarepror.y must carry out the written and verbal wishesexpressedto him regardingthepatient'shealth care;the basisforwhich is sornetimesjusthis understanding 0fwhatthepatientwould rvantif shewerea'vaketo makethe decision. 2. Theprox,voutweighsall othcr potcntjal dccisionmakcrs,including thc family. For example, a 75-year-old nar arrives at the emergency department febrile, shortof breath, and confused. Many family men.rbersaccornpany the patient, includinghis wife, his siblings, his children, and his grandchildren. The physician 'lvantsto perform an emergency lumbar puncture, which the patient's wife and siblingsare refusing. His 25-year-old granddaughter r,valksup r'vith a health-care prorylbrm signedby the patient designating her asthe proq'. Sheinsiststhat you dotheiumtrar puncture stating that washer understanding ofthe patient's wishes. Therestof the family, including the wife, refusesthe lumbar puncture stating that theyknorv the patient's wishesbetter.What do 1'oudo? [egai',youshould honor the health-care prory aboveall other decision makers,regardless ofthelevelof closenessin biological relationship or frequency of contact. You cannot tell ivhoin a family knows the paticnts wishesbest unlessthe patient is awaketo tell 1'ou.The prox,vdesignationis thc patient's wa). of telling ).ou who he feelswill representhis rvishes. In the absenceof an advancc directive there is a list of relative importance in terms of surrogatedecisionnakcrs. You should start first with the spouse,then parents,then adult children,then siblings,tl.renfriends. This is an approximation only. lf the farnily is split in itsrvishesthcreis no easysolution. When the family is split and there is no proxy, you nrust reterto lhcclhic.eommittecor thccorrrt for a iudsmerrl. LivingWill A living will is a written form of advance directive that outlines the care that a patient l,ould want for herself if she were to lose the ability to communicate or the capacity to undcrstandher medicalproblen.rs.The etiologyof the lossof decision-makingcapacityis irrelevant. A living will can range from being an extremely precise document outlining the exact typesof care that a patient wants or does not want all the way to being a vague, useless documentthat makesnonspecificstatementssuchas "no heroic care."The nain problem End-of-Lifelssues | 2s MEDICAL
  • 36. with the living will is trratmost of the tinrc it lacksprecisionbccausethe patientdoesnot explicitly statewhich testsaud treatmcntsshewants for herself,A clocumentsaying..no extraortlinary cilre" is virtua[y rvortrrless.wrr.t croes "extraordinary carc" nrean?I)oes that mean a yentilatoror chemothcrapy,or dialysis,or bloocitcsts,or all of thcnr, or none of thenr?If the living '' l is cxpricitin listing thc prccisenanes of the tcstsand lreatments that thc patient *'ould rike to receive(or not to receivc),then it is uselul.For instance,a living will that says"No intubatior, no cardiopulmonaryrcsuscitatron,no dialysis,and no blood transfusions" is vcry Lrsefuland allotvs fbr easyfollowinq. For example, a 78-year-old woman ts admitted with metastatic cancer Jeaclingto a changein mcntal status secondaryto hypercalcenria.Shehas a lir.ing will in her recordthat states, "In the cventthat I becomeunableto speakfor mvselffor any reasonI rvish to expressmy r,r,ishthat I not be intubated or placed on a ventilator under any circumst.ruces.I alsodo not wish to reccivedialysis.Blood tcstingand antibioticsareacceptable.,,What shouldyou do? Thc Iiving will is nost valid antr usabrc*.hcn specific tcstsand treatmerts are outrined. In the cascabove' follon' the directio' of the living will ancrcarry out the patient's wishes.A livi'g rvill woulcl overrurethe wishesof the fanrily beca,sctrrc riving will conmunicates the patieDt'so.wn wishes.As a nlatter of autonon),, the patient! clearly expressedrvishes alrvavstakeprecedenceoverthc n ishesof other decisionmakers,suchastamily mentbers. Thc rnajorissue'ith the useof a living will is that it is very difficult, ur advanceof trre'r- ness,to be celtain which medical treatmer.ttsand tcstswill be necessary.It is very difficuit for a layperson to sa1', "I do not tvant an albumin infusion with ny large volume paracen_ tesis,"or 'A biopsytbr diagnosticpLrrposesby interventior.ralradi.logy is acceptablc,but I do not want an opcn biopsvil the operatir.rgroorn,,,or.,l agreeto antibiotics,but not to amphotcricin." A hcalth-care proxy allows for far greatcr tt.*it itity. Hor.vevcl if a patient really does vvritcout the specific nantes of the nlost colftmon treatments and the param_ etersfor their use,ther.rthe living rvill can bc a vcry usefui documelr. NoCapacityandNoAdvanceDirectives Here is u'hat is vcry ciear atrout withholding and withclrarval of caredecisions: ' An adult with capacitycandecideto acceptor refuseanv therapyoflered. . An adult I'ithout capacitycanbe managedr,vitha health_careproxy or a living will if the Iiving l'ill is sufficientlyclearand specificenough. t t p ft p n b cl (ll!!p v:orcer
  • 37. End-of-LifeIssues I 5 l ot -lo at of tS a IO Llnfortunately,the 'ast majority of patients, e'en at oldcr age a'd with life-threatening illnesses,do not havea formal advancedirective. Decision making ca' be rmLchmo.. diffil cultin this circumstance.Ifthe family is united and in agreement,then there is 'o difficulty llith making decisions for the patient. The main issueagain comes to clemonstratjng tlte bestevidenccof lctowing the patier.rt'swishes. Forexample, a 64 year-old man suff-ersa severeiltracrallial bleed leavir]s him conatoseand paralyzed.His wife, sister,and four chjldren arc in the horpital. Theycome to seeyou bccausethey are unanimously asking that you remove the endotrachealtube and lcavethe patient to die. The patient repcatedlymadc this rvishknown to his family. What shoulclyou do? Forapatient without the capacitr to understand his 'redical problens and no hcarth care pror-vor livirg will, the path of rnanagement is clear asrong asa[ of the familv menrbers ar€in agreenlent.You can rcmovc the cnclotrachealtubc and let thc patrcnt dic if everv_ .ne savsthat is what the patient has saitrhe wanted.'rhe endotrach.artube is r medi.al therapylikc any other. The patient has the right to refuseit. The patientt fanrily or otrrers whoknow the patient well can provicle the patient's previously expressedwishes_an oral advanceddirective. For example, a 78-year old wouran has been aclmitted to a nursing homc with adyanceddementia. Shehasdifficulty maintaining oral intake sufficient to survive. Thenursinghome wantsto placea nasogastrictutrefor-fecding.The husbandand the son expresslystatethat the patient slid she.,never*lnted to be ntaintained like a vegetable"ar.rd "I don't want to be put on a ventilator or have a feedins tube dorvn my throat." What should you do? Arethe family members passinplon information about what the patient said, or are thev tellingyou to do what the1,think is bcst fbr the patient? you can do virtually anything in thecareof a patient, even without an aclvancedirecti'e, aslong as the family is united in passingon what the patient said she wanted for her own care. problems arise when the familyis in disagreementabout what the patient said or rvhcn.rnsteadof expressingthe patient'swishes, they are representing what they u.ant for the patient. Having a fan]ily memberexpressa patient's wishes is equivalent to having him cast a vote on the patient,s behall If a person cannot speak,another person can mail in her vote for her if she has clearlytold this person which candidate shervants to vote for. If she told her brother that lvantedto vote republican and he sendsin her ballot stating this, then he is acting on her
  • 38. r wishesasheragent.on the otherhand,if heberievesshewould preferto votedemocratifshewereawakeor hebelicvesonecandidateis betterfor her thentl.risrepresentsa muchlowerlevelof evidencefor hervote.Likewise,if hek'ows what shewantsbecauseshetorrlhim, then he canstopthe ventilator,the blood tcsting,the .h.tno,t "rupy,the dialysis,oranyothertreatment. EthicsCommitteeandReferraltotheCourts ihen there is no clear adyancedirective and the family does not agreeon what the patient wanted for hin.rserfor herself,then the right answer is to refer to the etnrcscommrttee andfinally the courts. Hcre is a list of thc various kinds of consent,with the most desirablelisted first: . Direct patient rvishverbalizedby the patient . Forrnaladvancedircctivesuchasa proxy or a living will . Oral advancedirective . United group of fimily and friends . Croup of family membersdisagreeingon what thc patient would havewanted In caseswhere there is no living will or proxy and the family members are not in agreement, :h. :*:.:,:p is to "encourage consensus,",,request discussion,,,or ..talk with the parties iltvolved" Ifconsensus is not possible,you should refer the caseto the hospital ethicscom_mittee. The last step to pursue in the absenceof a clear consensusis relerral to the courts or "seekjudicial intervention.,, For example,the Terry Schiavocaseended up in the courts becauseher husband and sev_eral of her friends said the patient told them that shc ncver wantecrto be maintained in apersrstentvegetativestate.The patientt parents stated that she never saiclthat. The ethics ::-.TiU:: does not have direct legalpower and their decisions are not alwaFsuniversally binding' Ifthe ethicscommittee is unabrcto bu'cr consensus,or the involved parties ignorethe ethics committee, thcn judicial intervention i, n.."rrn.y. r a t rl d li I(APLAN MEDICAL
  • 39. End-of-Lifetssues I ocrat if r much he told ysis,or atient 'eand "00 NoTRESUSCTTATE-(DNR)ORDERS A "Do Not Resuscitate,,(DNR) or< ;::t*:,f*.:f..H.il?#J+,':i;:*,,jffii,H;[i.*::i::::*::J:;thepatienrttr il;;;;;r/Lxrnrc medications'That is all' nothingmore.Deathof breath. ruddenlossof pulse,bloodpressure,and the ability to Forex&mple,a 42_year_oldHlV_positivernan is admittedfor hematuriathat is ffii:i"ilfiT':11]:t'J;;i HeisDNR'u"r"gvi''..""*r,edandthevthink notwanttodoeith".on.b..or:^"11"1:'-1ry"lllt-Tfhtectomv'however'thevdo whatshouJdyou tell them? sethe"patient isDNR a'd thereforepreterminal." Forexample,a 42_year_oldman with AII)S is bcingtakento the operatingroom j:1:::lfl*.-, r]resurgicarresidenrshrussh;,";;i;;;;.,ays,.oh wel, :i:::::'ff::iffifttli l*l)- tls itcr"oan'trea'vmitie.irth.s,',.g.,vi,of a relicfl"Whatshouldyou tell hrm? DNRdoesnot mean patientssh( resuscitation.DNRt,.;;;";;;;';'#,;',,.._";::f ruli?fJ:J::illJ#:$T:ffiIdorngor servesasan excusefor making" _irtut". oNnlo;;;;.", it is okayjust toletthepatientdie,or thar thc doctor doesnot have,. b. ;;;;;;i,^ patienrcanstilt be 'ties rm- lrts Themostcommonmisunderstandingabout DNR ordersis that beingDNR must mean jT.ffH'"t$lfr",tiJ,:,;:i*",, l",ti..A"'rh.;.;;;;;Jisuncrerstandinsisthat bropsies,or dialysis.o*o * ,r, "'..9i1 other aspectsof routine good caresuch"r r;;;: nomaandneedsab,.0,,"J;;;il:IiJ:;'"Tl]:lh,TiillT',il:[]::::i::fl.; $:lJ:;l;::lT*ffffi:i:::: *o;-p,t.;;,;;d;',ilo**o**;u.,-.u.,, whenapatientdiestt ".. i, o,uaa].ilp"1l^Tj:.1?,iT':t ": th''nd-pointoftherapv.usually ingintravenous;.;t;;;;;r,;;tn' raptoupgradein theamountof thetnpv,,u.h u, giu'- maystiluncrergo,",r;,;;;;;"::;1,;r,T:.,":il.1L..:i"j:T:fi:1,Ti,i:n.Apatint trhenpatientshavecardiopuln.ronaryarrest,thereis presumedconna'resuscitationunlessthe patient specificalryand expressivety;::i.J,X.*:tJi:; 1 a lcs Ilv IE
  • 40. I intubated and maintained on a ventilator if sheis DNR. This is a confusing point because endotrachealintubation is often a part of the normal resuscitativeprocess.If the pl rientir DNR and he loseshis pulse then the doctor would not intubate becausethe intubation here would be consicleredpart of the'Code' or resuscitative/CPRrlanagement. Hon'ever,ifthe patient remains alive and has advancing lung disease,the patient can still be intubated ln . this case,the doctor rvould only defer CPR if the patient were DNR. Doctors Jo not have ' to rernovethe endotrachealtube of all patients who chooseto be DNR. Many patients will not allow themselvesto be made DNR becausethey believethe medical staff will not beas aggressivein their other treatments. For example,a 68 year-oldman with basalcellcanceris aclmittedfor evaluation andtreatmentof a fever.He hasbecnDNR for thelastsixmonths.After the chest X ray,urinal)'sis,andblood culturearedonehestill hasafeverof unknownetiol ogy.When you askthe residentwhy he hasn'tdone more tests,the responseis, "Well, wervill getto it, but thereisn'tmuchof arush,alierall,thcpatientisDNR." What do you tell hin.r? DNR doesnot meanarrythingmorethan deferringCPR,suchaschestconpressionsand electricalcardioversion.In thiscase,it wouldalsoeliminateendotrachealintubationif this treatmentwerepart of the resuscitativceffort.That is all.Thedoctoris expectedto man- agepain andthe diaplnosisof othermedicalproblemsjust asaggressivelyasshewouldin a non-DNR patientunlessthepatienthasspecificallychosento deferthoseotherformsof therapy.Doctorsdo not automaticallyhaveto havethc DNR orderrcversedjust to bein the intensivecareunit or to goto surgery. A T] th o1 rt gi n h I ( A ? L A N M E D I C A L
  • 41. - End-of-Liferssues I ie is 'e € n e lt s FTUIDSANDNUTRITIONISSUES AdultswiththeCapacitytoUnderstand Theartificial administration of fluids a'<1nutrition is a medical procedure and treatmentthatcan be acccpted or refused t othertreatn.rent.,Artificial odr',ir" n competent adult in exactrythe same manner as any rhaneatins..Artinciar,,,o;;*,:::'l::r.ffii:xffi:;T,f,::.Tiil :i::il:l:Sastric'or jejunostomy turre pracement. 'Artificial nutrition" would also refer to intrave-']onslyadministered nutrition such astotar parentcrar nutrition (TpN), also referred to ashyperalin.rentation. For exqmple, a 47-year_oldtheoretical physicistwith amyotrophic lateral sclerosis nasDecomcprogressivelynore clisabledto thc point of beinj virtually immobiie in a rvhcelchairHe is unablc gastricreeding,ut.u,,a,.ru,'.o,lo:illT'::,:',':?tijJil,,";#;'":JJ:l;:j,*: nurse,isinsistingthat you reinsertit. What shouldyou tell her? Forcibleinscrtionby anyoneof an artificial feedingdeviceinto an adurtpatientwith thecapacityto understandthe meaningof its rcmovalis not allowed.If the patient,swishesareclearlyexprcsscd,thisisthesamcasrefusingaventilator,bloodtesting,or dialysis.Thisrefusalis subjectto the samecriteriaasthe refusaiof oth", tl.,.rupiar.you must be surethereisn'ta severedepressi.nunderryingtherefur", "ra ,n"t,i. riiuent undcrstandsthathema,vdie without the tube.If thereis no depression,tn"n tn. fnt,.r.,t hasthc right torefttseatherapye'en ifhe will diewithout it. putting thc tubetru.t in,o apersonwhohasrcfusedit isconsideredthesan.reasassaultandbattery,. AdultsWhoHaveLosttheCapacityto Understand Decidingwhat to do aboutartificialnutrition 'vherthe pati.rt cannotspeakfor herserfisamuchhardercircurnstance.Nutrition is the singlemostdifficult issuern termsof witb_holdingandwithdrawingtreatment.We canneverwithhold ordinarynutrition like foodtoeatandwaterto drink. The stanclardof certaintyregardinga patientt wishesin termsof.artificialnutrition is much higherthan wirh other ,i.ropri."- il"'rnort havevery clearefidenceof whatthepatient,swisheswerein regardto artificialnutrition. I I(AP'AN - M E D I C A L
  • 42. If there is a health-care proxy, and thc healthcare agent says, "The patient clearly told me that they didn't want tube feeds,"you nay withhold the therapy. If there is a living will rvhere the patient themselveswrote, "I do not tvish to have a nasogastric tube or other forms of artificial nutrition," you may withhold the therapy.lf there is no advancedirective, but the family is in uniforrn agreement that the patient had expressedthe wish never to have tube fecds,you may withhold the therapy.If clear wishesregarding fluids and nutri- tion were ncver clearly expressed,thcn there is inplied consent for the feeding basedon the presumption that it would be the patient's wish to be fed and on the f-actthat feeding and hydration are in thc patient's best interest. Emotionally, the standard is different from the standard used to dcternrine whether or not ordinary testing such as blood tests or CT scansshould be donc. The proxy and fanily can say, "The patient never specifically told rne that he would not want a liver biopsy or MRI of the brain, but mv understanding of his wishcs in generalis that he doesn't want to undergo theseprocedurcs."This would not be the same for nutrition. Decisions on with- holding and withdrawing of artificial fluids and nutrition should bc treated the sameway asany other medical treatnent. There is a higher standard of evidencefor decision making in some states.That is why this issueis so complex. Fcw people have left a specific,written document concerningtheir desirefor tube feecling. The evidence has to be clear that a paticnt does not want artificial nutritjon. Thc routine assumption is that rnost people wish to be fed asa part of ordinary care.If the evidenceis not clear,then a referral to an ethics comnittee, or possibly to the courts, is necessary.The level of evidence regarding the patient's wishes necessaryfor withholding and withdraw- ing artificial nutrition hasbeen treated by some courts asthe sameasthe level of evidence required in a criminal case.The system errs on the sidc of caution-perhaps letting a few guilty people go free rather than sending a single innocent person to jail or execution. If you lost consciousnesssuddenly and left no specific instructions, wouldn't you want there to be a need fbr colrvincing evidence that you didn't want to be fed before your relatives rvere able to withhold nutrition and possibly let you die? Ethical consensusholds that decisiolrs to provide or forgo artificial nutrition and hydration (ANH) for patients who lack capacity should be made according to thc same standardsasthose uscd for any other medical treatment. Despite this consensus,ccrtain statesimpose more stringent standards for withholding or withdrawal of ANH compared to other medical trearmenrs. P In hir lif, clr TT nc E A S ac ni th et Pa se th pr to as m m th AI l s $ l BI T TI (1:.^9 MEDTcAL
  • 43. End-of-Lifelssues I me vill hcr ive, to .ri- on rot ilv O I to L PHYSICIAN-ASSISTEDSUICIDE hrphysicianassistedsuicide, the physician provides the patient rvith the means of ending hisownlife.The doctor doesnot actuallyaclministerthe substancethat endsthe patient's life.Although there is national controversy on the issue,the answer on the USMLE is ver1. clear;physician assistedsuicide is alwaysconsidered incorrect and ethically unacceptablc. Thisis true even if there is a local state law permitting the procedure. W1rat is legal does n0tautomaticallyequal rvhat is ethical. Generally,physician assistedsuicide is requestedby patientswho havea terminal diseaseand a limited life expectancyanyway.Neverthelcss,the severityof the diseaseand eventhe discomfortand sufferingof the patient do not change theansu,er.Physician assistedsuicide is alwaysconsideredto bc wrong on the USMLE. The primaryissueis one of intent. Physician assistedsuicide is inimical, or absolutelycontrary, to the role of the physician to savelife. This is true even if thc patient is requesting the assistance.A physician cannot ethically honor a patient's wishcs to bc provided with the meansto end his life. EUTHANASIA fith euthanasiathe physician goesevenL.rrtherin ending a patient'slife than in physician- assistcdsuicide. Euthanasia actually means that the health-care worker is prescribing and administeringthe method of death. Thele is no place in the United Stateswhere eutha- nasiais legal.A physician cannot legally administer a lethal injection or any other form of therapythat will help end life. This is true even if the patient is preterminal. Fluthanasiais cthicallyunacceptable. TERMINATSEDATIONOFTHE"tAWOFDOUBTEEFFECT" Thereis an enormous difference between administering a lethal injection and givir.rgpain medicationsthat might inadvertentlyshortena patient'slife. The issueis one of intent. If theintent is to end life, it is wrong. If thc intent is to relievesuffering and accidentally-as anunintended effect-the patient's life is shortened,then the treatment is acceptable.This is comparable to the differencc bctwcen a charitable donation and being robbed. If I give S1,000to charityto help othersit is avirtue. IfI stealevenonedollar from you,it is a crime. Both eventsresult in thc transfer of monev. however.the ethical distinction is erormous. For example, a 67 year old man is admitted with metastaticprostate cancerto the bones.He is in cxcruciating pain despiteyour present treatment. He has a history ay ..n 1e is te le If 'e ]S lt o 'I s !!!!p iiaeorcer
  • 44. I chapterFive t!l!!p rueorcar- of COPDandthehousestaffareconccrnedthatincreasinghisparnmedications will decreasehis rcspiratorydrive.What shoulcll.ou do? Aslongasyou arenot purposelygivinghigh_doseopiatesin orderto endthepatient,slife. it is acceptabieto givethepain medicatior.rsevenif it might decreasehis rcspiratorvdrive. Theprimary ethicalduty is to rclievesuffering.you cannot.iustleavetl.repatientto suffer. Givethepatienttheamountof ntedicationtheyneedto rclievetheparncvenif, uninten_ tionally,thereisan adversecffecton the respiratorydrive. FUTITECARE Thephysicianis not urder an obrigationto givetreatmentor perform teststhat wi rot benefitthepatient.Thisistrueevenif thepatientor the familyisdemandingit. .I.he major problemin withholdingor withdrawingtherapyon thebasisof it beingfutileisbeingsure that therewill beno bencfit.It is hardalwaystr becertainif the treat'rent rvill not herp.If it isclearthattherewill beno benefitthcnyoushoLrldnot qiveit. For example,a 57 year-oldwoman with cryptogeniccirrhosisis uncleryour care.Sheis septicandhassevcrevaricealbleedingaswellasencephalopathvnot respondingto lactulose.Sheis hypotensiveand on pressorsaslvell asintubated from respiratoryfailure and you expecther to die from her liver diseasein the next few days.ShedevelopsHepatorenalsyndromeand hasdevelopeduremia. The family is requestingplacementof a fistulafor rlialysis.What shouldyou tcll them? In this case,thereisa clearurderlying preterminarcondition.you shouldnot startdiarvsis or placethe fistula.Dialysisin this casewi| not changethe outcome.DirJysisin this casc would not prolong meaningfullife. Becausein this casetlialysisrvouldonly prolong the dyingprocess,withholdingit is ethical,evenif thefamily is requestingn. DETERMINATIONOFDEATHANDBRAINDEATH 'lhc trvo nethods of def ing death arc ternination of heartbeat and brain death. rf the heart is still beating,but thc paticnt is brain dead,thcn thc person is clead.Brain_death criteria have enormous significance for the abilitv to harvest organs for donation as well asin criminal cases. Br Br an br Br Yc of st br of Br of p. u! tn
  • 45. End-of-Lifetssues I i9 t sLife, drive. suffer. inten ll not malor gsure elp.If For example, a man is arrested fcrr arrned robbery in which hc assaultsanothcr man. The victim has sustained cerebral hcrniation and has lost all spontaneous respirations,cognitive function, and brainstem reflexes.you are calledasan expert witnessto advisethe court. The allegedassailant,sclefenselawyer tellsthe.judgethat thechargeonhisclientshouldor.ybcassaultandbattery,notmurcler,becausethe patient'sheart is still beating. The defcnselawyer contends that the victim can bc alivefor many ycars ir.rthis condition. 'l,hc ma,rimum penaltv in some statesfor murdcr is life imprisonment or cxecution.The penaltyfor assaultmay be onlv l0 to 20 yearsin prison. What shoulcl vou tell the juclge? Braindeath is the legal definition of death. An assaultleading to brain death is a murder. Braindeathis irreversibleand permanent.A bcatingheart that maintainsbrood pressure andpulse does not equal being alivc. When wc, as physicians, determine the criteria for braindeath are present,this is the legally acceptedstandard of death. Braindeathis a lossof brainstemreflexessuchas: I ,t c L. I Pupilary light reflex Cornealreflexes Oculocephalic(doll's cyes)reflexes Calotic responsesto icedrvaterstintulation of the tynrpanic membrane fhe.rb:en(cof .furrt.rneorrsrespir.rtions vsis case gthe f the teath well bu candeternine lossof respirations by rentoving the ventilator and observing for signs of respiration.lf the criteria for brain death are rnet, theD an EEG or ccrebral blood-flow stud)'arenot necessary.In other words, the clinical criteria of the absenceof breathins and brainstcmreflexesaremore important than an EEG.This is becauseEECactivitywou]d be of limited mcaning if a patier.rtmects tl.reclinical brain-death crjteria. Braindcath should only be determined to be present if you have excluded other causes of markedly decreasedbrainstem and rcspiratory function. you must be certain that the patientis not suffering fron an overdoseof barbiturates,hypothermia, hypotension, or the useof neuromuscularblockingagentssuclraspancuroniumrvecurontum,or succinvlcho line.Thesecanall simulatebrain death. For example, a 35-year-old yroman is admitted aftcr having a serzurear a parr)i. HerheadCT scanshowsan intracranialbleed.Sheis intubatedbecauseof the loss of spoDtaneousrespiration.Therearc no pupilary,corneal,oculocephalic,or cold caloric reflexeselicited. Which of the follorving shoultl you clonext? I ( A P L A N H MEDICAL
  • 46. r | $rapterrive a, b. c. d. e. EEG EKG Urine toxicologyscreen Psychiatriccvaluation Ethicscommitteeevaluation This patient hasmet most of the criteria for brain death.An EKG j nosedeathat anypoint. Auscultationof the heartis all ,h", i, ..;'"irt":",:t::::t#tlf: stoppingof the hcart'you shouldexcrucreintoxicationwith cNs depressantdrugsandhypothermiaprior to determiningthatthe patientis brain dead. Brain deathdoesnot specificallyrequiredeterminationby a neurologistif the physician managingthe patientis comfortablewith the criteriadescribedanclhow to verify them.Thisis similarto not needingapsychiatristin orderto determinccapacity.If thepatientisclearlynot braindeadbecausetheirpupilsarcreactiveor theyhavespontaneousbreathing, a neurologistis unnecessary. If thepatientisbrain dead,thentheyaredead.Thephysiciandoesnor needa court orderor a relative'spern.rissionto removelife support.Thedoctordoesnot needto askanyone,spermissionto stop the ventilatoror.other treatment,althoughUSMLEwill alwayswantyou to answer"discuss with thefamily,,',,explainyour finding.s,,,or ..build consensus,,firstratherthanjust turning offthe ventirator.Thismaysee- .nritrudi.tory.voushouldalwaysexplainwhat brain deathmeansto the patient,sfanily. you needto answer.,explainthemeaningof brain deathto the family,,asthe first choiceif it appears. on the otherhand'the farnilyi permissionor conscntis not requiredfor terminatinglifesupport,becausea personwhois brain cleadis considereddead.Insurancecompanreswillnot pay for the hospitalizationor managcmentof thosepatientswl.roaredead.you canharvestorgansfor transplantationfrom abrain-deadpersonif thefamilyconsents.you donot haveto waitfor theperson,sheartto stop.Actually,it ispreferableto removetheorganswhiletheheart isstill beatingbecausetheviabilityof th" t."nrplur.,t.aorganstronglycor_relatesto how longit wasunperfusedafterremovalfrom the donor,sbodv- A te c( sl te at Pi P( AI n( T] th at In of fe !(AplAN ) MFDtcAl
  • 47. l + r Chapter6: Reproductivelssues ABORTION Ar adultwoman hasan unrestricted acccssto abortion through the end ofthe first trimes- terFirst-trimesterabortions are clearly unrestricted. Women do not need the approval or consentofanyone elseto obtain a first-trimester abortion. In the secondtrimester the deci_ sionisstill betweena woman and her physician, but the easeof accessto a second trimes_ terabortionis lessclear Statesmay place regulations on free accessto a second_trimester abortion.However, second-trimester abortions arc preponderantly still performed at the patient'sdiscretion. Third-trimester abortions are not freely available,becausethe fetus is p'tentiall)'viable.Third-trimester abortions are clearryrestricted.consent by the father for anabortionis not required; the fetus is consideredasa part of the woman,sbody antl does nothavethe individual rights of,personhood' until after birth. Thereis no compulsion on the part of the physician to perform an abortron if performing thisprocedureis ethically unacceptableto the physician. The patient hasa right to have an abortion,but they don't have a right to force you to do it, if it is objectionable to you. Inaddition,it is considered unethical for a patient to seekan abortion for the purposes otgendcrselection.It is considered ethically unacceptableto creterminethe gender of the fetusandthen abort the fetus if the sexin unacceptableto the patient. Forexample,a 23-year-old woman contesto your office seekingan abortion. you performgynecologicproceduresand you havebeen trained to d-oabortions in the past.However,you no longer find it morally acceptableto perform the procedure althoughyou know how. The paticnt is insisting on having the abortion and is angrywith you for "abandoning your parjcnr...,!'hatshouldyou tell her?
  • 48. 42 | chaptersix O: I q:"*t rule,if therc is a procedurethat the patientwantsbut that you do not feelethicallycomfortableperfornir.rg,you shouldrcferthe patientto onoth.. physician.you cannotbecompelredto accepta paticntyou d., not wantor to do aprocedurewith rvhich ),oudo not agrce.J'hephysicianmustvoluntarilyagreeto the relationship. CONTRACEPTION Thcre is no limitation on the accessto contraception for either a lran or a wontan. lt iscntirelyat the discretion of the patient. This is equally t..r. fo, ,rri,.,o.r. Contraception isone of the issuesfor which a n.rin, is not necessaryto ot,anir,'.onrro.lJ,ls considered partially emancipated Parental consent STERII.IZATION Both women and men have free accessto sterilization. Consent is or. y necessaryfrom thepatient. Spousalconsentis not I inslaborti,o',.on.,u."p,io,,,ona",:J.fi1,*'*l,jIl';#J:ri:..Tft:::il:?:f;spouse.Each person has autonomv over hr.sor her own bodv. MINORS l"j:11^.:.1:, l, not necessaryfor cithercontraceptionor prenatatcare.ln casesinvolr-lng reproductiveissuesfor minor-s,the ansrverrvi te eitherto just treatthc minor or to-encourage discussion,'with the usMrF;r,t'r;;;;r";;:;::".*:1,':il1"J*l:..."x,:;ff:T::i[?:::,.,,J;issuesinvolving minors. The rules on parental consent in the caseof abortion are lessclear.Some statesrequire parental consent for abortions and others do not. BecauseUSMLE isa national exanrination and the rules on parental _r*n, f'J"rion vary frorn state tostate,there cannot be a singleanswerthat cither says,,noparental consent,,or ,,Ies, parentalconsent is necessary.,,Therefore, the ansrverwill be,.eniourage the patient to discusstherssuelvith parents.', D T Ir T] d( I ( A P L A N MEDICAL
  • 49. Reproductivetssues I DONATIONOFSPERMANDEGGS #::il,::,:T:1."1i,:l"i:"j::::::"J^':::10'l',r':donatespernandunfertirizedeggsIn addition, paymeut nlay be recei, , - --.---"t sl'crrD ano untertrfized eggs. rvords,there is nn l.-.r /eo ror spcrm ilncl unti:rtilizecl cgg donations. In ot_h"errvords,thereis no legalor ethicalc, -' Lcggoonattons.ln lherr.ic h^-,-.,-_ onrrnndr.rtion t,r sellingsperntand unfertilized donated,but not sold. Thercis, however,a prohibition .. '' Jlrurr. sPcror ano un .loner.rr.,,+_^. - rr Lgainstselling tertilizecleggs.Fertiljzcd eggsmayy b e lo rot feel lclan.you 'ith rvhich nan.It is eptionis consent romthe includ r of thc nvolr- r o r t o essarl) uctive clear. ILEis ateto rental ;sthe qj:19 MTDICAL
  • 50. Chapter7. OrganandTissueDonation l 4 s AUTONOMYOFTHEDONOR Organand tissuedonation is a voluntary event entirely at the discretion of the live donor, Theprinciple of autonomy is fully in play here. For example, a 35-year-old-man is dying of hepatic failure. His brother is fully HLA rnatched and a highly compatible donor. There are no other donors at this time and the patient will likely not survive long enough to find another donor. You arescreeningthe brother for the donation, but hc is not willing to undergo the surgeryfor the partial donation.What shouldyou do? Thereis nothing you or anyone-including a court of law can do to compel a person to donatean organ or tissueif he clearly choosesnot to do so.The needof the recipient hasno impacton mandatin€la donor to donate.This is true evenif the donation is uncomplicated forthe donor and the recipient will die without it. ORGANDONORNETWORKASKSFORCONSENTFORDONATION For example, a 30-year-old woman is your patient in the intensive care unit for respiratory failure. The patient hashad a motor vehicle accident and hassustained a massiveintracranial hemorrhage. The patient is brain dead and will be removed from the ventilator. You know that there are numerous patients in your hospital waiting for organs. The family of the patient is with you. You have an excellent relationship with the family and they trust you. What should you do about the donation? I(APLA'I MFDICAL
  • 51. 46 I chapterseven Only the organ donor neturcrk or uniform network for organ sharing shoulci obtain con_ sent for an organ donation. The medical tcant taking care of the patient shoukl not ask for the donation. Even if your relationship witr.r the familv is excelent, the organ donor network hasan enormouslygreatersuccessrate in obtaining consent.physiciansthat ask for consent for organ donatiolr are far rnore likely to be refused.Becausea greater number .f refusalswould leadto a lossof potentiarorgansfor donation when the shortagestatute of availableorgansis critical,it is againstthe raw f.r doctors to obtain this consent.By legalstatute,only thosespecificallytrained to obtain consentfor orqan donation should approachthe family for this consent. In addition, thereis thc perceptionof an enormous conflict of intereston the part of the family n'hen a caregiverattempts to obtain consent.{4ren a caregivcr asksfor consent,it leadssome familiesto believethat the health-careteam is not doing everythingpossible to preservethe lifc of the paticnt. This makes it scer' that the priority of the health-care team is to obtain organs. It is essentiarfor the hcarth-care teanl to preserve its reration ship rvith thc family asthe advocatesfor prcserving the rife of the patient. In addition, the organdonation networkhasa much greaterchanccof obtainingcolscnt, soif the primarv health careteamtries to obtain consent,it cor.rldlcad to a lossof neededorsans. PAYMENTFORDONATIONS With the exception of renewatrletissuessuch as spermr unfertilizcd eggs,and blood, pay_ n.rcntfor organs is consideredethicarly unacceptable.peoplc must not bc in the busincssof sellingorgans.The economicaspectsof organ clonationmust be ninimizecl so that pcople believethat the patients who need organs the most w I get them, not that the rvealthvwill get preferential treatment. It is,howeveq acceptableto cover the costto the donor of dona- tior.r. 'l here is a diff'erencebetwecn reimbursing the donor for the cost of donation and creatinga finalcial incentivcfor pcopleto,,sell',organs. ORGANDONORCARDS Although an organ donor card givesan indication of a patient'swishcs for donation, fam ily consent is still necessaryfor donation. Family objection can overrule the organ donor card. P II o S rl r( Pi al ca SC ir P tl rl P P lc tl Tr tr fc al bi !t,qptl N) N4r orc.Ar
  • 52. 47 n con ot ask donor Latask rnbcr ;latutc nt. By hould of the ent,tt rssible r-care ttion- n,the LlTlAIV pay- :ssof eople ywill ona- L a n d Physiciansand laboratoriesaremandatedto report a nunber of medicaiillnesses.The mai'purposein reportinginncssesisbothepicremiologicaswenasto rnterruptthespread ofcertaincommunicablediseases.The illnessesthat arealrval.sreportableincludeAIDS, q'philis,tuberculosis,gonorrhea,andallofthe childhooddiseasessuchasmeaslcs,mumos. rubella'andpertussis.Thc list of other reportablediscasesis extensiye.Thesediseasesare reportableby sonrebody,not necessarilythephysician. Phvsiciansarealwayslegallyprotectedfor participatingin partnernotification.In gcneral, thehealthdepartmentperformsthe majority of contact_tracingeventsaswellasnotilying thosethathavebeenilr closecontactof thepossibity of infection. 'fhe nameof theso,.rrce patientisalwaysprotected. Partnernotificationexistsfor diseasessuchasHIV/AIDS,syphilis,gonorrhea,andtubercu losis.In addition,thehealthdepartmentcanincarceratepatientswith tuberculosisto pre r,entthespreadof diseasc.If a patientwon,ttell his or herpartner,thenyou must ansu/er thatyou must follow your duty to report.If the sourcepatiert still won,ttell his or her partner,youarewithin your legarright to tell theinnocentthird party.partnernotification andreportableillnessesarcanexampleof oncof thefewtimesthatthc patrentsautonomy canbesupersededbecauseofthe necessityofprotectingothcrs.My rightto autonomyends rvhereyour safetybegins.I havean absoluteright to privacy,exceptwhen my restaurant ser'esfoodinfectedwithsarmonera,andthenthepatient'srightstoautonomybecomeress importantthanprotectingothersfron harm. Tuberculosishasspecialreportingand public healthissues.In additronto doing contact tracingofthe contactsin orderto do ppD testing,thereisthespeciarrssueof incarceration fortuberculosis.Patientswith tubercurosisshouldbeisolatedfor abouttwo weeks,whichis approximatelytheamountof time thatistakesfor sputumto becomenegativefor acid-fast bacillilf a patientrefusesto takeantituberculosistherapy,physicianshavethe option of ChapterB: Reportablelllnesses [am lnor MTDICAL
  • 53. | .r'uo,"rrirr', lrcarceratingthepatientto preventthemfrom spreadingthedisease.Thisis onlyfor thosewho stiJlhavepositivestainsof their sputumfor acid_faitbacilli. Incarcerationfor tuberculosisis not thesamething asbeingarrested.It hasnothingtod0with the criminal-justicesystem.The incarcerationo..ur,-in a hosprtalnot in a prison or jail. Youcannotforce-feedtuberculosismedications,but you canpreventpeoplefromwalkingat their leisurein the community to spreaddisease.Incarcerattonis a lastresort andis.onlyusedaftera'other optionshavebeenexhaustedin termsot havingdiscussions with thepatientandofferingdirectlyobservecltherapyat home. co A s phv ity r ther hen wh add aut( test Pre POS Altl nan righ HI ll{!!P urorcer-
  • 54. l r s l d o ison rom SOIt ions Chapter9: HIV-Relatedlssues CONFIDENTIATITY Aswith all medical information there is a presumption of confidentiality on the part ofthe phvsician.Becauseof the social stigma of HIV there is an additional layer of confidential- ity and consent required. When a patient enters the hospital or other health-care facility thereis general consent given that allows the routine testing of blood for chernistry and hematologyand so on. There is an additional HIV-related consent required to testfor HIV. hen a patient signs a releaseto distribute or transmit medical information there is an additionalconsent required for HIV or AIDS-related information. You cannot mandate automaticHIV testing of patients without their specificinformed consent that you will be testingfor HIV For example, a woman com€s at 10 weeksof pregnancy for prenatal care.Shehas a historv of sexually transmitted diseasessuch asgonorrhea. You offer HIV test- ing,which the patient refuses,asa routine part of prenatal care.Shereturns at 14 and l8 weeksof pregnancy but is still refusing becauseof anxiety that shemay be positive.You inform the patient that there are medications that can reduce trans- mission from mother to child to lessthan 2 percent. She persistsin her refusal. 4rat should you do now? Althoughthere are medications to prevent transmission of HIV to the fetus during preg- nancl,you cannot compel mandatory testing of pregnant women. The woman has the right to refuse testing as well as to refuse antiretrovirals. Therefore, you should offer HIV testing universally to all pregnant women-but there is no mandatory testing of the pregnantwoman without her expressconsent to do so.If the woman is found to be HIV- positiveyou cannot mandate the use of antiretrovirals even though they are safe and I ( A P L A N MEDICAL
  • 55. T] til 50 | chapterNine !l!!!p rureorcar- effective in preventing transmission of the virus from mother to ch d. Althoueh from time to time, there is aberrancyin the Jegalsysten.rthat tries to prosecutea drug_u.ingor alcohol-using pregnant woman, the autonomy of the mother lcgalry outweighs thc safety of the fetus. For example, an HlV-positive tvoman comes to labor and clelitery at 40 weeks of pregnancy.Shehas a very low CD4 coult (lessthan 50) and a high viral load (more than 500,000).You offer her a Caesariansection and intravenous zidovu_ dine, which can cut the transmission rate in half even on the day of delivery. The woman is anxious, but clearly has the capacity to understand the implications of this decisionon both her healthand the healthofher child.Sheis stirrrefusinsthe C sectionand medications.What should you do next? Fortunately this circumstanceis rare and the HIV perinatar transmission rate in the united Statesis well undcr 5 percent.However,a rvoman'sright to chooseher own forms ofhealth careare consideredsuperior to virtually all other treatment concerns.The wrong answerin a question like this would be to give the medications an1.way,to gct a court order to compel the patient to take the zidovudine, to askthe father for consentfor either the zidovudine or the C section, or to sedatethe patient and pcrforn the C sectior.r. The autonomy of the mother is regarlvsuperior to beneficencefor the fetus. Althoush a 40- lveekfetus is a viable child, the fetus is still inside the woman's body and doesnt become a person until it is delivered.A noman has the right to refuse HIV testing in pregnancl, to refuseantiretroviral medicati.ns in pregnancy,ancito refuse a c-section even if it will markedly benefit the child. PARTNERNOTIFICATION Thc highlevelof confidentialityconcerningHIV canonlybebreachedunderveryspecific cucumstancessuchaswhenthe healthof a third party is at risk.A crrcumstancesuchas thiswouldbervhenanHIV-positivepersonhasasexualor needle-sharingpartnerthatisat risk.Themethodof notificationfollowsthestepsof first counseringapatientto notiryhis partnersvoluntarily'This wourdbeidealandfollowsthegeneralthemeof uSMLE,rvhich is to first answer"encourage discussions,,when listedasoneof the choices.If the patient is eitheremotionallyunableor unwilling to notj$ their partnersthenext stepis to notiry the Departmentof Healthto startthe processof contacttracing.Thehealthdepartment interviewsthepatientand attemptsto constructa list of partnersin orderto notify them. Pi ol T tt rf t( P y( yl st s( t t v s t